You are on page 1of 89

JURISDICTION; FOR CASE

DIGEST

Alfeo Vivas, on his behalf and on behalf of


the Shareholders of the Eurocredit
Community Bank, Petitioner, vs. The
Monetary Board of the BSP and the
PDIC, Respondents.
GR No. 191424; August 7, 2013

Facts: The Monetary Board placed the Eurocredit


Community Bank under Prompt Corrective Action
framework on account of the findings of serious
findings and supervisory concerns. Vivas moved
for the reconsideration of such action. ECBI also
unjustly refused to allow the BSP examiners from
inspecting its books and records. The MB issued
Resolution No. 276 placing ECBI under
receivership, because of its inability to pay its
liabilities, insufficient realizable assets and
violation of cease and desist order of the MB for
acts constituting unsound banking practices.
Vivas argued that the MB committed grave
abuse of discretion for placing ECBI under
receivership without prior notice and hearing,
pursuant to RA 7353, Sec. 11.

Issue: Whether or not the MB committed grave


abuse of discretion in placing ECBI under
receivership without notice and hearing.

Ruling:
No, the MB did not gravely abuse its
discretion. The ECBI was given every chance to

be heard and improve its financial standing.


Moreover, the MB has the power to forbid a bank
from doing business and place it under
receivership without prior notice and hearing,
when the circumstances warrant it. Under RA
7653, the MB was given with more power of
closure and placement of a bank in receivership
for insolvency or if the continuance in the
business would result in the loss of depositors or
creditors. The close now, hear later doctrine
was justified on practical and legal
considerations to preclude unwarranted
dissipation of the banks assets and as valid
exercise of police power to protect creditors,
depositors, stockholders and the general public.

Llamas vs Court of Appeals

Before this Court is a Motion for


Reconsideration filed by herein petitionerspouses Francisco R. Llamas and Carmelita C.
Llamas. On September 29, 2009, this Court
promulgated a Decision[1] in the above-captioned
case, denying the petition for Annulment of
Judgment
and Certiorari,
with
Preliminary
Injunction filed by petitioners. Petitioners are
assailing the decision of the Regional Trial Court
(RTC) of Makati City convicting them of the
offense Other Forms of Swindling punishable
under Article 316, paragraph 2, of the Revised
Penal Code (RPC).

Briefly, the antecedent facts are as follows:

On August 14, 1984,


petitioners were charged before
the Regional Trial Court (RTC)
of Makati with, as aforesaid, the
crime
of
other
forms
of
swindling in the Information,
docketed as Criminal Case No.
11787, which reads:

That on
or
about
the
20th day
of
November, 1978,
in
the
Municipality
of
Paraaque, Metro
Manila,
Philippines, and
within
the
jurisdiction of this
Honorable Court,
the above-named
accused,
conspiring
and
confederating
together
and
mutually helping
and aiding one
another,
well
1|P a g e

JURISDICTION; FOR CASE


DIGEST

knowing
that
their parcel of
land known as
Lot No. 11, Block
No. 6 of the
Subdivision Plan
(LRC) Psd 67036,
Cadastral Survey
of
Paraaque,
LRC Record No.
N-26926,
Case
No.
4896,
situated at Barrio
San
Dionisio,
Municipality
of
Paraaque, Metro
Manila,
was
mortgaged to the
Rural Bank of
Imus, did then
and
there
willfully,
unlawfully
and
feloniously
sell
said property to
one Conrado P.
Avila,
falsely
representing the
same to be free
from all liens and
encumbrances
whatsoever, and
said Conrado P.
Avila bought the
aforementioned
property for the
sum
of P12,895.00
which was paid

to the accused,
to the damage
and prejudice of
said Conrado P.
Avila
in
the
aforementioned
amount
of P12,895.00.

Contrary
to law.

After trial on the merits,


the RTC rendered its Decision on
June 30, 1994, finding petitioners
guilty beyond reasonable doubt
of the crime charged and
sentencing them to suffer the
penalty of imprisonment for two
months and to pay the fine
of P18,085.00 each.

On appeal, the Court of


Appeals, in its February 19, 1999
Decision in CA-G.R. No. CR No.
18270, affirmed the decision of
the trial court. In its December
22,
1999
Resolution,
the
appellate court further denied

petitioners
motion
reconsideration.

for

Assailing the aforesaid


issuances of the appellate court,
petitioners filed before this Court,
on February 11, 2000, their
petition for review, docketed as
G.R. No. 141208. The Court,
however, on March 13, 2000,
denied the same for petitioners
failure to state the material
dates. Since it subsequently
denied petitioners motion for
reconsideration on June 28,
2000, the judgment of conviction
became final and executory.

With
the
consequent
issuance by the trial court of the
April 19, 2001 Warrant of Arrest,
the police arrested, on April 27,
2001, petitioner Carmelita C.
Llamas for her to serve her 2month jail term. The police,
nevertheless, failed to arrest
petitioner Francisco R. Llamas
because he was nowhere to be
found.

On
July
16,
2001,
petitioner Francisco moved for
2|P a g e

JURISDICTION; FOR CASE


DIGEST

the lifting or recall of the warrant


of arrest, raising for the first time
the issue that the trial court had
no jurisdiction over the offense
charged.

There being no action


taken by the trial court on the
said
motion,
petitioners
instituted, on September 13,
2001, the instant proceedings for
the annulment of the trial and
the appellate courts decisions.

The
Court
initially
dismissed on technical grounds
the petition in the September 24,
2001 Resolution, but reinstated
the
same,
on
motion
for
reconsideration, in the October
22, 2001 Resolution. [2]

In its September 29, 2009 Decision, this


Court held that, following the ruling in People v.
Bitanga,[3] the remedy of annulment of judgment
cannot be availed of in criminal cases. The Court
likewise rejected petitioners contention that the
trial court had no jurisdiction over the case.

Petitioners are now before this Court


seeking the reversal of the September 29, 2009
Decision and, consequently, the annulment of
their conviction by the trial court. In their Verified
Motion for Reconsideration,[4] petitioners ask this
Court to revisit and take a second look at the
issues in the case without being unduly
hampered by any perceived technical shortfalls
of a beleaguered innocent litigant. In particular,
they raise the following issues:

1.
WITH
ALL
DUE RESPECT, AND IN LIGHT OF
THE CORRECT APPLICATIONS OF
DOCTRINAL
JURISPRUDENCE,
PETITIONERS
HAD
PURSUED
THEIR MORE THAN TWENTY FIVE
(25) YEARS QUEST FOR JUSTICE
AS INNOCENT MEN, AND HAD
HONESTLY MAINTAINED THAT
THEIR RESORT TO REVERSE, SET
ASIDE AND/OR ANNUL, IS IN LINE
WITH
JURISPRUDENCE
AND
LAW, ANY
TECHNICAL
SHORTFALLS [OR] DEFECTS
NOTWITHSTANDING[;]

2.
WITH
ALL
DUE RESPECT, AGAIN IN LIGHT
OF APPLICABLE JURISPRUDENCE
ON THE ISSUE OF JURISDICTION,
PETITIONERS ARE NOT BARRED
FROM RAISING SUCH QUESTION
OF JURISDICTION AT ANY TIME
AND IN FACT MAINTAIN THAT

RESPONDNET COURTS HAD NO


JURISDICTION IN
LAW
AND
ENLIGHTENING DOCTRINES TO
TRY AND DECIDE THIS CASE;

3.
AGAIN WITH
ALL
DUE
RESPECT
AND UNFORTUNATELY,
THE
VERY JUSTIFYING MERITS OF
PETITIONERS APPROPRIATE
INSTANT
REMEDY; HAD
NOT
CONSEQUENTLY BEEN PASSED
UPON,
TO
UPHOLD
THE
PARAMOUNT
CONSTITUTIONAL CHERISED
MANDATE, THE PRESUMPTION
OF
INNOCENCE
MUST
BE
UPHELD, EXCEPT ONLY UPON
ESTABLISHED AND ADMISSIBLE
EVIDENCE BEYOND REASONABLE
DOUBT; AND

4.
PETITIONER
S VERY HUMBLY BESEECH THIS
HONORABLE COURTS HIGHEST
SENSE
OF
MAGNANIMITY,
UNDERSTANDING,
JUDICIOUS
WISDOM AND COMPASSION, SO
THAT JUSTICE MAY TRULY AND
JUSTLY BE RENDERED IN FAVOR
OF PETITIONERS AS IT MUST,
GIVEN THE VERY UNIQUE AND
COMPELLING
JUSTIFICATIONS
HEREOF[.][5]
3|P a g e

JURISDICTION; FOR CASE


DIGEST

Petitioners likewise pray for a referral of


the case to the Court En Banc for oral argument
or to be allowed to submit written supplementary
pleadings for them to state the compelling
reasons why their motion for reconsideration
should be allowed.

tools designed to facilitate the attainment of


justice. The strict and rigid application of rules
that tend to frustrate rather than promote
substantial justice must always be avoided. It is
far better and more prudent for the court to
excuse a technical lapse and afford the parties a
review of the case to attain the ends of justice,
rather than dispose of the case on technicality
and cause grave injustice to the parties.[7]

Next, we proceed to resolve


substantive issues raised by petitioners.

the

Article 316 (2) of the Revised Penal Code


In the interest of justice and for
humanitarian reasons, the Court deems it
necessary to re-examine this case.

Admittedly,
petitioners
took
many
procedural missteps in this case, from the time it
was pending in the trial court until it reached this
Court, all of which could serve as enough basis to
dismiss the present motion for reconsideration.
However, considering petitioners advanced age,
the length of time this case has been pending,
and the imminent loss of personal liberty as a
result of petitioners conviction, the Court
resolves to grant pro hac vice the motion for
reconsideration.

This Court has, on occasion, suspended


the application of technical rules of procedure
where matters of life, liberty, honor or property,
among other instances, are at stake.[6] It has
allowed some meritorious cases to proceed
despite inherent procedural defects and lapses
on the principle that rules of procedure are mere

states:
This Court notes that the case was
allowed to run its course as a petition
for certiorari, such that in its April 12, 2004
Resolution, it said Considering the allegations,
issues and arguments adduced in the petition for
review on certiorari x x x. Likewise, in its
February 10, 2003 Resolution,[8] the Court said,
It appearing that Atty. Francisco R. Llamas, in
his own behalf and as counsel for petitioners, has
failed to file their reply to the Solicitor Generals
comment
on
the
petition
for
review
on certiorari within the extended period x x x.

ART. 316. Other forms of


swindling.
The
penalty
of arresto mayor in its minimum
and medium periods and a fine
of not less than the value of the
damage caused and not more
than three times such value,
shall be imposed upon:

xxx
Thus, the Court, at the first instance, had
recognized that the petition, although captioned
differently, was indeed one for certiorari.

Since we have resolved to treat the


petition as one for certiorari, the doctrine
in People v. Bitanga[9] no longer finds application
in this case.

2.
Any
person
who,
knowing that real
property
is
encumbered,
shall dispose of
the
same,
although
such
encumbrance be
not recorded;
4|P a g e

JURISDICTION; FOR CASE


DIGEST

xxx

In every criminal prosecution, the State


must prove beyond reasonable doubt all the
elements
of
the
crime
charged and the
complicity or participation of the accused.[10]

For petitioners to be convicted of the


crime of swindling under Article 316 (2) of the
Revised Penal Code, the prosecution had the
burden to prove the confluence of the following
essential elements of the crime:

1. that the thing disposed of be


real property;
2. that the offender knew that
the real property was
encumbered,

One of the essential elements of


swindling under Article 316, paragraph 2, is that
the act of disposing the encumbered real
property is made to the damage of another. In
this case, neither the trial court nor the CA made
any finding of any damage to the offended party.
Nowhere in the Decision of the RTC or that of the
CA is there any discussion that there was
damage suffered by complainant Avila, or any
finding that his rights over the property were
prejudiced.

On the contrary, complainant had


possession and control of the land even as the
cases were being heard. His possession and right
to exercise dominion over the property was not
disturbed. Admittedly, there was delay in the
delivery of the title. This, however, was the
subject of a separate case, which was eventually
decided in petitioners favor.[12]

whether the encumbrance is


recorded or not;
3. that there must be express
representation by
the
offender
that
the
real
property
is
free from
encumbrance; and
4. that the act of disposing of
the real property be made to
the damage of another.[11]

If no damage should result from the sale,


no crime of estafa would have been committed
by the vendor, as the element of damage would
then be lacking.[13] The inevitable conclusion,
therefore, is that petitioners should be acquitted
of the crime charged.

WHEREFORE, the foregoing premises


considered, the Motion for Reconsideration
is GRANTED. The assailed Decision dated
September 29, 2009 is SET ASIDE and a new
one is entered ACQUITTING petitioners of the
crime charged on the ground of the prosecutions
failure to prove their guilt beyond reasonable
doubt.

SO ORDERED.

DEPUTY DIRECTOR GENERAL ROBERTO


LASTIMOSO, ACTING CHIEF PHILIPPINE
NATIONAL POLICE (PNP), DIRECTORATE FOR
PERSONNEL AND RECORDS MANAGEMENT
(DPRM), INSPECTOR GENERAL, P/CHIEF
SUPT. RAMSEY OCAMPO and P/SUPT. ELMER
REJANO, petitioners,
vs.
P/SENIOR INSPECTOR JOSE J.
ASAYO, respondent.

AUSTRIA-MARTINEZ, J.:
Before the Court is respondents Motion for
Reconsideration of the Decision promulgated on
March 6, 2007. In said Decision, the Court
granted the petition, holding that the Philippine
National Police (PNP) Chief had jurisdiction to
take cognizance of the civilian complaint against
5|P a g e

JURISDICTION; FOR CASE


DIGEST

respondent and that the latter was accorded due


process during the summary hearing.
Facts: Respondent argues that the decision
should be reconsidered for the following reasons:
1. The summary proceeding was null and void
because no hearing was conducted; and
2. The evidence presented at the summary
hearing does not prove that respondent is guilty
of the charges against him.
Respondent insists that the summary hearing
officer did not conduct any hearing at all but only
relied on the affidavits and pleadings submitted
to him, without propounding further questions to
complainant's witnesses, or calling in other
witnesses such as PO2 Villarama. It should,
however, be borne in mind that the fact that
there was no full-blown trial before the summary
hearing officer does not invalidate said
proceedings. In Samalio v. Court of Appeals,1 the
Court reiterated the time-honored principle that:
Due process in an administrative context
does not require trial-type proceedings
similar to those in courts of justice. Where
opportunity to be heard either through oral
arguments or through pleadings is accorded,
there is no denial of procedural due process. A
formal or trial-type hearing is not at all
times and in all instances essential. The
requirements are satisfied where the parties are
afforded fair and reasonable opportunity to
explain their side of the controversy at hand. The
standard of due process that must be met in
administrative tribunals allows a certain degree
of latitude as long as fairness is not ignored. In

other words, it is not legally objectionable


for being violative of due process for an
administrative agency to resolve a case
based solely on position papers, affidavits
or documentary evidence submitted by the
parties as affidavits of witnesses may take
the place of their direct
testimony.2(Emphasis supplied)
Ruling: The first issue presented by respondent
must, therefore, be struck down.
To resolve the second issue, respondent would
have the Court re-calibrate the weight of
evidence presented before the summary hearing
officer, arguing that said evidence is insufficient
to prove respondent's guilt of the charges
against him.
However, it must be emphasized that the action
commenced by respondent before the Regional
Trial Court is one for certiorari under Rule 65 of
the Rules of Court and as held in People v. Court
of Appeals,3 where the issue or question involved
affects the wisdom or legal soundness of the
decision not the jurisdiction of the court to
render said decision the same is beyond the
province of a special civil action for certiorari.
Yet, respondent-movant's arguments and the fact
that the administrative case against respondent
was filed way back in 1997, convinced the Court
to suspend the rules of procedure.
The general rule is that the filing of a petition
for certiorari does not toll the running of the
period to appeal.4

However, Section 1, Rule 1 of the Rules of Court


provides that the Rules shall be liberally
construed in order to promote their objective of
securing a just, speedy and inexpensive
disposition of every action and proceeding.
InGinete v. Court of Appeals5 and Sanchez v.
Court of Appeals,6 the Court saw it proper to
suspend rules of procedure in order to promote
substantial justice where matters of life, liberty,
honor or property, among other instances, are at
stake.
The present case clearly involves the honor of a
police officer who has rendered years of service
to the country.
In addition, it is also understandable why
respondent immediately resorted to the remedy
of certiorari instead of pursuing his motion for
reconsideration of the PNP Chiefs decision as an
appeal before the National Appellate Board
(NAB). It was quite easy to get confused as to
which body had jurisdiction over his case. The
complaint filed against respondent could fall
under both Sections 41 and 42 of Republic Act
(R.A.) No. 6975 or the Department of the Interior
and Local Government Act of 1990. Section 41
states that citizens' complaints should be
brought before the People's Law Enforcement
Board (PLEB), while Section 42 states that it is
the PNP Chief who has authority to immediately
remove or dismiss a PNP member who is guilty of
conduct unbecoming a police officer.
It was only in Quiambao v. Court of
Appeals,7 promulgated in 2005 or after
respondent had already filed the petition
for certiorari with the trial court, when the
Court resolved the issue of which body has
6|P a g e

JURISDICTION; FOR CASE


DIGEST

jurisdiction over cases that fall under both


Sections 41 and 42 of R.A. No. 6975. The Court
held that the PLEB and the PNP Chief and
regional directors have concurrent jurisdiction
over administrative cases filed against
members of the PNP which may warrant
dismissal from service, but once a complaint is
filed with the PNP Chief or regional directors, said
authorities shall acquire exclusive original
jurisdiction over the case.
With the foregoing peculiar circumstances in this
case, respondent should not be deprived of the
opportunity to fully ventilate his arguments
against the factual findings of the PNP Chief. He
may file an appeal before the NAB, pursuant to
Section 45, R.A. No. 6925. It is a settled
jurisprudence that in administrative proceedings,
technical rules of procedure and evidence are
not strictly applied.8 In Land Bank of the
Philippines v. Celada,9 the Court stressed thus:
After all, technical rules of procedure are not
ends in themselves but are primarily devised to
help in the proper and expedient dispensation of
justice. In appropriate cases, therefore, the rules
may be construed liberally in order to meet and
advance the cause of substantial justice.10
Thus, the opportunity to pursue an appeal before
the NAB should be deemed available to
respondent in the higher interest of substantial
justice.
WHEREFORE, respondent's Motion for
Reconsideration is partly GRANTED. The
Decision of the Court dated March 6, 2007
is MODIFIED such that respondent is hereby
allowed to file his appeal with the National

Appellate Board within ten (10) days from finality


of herein Resolution.

Garcia vs. Sandiganbayan 460 SCRA 588 June


22, 2005, TINGA
Facts: -Major General Carlos F. Garcia was the
Deputy Chief of Staff for Comptrollership of the
AFP.

Ombudsman before the SB vs. Garcia, his wife


and 3 sons: Ombudsman, after conducting
inquiry (similar to PI) has determined a prima
facie case exists vs. Maj. Gen Garcia since during
his incumbency as a soldier and public officer he
acquired huge amounts of money and properties
manifestly out of proportion to his salary as such
public officer and his other lawful income SB
GRANTED PETITION, ISSUED WRIT OF
PRELIMINARY ATTACHMENT
-Garcia filed MTD then this PETITION (same day):

-On Sept27, 2004, Atty. Maria Olivia Roxas, Graft


Investigation and Prosecution Officer of the Field
Investigation Office of the Office of the
Ombudsman, after due investigation, filed a
COMPLAINT vs. Garcia for VIOLATION OF

a.LACK OF JURISDICTION over forfeiture


proceedings (CIVIL ACTION) under RA 1379
should be w/ RTC as provided under SEC2(9) of
the law

1. SECTION 8 (IN RE Section 11) of RA 6713(Code


of Conduct of Ethical Standards for Public
Officials and Employees)

b. Sandiganbayans jurisdiction in Civil Actions


pertains only to separate actions for recovery of
unlawfully acquired property vs. Pres. Marcos etc.

2. Art 183, RPC

c. Sandiganbayan was intended principally as a


criminal court

3. Sec52(A)(1), (3) & (20) of the Civil Service Law


-based on this complaint, a case was filed vs.
Petitioner
-Wife and 3 sons were impleaded for violation of
RA 1379 insofar as they acted as conspirators,
conduits, dummies and fronts of petitioner in
receiving, accumulating, using and disposing of
ill-gotten wealth
-Also, a PETITION W/ VERIFIED URGENT EX PARTE
APPLICATION FOR THE ISSUANCE OF A WRIT OF
PRELIMINARY ATTACHMENT was filed by

BASIS: Presidential issuances and laws


d. Granting that SB has jurisdiction, petition for
forfeiture is fatally defective for failing to comply
with jurisdictional requirements under RA 1379,
SEC2:
i. inquiry similar to a PI
ii. Certification to SOLGEN of prima facie case
here: no certification
iii. action filed by SOLGEN - here: by Ombudsman
7|P a g e

JURISDICTION; FOR CASE


DIGEST

COMMENT by Sandigaybayan:

COMMENT BY OMBUDSMAN:

1.Republic vs. SB: there is no issue that


jurisdiction over violations of [R.A.] Nos. 3019
and 1379 now rests with the Sandiganbayan.

1. Republic vs. SB

2. Under Consti and prevailing statutes, SB is


vested w/ authority and jurisdiction over the
petition for forfeiture under RA 1379
3. Section4a(1), PD 1606, not Section 2(9), RA
1379 should be made the basis of SBs
jurisdiction:
a. Violations of Republic Act No. 3019, as
amended, otherwise known as the Anti-Graft and
Corrupt Practices Act, Republic Act No. 1379, and
Chapter II, Section 2, Title VII, Book II of the
Revised Penal Code, where one or more of the
accused are officials occupying the following
positions in the government, whether in a
permanent, acting or interim capacity, at the
time of the commission of the offense:

over forfeiture cases had been removed w/o


subsequent amendments expressly restoring
such civil jurisdiction

2. Grant of jurisdiction over violations of RA 1379


did not change even under the amendments of
RA7975 and RA 8294, though it came to be
limited to cases involving high-ranking public
officials

2. Petition for forfeiture is not an ancilliary action


for the criminal action against him, so not under
jurisdiction of Sandiganbayan

3. It has authority to investigate and initiate


forfeiture proceedings vs. petitioner based on
Consti and RA 6770: The constitutional power of
investigation of the Office of the Ombudsman is
plenary and unqualified; its power to investigate
any act of a public official or employee which
appears to be illegal, unjust, improper or
inefficient covers the unlawful acquisition of
wealth by public officials as defined under R.A.
No. 1379

1. WON SB has jurisdiction over petitions for


forfeiture under RA 1379

4. Section 15, RA 6770 expressly empowers


Ombudsman to investigate and prosecute such
cases of unlawful acquisition of wealth.

(1) Officials of the executive branch occupying


the positions of regional director and higher,
otherwise classified as Grade 27 and higher of
the Compensation and Position Classification Act
of 1989 (Republic Act No. 6758), specifically
including:

5. ON REQUIREMENTS under RA 1379: inquiry


was conducted similar to PI + SOLGENs
participation no longer required since
Ombudsman endowed w/ authority to investigate
and prosecute

(d) Philippine army and air force colonels, naval


captains, and all officers of higher ranks;

6. dismiss petition for forum shopping: MTD was


already filed before SB

4. SBs jurisdiction based on PD 1606


encompasses all cases involving violations of RA
3019 IRRESPECTIVE OF WON THESE CASES ARE
CIVIL OR CRIMINAL IN NATURE

REPLY by Garcia
1. SBs criminal jurisdiction is separate and
distinct from its civil jurisdiction : SBs jurisdiction

ISSUES:

2. WON Ombudsman has authority to


investigate, initiate and prosecute such petitions
for forfeiture
3. WON petitioner is guilty of forum shopping
HELD: Petition W/O MERIT, dismissed
1. SB HAS JURISDICTION
Reasoning: Republic vs. Sandiganbayan:
Originally, SOLGEN was authorized to initiate
forfeiture proceedings before then CFI of the city
or province where the public officer/employee
resides or holds office [RA 1379, SEC2]
Upon the creation of the Sandiganbayan [PD
1486], original and exclusive jurisdiction over
such violations was vested in SB.
PD 1606: repealed 1486 and modified
jurisdiction of SB by removing its jurisdiction over
civil actions brought in connection w/ crimes w/n
exclusive jurisdiction of SB, including:
8|P a g e

JURISDICTION; FOR CASE


DIGEST

> restitution or reparation for damages

>>ChapII, Sec2, Title VII, Book II of the RPC

>recovery of instruments and effects of the


crime

Where 1 or more of the accused are officials


occupying the following positions, whether in a
permanent, acting or interim capacity, at the
time of the commission of the offense (see
above)

>civil actions under Art32 and 34 of the Civil


Code
>and forfeiture proceedings provided under RA
1379
BP 129: abolished concurrent jurisdiction of SB
and regular courts, expanded EOJ of SB over
offenses enumerated in SEC4 of PD1606 to
embrace all such offenses irrespective of
imposable penalty.
PD1606 was later amended by PD 1869 and
eventually by PD 1861 because of the
proliferation of filing cases w/ penalty not higher
than PC or its equivalent and even such cases
not serious in nature
*jurisdiction over violations of RA 3019 and 1379
is lodged w/ SB
under RA 8249: SB vested w/ EOJ in all cases
involving violations of :
>>RA 3019 - Anti-Graft and Corrupt Practices
Act
>>RA 1379 - An Act Declaring Forfeiture in Favor
of the State Any Property Found To Have Been
Unlawfully Acquired By Any Public Officer or
Employee and Providing for the Proceedings
Therefor.

PD 1486: vested SB w/ jurisdiction over RA 1379


forfeiture proceedings
Sec12: Chief Special Prosecutor has authority
to file and prosecute forfeiture cases, not
SOLGEN, to SB, not CFI (BUT THIS IS JUST AN
IMPLIED REPEAL as may be derived from the
repealing clause of PD 1486)

ON CIVIL NATUR OF FORFEITURE ACTIONS


PD 1487: created Ombudsman
-they are actions in rem, therefore, civil in nature
BUT FORFEITURE OF AN ILLEGALLY ACQUIRED
PROPERTY PARTAKES THE NATURE OF A PENALTY
[as discussed in Cabal vs. Kapunan]
SB VESTED W/ JURISDICTION OVER VIOLATIONS
OF RA 1379 [An Act Declaring Forfeiture In Favor
of the State Any Property Found to Have Been
Unlawfully Acquired By Any Public Officer or
Employee and Providing For the Proceedings
Therefor.]: the law provides a procedure for
forfeiture in case a public officer has acquired
during his incumbency an amount of property
manifestly out of proportion to his salary as such
public officer or employee and to his lawful
income and income from legitimately acquired
property. No penalty for the public officer for
unlawful acquisition but the law imposes
forfeiture as a penalty for unlawfully acquired
properties
2. YES, as resolved in Republic vs. SB (it was the
main issue there)
RA 1379, Sec2: SOLGEN authorized to initiate
forfeiture proceedings

PD 1606 repealed expressly PD 1486


PD 1607 provided that Office of the Chief Special
Prosecutor has exclusive authority to conduct
preliminary investigation of all cases cognizable
by the SB, file info therefore, and direct and
control prosecution of said cases
also removed authority to file actions for
forfeiture under RA 1379
the repeal of P.D. No. 1486 by P.D. No. 1606
necessarily revived the authority of the Solicitor
General to file a petition for forfeiture under R.A.
No. 1379, but not the jurisdiction of the Courts of
First Instance over the case nor the authority of
the Provincial or City Fiscals (now Prosecutors) to
conduct the preliminary investigation therefore,
since said powers at that time remained in the
Sandiganbayan and the Chief Special Prosecutor.
PD 1630: expanded the Tanodbayans authority:
given exclusive authority to conduct PI of all
cases cognizable by SB, to file info therefore and
to direct and control the prosecution of said
cases
9|P a g e

JURISDICTION; FOR CASE


DIGEST

**1987 CONSTI enacted


RA 6770 + ART XI, SEC 13 of 1987 CONSTI:
POWERS OF OMBUDSMAN:
1) Investigate and prosecute on its own or on
complaint by any person, any act or omission of
any public officer or employee, office or agency,
when such act or omission appears to be illegal,
unjust, improper or inefficient. It has primary
jurisdiction over cases cognizable by the
Sandiganbayan and, in the exercise of this
primary jurisdiction, may take over, at any stage,
from any investigatory agency of Government,
the investigation of such cases;
(11) Investigate and initiate the proper action
for the recovery of ill-gotten and/or unexplained
wealth amassed after 25 February 1986 and the
prosecution of the parties involved therein.
*It is the Ombudsman who should file petition for
forfeiture under RA 1379
BUT powers to investigate and initiate proper
action for recovery of ill-gotten and/or
unexplained wealth is restricted only to cases for
the recovery of ill-gotten and/or unexplained
wealth amassed AFTER FEB 1986
3. ON FORUM SHOPPING: GUILTY
Garcia failed to inform the court that he had
filed a MTD in relation to the petition for
forfeiture before the SB.
A scrutiny of the Motion to Dismiss reveals that
petitioner raised substantially the same issues

and prayed for the same reliefs therein as it has


in the instant petition. In fact, the Arguments and
Discussion[89] in the Petition of petitioners
thesis that the Sandiganbayan has no jurisdiction
over separate civil actions for forfeiture of
unlawfully acquired properties appears to be
wholly lifted from the Motion to Dismiss. The only
difference between the two is that in the Petition,
petitioner raises the ground of failure of the
petition for forfeiture to comply with the
procedural requirements of R.A. No. 1379, and
petitioner prays for the annulment of the
Sandiganbayans Resolution dated 29 October
2004 and Writ of Preliminary Attachment dated 2
November 2004. Nevertheless, these differences
are only superficial. Both Petition and Motion to
Dismiss have the same intent of dismissing the
case for forfeiture filed against petitioner, his
wife and their sons. It is undeniable that
petitioner had failed to fulfill his undertaking.
This is incontestably forum-shopping which is
reason enough to dismiss the petition outright,
without prejudice to the taking of appropriate
action against the counsel and party concerned.

FRANKLlN ALEJANDRO, vs. OFFICE OF THE


OMBUDSMAN FACT-FINDING AND
INTELLIGENCE BUREAU,
G.R.
No. 173121 April 3, 2013
On May 4, 2000, the Head of the Non-Revenue
Water Reduction Department of the Manila Water
Services, Inc. (MWSI) received a report from an
Inspectorate and Special Projects team that the
Mico Car Wash (MICO), owned by Alfredo Rap
Alejandro, has been illegally opening an MWSI

fire hydrant and using it to operate its car-wash


business in Binondo, Manila.
On May 10, 2000, the MWSI, in coordination with
the PNP-CIDG, conducted an anti-water pilferage
operation against MICO.
During the anti-water pilferage operation, the
PNP-CIDG discovered that MICOs car-wash boys
indeed had been illegally getting water from an
MWSI fire hydrant. The PNP-CIDG arrested the
car-wash boys and confiscated the containers
used in getting water. At this point, the
petitioner, Alfredos father and the Barangay
Chairman or punong barangay of Barangay 293,
Zone 28, Binondo, Manila, interfered with the
PNP-CIDGs operation by ordering several men to
unload the confiscated containers. This
intervention caused further commotion and
created an opportunity for the apprehended carwash boys to escape.
On August 5, 2003, the respondent Office of the
Ombudsman Fact-Finding and Intelligence
Bureau, after conducting its initial investigation,
filed with the Office of the Overall Deputy
Ombudsman an administrative complaint against
the petitioner for his blatant refusal to recognize
a joint legitimate police activity, and for his
unwarranted intervention.
In its decision dated August 20, 2004, the Office
of the Deputy Ombudsman found the petitioner
guilty of grave misconduct and ordered his
dismissal from the service. The Deputy
Ombudsman ruled that the petitioner cannot
overextend his authority as Barangay Chairman
and induce other people to disrespect proper
authorities. The Deputy Ombudsman also added
10 | P a g e

JURISDICTION; FOR CASE


DIGEST

that the petitioner had tolerated the illegal acts


of MICOs car-wash boys.
Alejandro filed MFR, denied by the Office of the
Ombudsman
Alejandro appealed to CA but dismissed it due to
premature filing as Alejandro failed to exhaust
proper administrative remedies.
Issue: WHETHER THE OFFICE OF THE
OMBUDSMAN HAS JURISDICTION OVER ELECTIVE
OFFICIALS AND HAS THE POWER TO ORDER
THEIR DISMISSAL FROM THE SERVICE.
Held: The Ombudsman has concurrent
jurisdiction over administrative cases which are
within the jurisdiction of the regular courts or
administrative agencies
The Office of the Ombudsman was created by no
less than the Constitution. It is tasked to exercise
disciplinary authority over all elective and
appointive officials, save only for impeachable
officers. While Section 21 of The Ombudsman
Act and the Local Government Code both provide
for the procedure to discipline elective officials,
the seeming conflicts between the two laws have
been resolved in cases decided by this Court.
In Hagad v. Gozo-Dadole, we pointed out that
"there is nothing in the Local Government Code
to indicate that it has repealed, whether
expressly or impliedly, the pertinent provisions of
the Ombudsman Act. The two statutes on the
specific matter in question are not so
inconsistent x x x as to compel us to only uphold
one and strike down the other." The two laws

may be reconciled by understanding the primary


jurisdiction and concurrent jurisdiction of the
Office of the Ombudsman.
The Ombudsman has primary jurisdiction to
investigate any act or omission of a public officer
or employee who is under the jurisdiction of the
Sandiganbayan. RA 6770 provides:
Section 15. Powers, Functions and Duties. The
Office of the Ombudsman shall have the
following powers, functions and duties:
(1) Investigate and prosecute on its own or on
complaint by any person, any act or omission of
any public officer or employee, office or agency,
when such act or omission appears to be illegal,
unjust, improper or inefficient. It has primary
jurisdiction over cases cognizable by the
Sandiganbayan and, in the exercise of this
primary jurisdiction, it may take over, at any
stage, from any investigatory agency of
Government, the investigation of such cases.
[italics supplied; emphasis and underscore ours]
The Sandiganbayans jurisdiction extends only to
public officials occupying positions corresponding
to salary grade 27 and higher.
Consequently, as we held in Office of the
Ombudsman v. Rodriguez, any act or omission of
a public officer or employee occupying a salary
grade lower than 27 is within the concurrent
jurisdiction of the Ombudsman and of the regular
courts or other investigative agencies.

authorities, the body where the complaint is filed


first, and which opts to take cognizance of the
case, acquires jurisdiction to the exclusion of
other tribunals exercising concurrent
jurisdiction. In this case, the petitioner is a
Barangay Chairman, occupying a position
corresponding to salary grade 14. Under RA
7160, the sangguniang panlungsod or
sangguniang bayan has disciplinary authority
over any elective barangay official, as follows:
Section 61. Form and Filing of Administrative
Complaints. A verified complaint against any
erring local elective official shall be prepared as
follows:
(c) A complaint against any elective barangay
official shall be filed before the sangguniang
panlungsod or sangguniang bayan concerned
whose decision shall be final and executory.
[italics supplied]
Since the complaint against the petitioner
was initially filed with the Office of the
Ombudsman, the Ombudsman's exercise of
jurisdiction is to the exclusion of the sangguniang
bayan whose exercise of jurisdiction
is concurrent.

Darma Maslag vs Elizabeth Monzon,


William Geston, and Registry of Deeds of
Benguet
GR. No. 174908 June 17, 2013

In administrative cases involving the concurrent


jurisdiction of two or more disciplining
11 | P a g e

JURISDICTION; FOR CASE


DIGEST

Facts: In 1998, petitioner filed a Complaint for


reconveyance of real property with declaration of
nullity of original certificate of title against
respondents. The Complaint was filed before the
Municipal Trial Court. After trial, the MTC found
respondent Monzon guilty of fraud in obtaining
an OCT over petitioners property. Respondents
appealed to the Regional Trial Court (RTC)
declaring the MTC without jurisdiction over
petitioners cause of action. The presiding judge
declared that it will take cognizance of the case
pursuant to Section 8, Rule 40of the Rules of
Court which provides for appeal from orders
dismissing the case without trial; lack
of jurisdiction.RTC thereafter reversed the
decision of the MTC, prompting the petitioner to
file a Notice of Appeal. The Court of Appeals
dismissed the said appeal and affirmed the
respondents contention that the proper remedy
is a Petition for Review under Rule 42, and not an
ordinary appeal.
Hence, the present Petition for Review on
Certiorari.
Issue: W/N petitioners ordinary appeal is the
proper remedy
Held: No. The CA is correct in holding that the
proper mode of appeal should have been a
Petition for Review under Rule42 of the Rules of
Court, and not an ordinary appeal under Rule
41.Under the present state of the law, in cases
involving title to real property, original and
exclusive jurisdiction belongs to either the RTC or
the MTC, depending on the assessed value of the
subject property. Since the assessed value of the
disputed property is onlyP12,400, MTC has
original and exclusive jurisdiction over the

subject matter of the case. In fact and in law, the


RTC Resolution was a continuation of the
proceedings that originated from the MTC. It was
a judgment issued by the RTC in the exercise
of its appellate jurisdiction. It cannot be
overemphasized that jurisdiction over the subject
matter is conferred only by law and it is not
within the courts, let alone the parties, to
themselves determine or conveniently set
aside.
Neither would theactive participation of the
parties nor estoppels operate to confer original
and exclusive jurisdiction where the court or
tribunal only wields appellate jurisdiction over
the case. The present court looks at what type of
jurisdiction was actually exercised by the RTC,
and not into what type of jurisdiction the RTC
should have exercised. Inquiring into what the
RTC should have done in disposing of the case is
a question that already involves the merits of the
appeal, but the court obviously cannot go into
that where the mode of appeal was improper to
begin with. Wherefore, Petition for Review is
denied for lack of merit. The Court affirms the
decision of the Court of Appeals.

MARK JEROME S. MAGLALANG v. PHILIPPINE


AMUSEMENT AND GAMING CORPORATION
(PAGCOR), AS REPRESENTED BY ITS
INCUMBENT CHAIRMAN EFRAIM GENUINO
G.R. No. 190566 : December 11, 2013

Facts: Petitioner was a teller at the Casino


Filipino, Angeles City Branch, Angeles City, which
was operated by respondent Philippine

Amusement and Gaming Corporation (PAGCOR).


While he was performing his functions as teller, a
lady customer identified later as one Cecilia
Nakasato approached him in his booth and
handed to him an undetermined amount of cash
consisting of mixed P1,000.00 and P500.00 bills.
There were forty-five P1,000.00 and ten P500.00
bills for the total amount of P50,000.00.
Following casino procedure, petitioner laid the
bills on the spreading board. However, he
erroneously spread the bills into only four
clusters instead of five clusters worth P
10,000.00 per cluster. He then placed markers
for P10,000.00 each cluster of cash and declared
the total amount of P40,000.00 to Cecilia.
Perplexed, Cecilia asked petitioner why the latter
only dished out P40,000.00. She then pointed to
the first cluster of bills and requested petitioner
to check the first cluster which she observed to
be thicker than the others. Petitioner performed
a recount and found that the said cluster
contained 20 pieces of P1,000.00 bills. Petitioner
apologized to Cecilia and rectified the error by
declaring the full and correct amount handed to
him by the latter. Petitioner, however, averred
that Cecilia accused him of trying to shortchange
her and that petitioner tried to deliberately fool
her of her money. Petitioner tried to explain, but
Cecilia allegedly continued to berate and curse
him. To ease the tension, petitioner was asked to
take a break. After ten minutes, petitioner
returned to his booth. However, Cecilia allegedly
showed up and continued to berate petitioner. As
a result, the two of them were invited to the
casinos Internal Security Office in order to air
their respective sides. Thereafter, petitioner was
required to file an Incident Report which he
submitted on the same day of the incident.

12 | P a g e

JURISDICTION; FOR CASE


DIGEST

On January 8, 2009, petitioner received a


Memorandum issued by the casino informing him
that he was being charged with Discourtesy
towards a casino customer and directing him to
explain within 72 hours upon receipt of the
memorandum why he should not be sanctioned
or dismissed. In compliance therewith, petitioner
submitted a letter-explanation dated January 10,
2009.
On March 31, 2009, petitioner received another
Memorandum dated March 19, 2009, stating that
the Board of Directors of PAGCOR found him
guilty of Discourtesy towards a casino customer
and imposed on him a 30-day suspension for this
first offense. Aggrieved, on April 2, 2009,
petitioner filed a Motion for Reconsideration
seeking a reversal of the boards decision and
further prayed in the alternative that if he is
indeed found guilty as charged, the penalty be
only a reprimand as it is the appropriate penalty.
During the pendency of said motion, petitioner
also filed a Motion for Production dated April 20,
2009, praying that he be furnished with copies of
documents relative to the case including the
recommendation of the investigating committee
and the Decision/Resolution of the Board
supposedly containing the latters factual
findings.
Subsequently, on June 18, 2009, PAGCOR issued
a Memorandum dated June 18, 2009 practically
reiterating the contents of its March 19, 2009
Memorandum. It informed petitioner that the
Board of Directors 2009 resolved to deny his
appeal for reconsideration for lack of merit.
On August 17, 2009, petitioner filed a petition for
certiorari under Rule 65 of the 1997 Rules of Civil

Procedure, as amended, before the CA, averring


that there is no evidence, much less factual and
legal basis to support the finding of guilt against
him. Moreover, petitioner ascribed grave abuse
of discretion amounting to lack or excess of
jurisdiction to the acts of PAGCOR in adjudging
him guilty of the charge, in failing to observe the
proper procedure in the rendition of its decision
and in imposing the harsh penalty of a 30 -day
suspension. Justifying his recourse to the CA,
petitioner explained that he did not appeal to the
Civil Service Commission (CSC) because the
penalty imposed on him was only a 30- day
suspension which is not within the CSCs
appellate jurisdiction. He also claimed that
discourtesy in the performance of official duties
is classified as a light offense which is punishable
only by reprimand.
In its assailed Resolution dated September 30,
2009, the CA outrightly dismissed the petition for
certiorari for being premature as petitioner failed
to exhaust administrative remedies before
seeking recourse from the CA. Invoking Section
2(1), Article IX-B of the 1987 Constitution, the CA
held that the CSC has jurisdiction over issues
involving the employer-employee relationship in
all branches, subdivisions, instrumentalities and
agencies of the Government, including
government- owned or controlled corporations
with original charters such as PAGCOR. Petitioner
filed his Motion for Reconsideration which the CA
denied in the assailed Resolution dated
November 26, 2009. In denying the said motion,
the CA relied on this Courts ruling in Duty Free
Philippines v. Mojica citing Philippine Amusement
and Gaming Corp. v. CA, where this Court held as
follows: It is now settled that, conformably to
Article IX-B, Section 2(1), [of the 1987

Constitution] government-owned or controlled


corporations shall be considered part of the Civil
Service only if they have original charters, as
distinguished from those created under general
law. PAGCOR belongs to the Civil Service because
it was created directly by PD 1869 on July 11,
1983. Consequently, controversies concerning
the relations of the employee with the
management of PAGCOR should come under the
jurisdiction of the Merit System Protection Board
and the Civil Service Commission, conformably to
the Administrative Code of 1987. Section 16(2) of
the said Code vest[s] in the Merit System
Protection Board the power inter alia to: a) Hear
and decide on appeal administrative cases
involving officials and employees of the Civil
Service. Its decision shall be final except those
involving dismissal or separation from the
service which may be appealed to the
Commission.
Hence, this petition where petitioner argues that
the CA committed grave and substantial error of
judgment
ISSUE: Was the CA correct in outrightly
dismissing the petition for certiorari filed before
it on the ground of non-exhaustion of
administrative remedies?
HELD: Court of Appeals decision reversed. It
bears stressing that the judicial recourse
petitioner availed of in this case before the CA is
a special civil action for certiorari ascribing grave
abuse of discretion, amounting to lack or excess
of jurisdiction on the part of PAGCOR, not an
appeal. Suffice it to state that an appeal and a
special civil action such as certiorari under Rule
65 are entirely distinct and separate from each
13 | P a g e

JURISDICTION; FOR CASE


DIGEST

other. One cannot file petition for certiorari under


Rule 65 of the Rules where appeal is available,
even if the ground availed of is grave abuse of
discretion. A special civil action for certiorari
under Rule 65 lies only when there is no appeal,
or plain, speedy and adequate remedy in the
ordinary course of law. Certiorari cannot be
allowed when a party to a case fails to appeal a
judgment despite the availability of that remedy,
as the same should not be a substitute for the
lost remedy of appeal. The remedies of appeal
and certiorari are mutually exclusive and not
alternative or successive.
In sum, there being no appeal or any plain,
speedy, and adequate remedy in the ordinary
course of law in view of petitioner's allegation
that PAGCOR has acted without or in excess of
jurisdiction, or with grave abuse of discretion
amounting to lack or excess of jurisdiction, the
CA's outright dismissal of the petition for
certiorari on the basis of non-exhaustion of
administrative remedies is bereft of any legal
standing and should therefore be set aside.
Finally, as a rule, a petition for certiorari under
Rule 65 is valid only when the question involved
is an error of jurisdiction, or when there is grave
abuse of discretion amounting to lack or excess
of jurisdiction on the part of the court or tribunals
exercising quasi-judicial functions. Hence, courts
exercising certiorari jurisdiction should refrain
from reviewing factual assessments of the
respondent court or agency. Occasionally,
however, they are constrained to wade into
factual matters when the evidence on record
does not support those factual findings; or when
too much is concluded, inferred or deduced from
the bare or incomplete facts appearing on

record. Considering the circumstances and since


this Court is not a trier of facts, remand of this
case to the CA for its judicious resolution is in
order.

ROBERTO DIPAD and SANDRA DIPAD, vs.


SPOUSES ROLANDO OLIVAN and BRIGIDA
OLIVAN, and BRIGIDA OLIVAN, and RUBIO
GUIJON MADRIGALLO
Facts: Due to a collision between the car of
petitioner spouses Dipad and the passenger jeep
owned by respondents, the former filed a civil
action for damages before the sala of Municipal
Trial Court (MTC) Judge Clavecilla.
During trial, Roberto Dipad mentioned in his
direct testimony that because he was not able to
make use of his vehicle for his buy-and-sell
business, he suffered damages by way of lost
income for three months amounting to P40,000.
Then, during cross-examination, the defense
required him to produce his personal copy of his
ITRs for the years 2001, 2002 and 2003.
Dipad vehemently objected on the ground of
confidentiality of the ITRs. He also claimed that
the demand therefor was incriminatory and in
the nature of a fishing expedition.
By reason of the opposition, Judge Clavecilla
suspended the trial and required petitioners to

show their basis for invoking the confidentiality


of the ITRs. After the parties submitted their
respective Comments on the matter, the MTC in
its 3 February 2005 Order required the
production of the ITRs.
Aggrieved, the spouses Dipad filed a Motion for
Reconsideration, which was denied by Judge
Clavecilla. Thereafter, they instituted a Rule 65
Petition for Certiorari and Prohibition before the
RTC, assailing the 3 February 2005 Order of the
MTC for having been issued with grave abuse of
discretion amounting to lack or excess of
jurisdiction. In that Petition, they opposed Judge
Clavecillas ruling in this wise:
The respondent Judge stated in his order that the
cited provision does not apply, stating that "what
is being requested to be produced is plaintiffs
copy of their tax returns for the years 2001 to
2003, " thereby ordering Dipad, "to furnish
defendants counsel within five (5) days from
receipt of this order copy of their income tax
returns for the years 2001 to 2003, inclusive."
RTC: We beg to differ to such holding, because if
a copy of a taxpayers return filed with the
Bureau of Internal Revenue can be open to
inspection only upon the order of the President of
the Philippines, such provision presupposes the
confidentiality of the document; and with more
reason that the taxpayer cannot be compelled to
yield his copy of the said document.
Thus, it is indubitable that compelling the
petitioners to produce petitioner Roberto Dipads
Income Tax Returns and furnish copies thereof to
the private respondents would be violative of the
provisions of the National Internal Revenue Code
14 | P a g e

JURISDICTION; FOR CASE


DIGEST

on the rule on confidentiality of Income Tax


return as discussed above.

mistakes in the courts findings based on a


mistake of law or of fact.

In its 6 May 2005 Decision, the RTC dismissed


the Rule 65 Petition for being an inappropriate
remedy. According to the trial court, the errors
committed by Judge Clavecilla were, if at all,
mere errors of judgment correctible not by the
extraordinary writ of certiorari, but by ordinary
appeal.

Here, it is patently clear that petitioners do not


question whether the MTC has jurisdiction or
authority to resolve the issue of confidentiality of
ITRs. Rather, they assail the wisdom of the MTCs
very judgment and appreciation of the ITR as not
confidential. Specifically, they claim that the
ruling violated the provisions of the NIRC on the
alleged rule on confidentiality of ITRs.

Petitioners moved for reconsideration, but their


motion was denied by the RTC.

Held: Yes, Court stressed that it is basic in our


jurisdiction that a petition for certiorari under
Rule 65 is not a mode of appeal. The remedy,
which is narrow in scope, only corrects errors of
jurisdiction. Thus, if the issue involves an error of
judgment, the error is correctible by an appeal
via a Rule 45 petition, and not by a writ of
certiorari under Rule 65 of the Rules of Court.

Based on the definitions above, we conclude


similarly as the RTC that if there is an error to
speak of the error relates only to a mistake in the
application of law, and not to an error of
jurisdiction or grave abuse of discretion
amounting to excess of jurisdiction. The only
error petitioners raise refers to Judge Clavecillas
mistake of not applying Section 71, which
allegedly prohibits the production of ITRs
because of confidentiality. Certainly, as correctly
posited by the court a quo, if every error
committed by the trial court is subject to
certiorari, trial would never come to an end, and
the docket will be clogged ad infinitum.

As defined in jurisprudence, errors of jurisdiction


occur when the court exercises jurisdiction not
conferred upon it by law. They may also occur
when the court or tribunal, although it has
jurisdiction, acts in excess of it or with grave
abuse of discretion amounting to lack of
jurisdiction.

[G.R. No. 118251. June 29, 2001]

Issue: Whether or not the RTC is correct in


dismissing the petition for being an inappropriate
remedy.

On the contrary, errors of judgment are those


that the court may commit in the exercise of its
jurisdiction. They include errors of procedure or

METROPOLITAN BANK AND TRUST


COMPANY, petitioner, vs. HON.
REGINO T. VERIDIANO II, Presiding

Judge, RTCManila, Branch 31, and


DOMINADOR ONG, respondents.
FACTS:
Accused, as Treasurer of the Sun Ray Metal, Inc.,
a corporation duly organized under the laws of
the Philippines, did then and there willfully,
unlawfully,
and
feloniously
defraud
the
Metropolitan Bank and Trust Company, a banking
entity organized and doing business under the
laws of the Philippines, with office situated at
Dasmarias corner Ugalde Streets, Binondo, this
City, represented by LUCILA Y. UY, its Senior
Manager, in the following manner, to wit: the
said accused, under trust Receipts dated
September 6, 1989 and September 15, 1989
executed by the said Dominador Ong/Sun Ray
Metal, Inc. in favor of the said Metropolitan Bank
and Trust Company, received in trust from the
latter or all valued at P413,133.00 for the
purpose of holding the said merchandise in trust
under the express obligation on his part to
dispose of the same and turn over the proceeds
of the sale to the said bank, if sold, or to the
account for or return the same, if unsold, on its
due date or upon demand, but the said accused,
once in possession of the same, far from
complying with his aforesaid obligation, failed
and refused and still fails and refuses to do so
despite repeated demands made upon him to
that effect and with intent to defraud, the said
accused did then and there willfully, unlawfully
and feloniously misappropriate, misapply and
convert the same or the value thereof, to his own
personal use and benefit, to the damage and
prejudice of the said Metropolitan Bank and Trust
Company represented by LUCILA Y. UY in the
15 | P a g e

JURISDICTION; FOR CASE


DIGEST

total amount
Currency.[2]

of

413,133.00,

Philippine

The public respondent ratiocinated that the


private respondent was not duly authorized to
represent Sun Ray Metal, Inc. considering that
there were other high-ranking officers who could
have negotiated the contract; that there was no
written authority from the board of directors of
the said corporation authorizing the private
respondent to execute the trust receipts in its
behalf in favor of petitioner bank. Moreover, the
defense sufficiently proved that the restructuring
agreement effectively novated the obligation
under the trust receipts.
ISSUE:
The public respondent judge gravely abused his
discretion amounting to lack or excess of
jurisdiction when the latter acquitted the private
respondent.
HELD:
The petition is not meritorious.
We have thoroughly perused the records
and carefully analyzed the cases relied upon by
the petitioner and found no cogent reason to
depart from the judgment of the public
respondent in the case at bar.
In the present case, inasmuch as the
prosecution was never denied any opportunity to
present its case and that there is no indication or
proof that the trial was a sham, a review and
consequent setting aside of the trial courts

decision of acquittal will put the private


respondent in double jeopardy. Double jeopardy
attaches only: (1) upon valid indictment; (2)
before a competent court; (3) after arraignment;
(4) when a valid plea has been entered; and (5)
when the defendant was acquitted or convicted
or the case was dismissed or otherwise
terminated without the express consent of the
accused.[11] Consequently in such an event, the
conviction or acquittal of the accused or the
dismissal of the case shall be a bar to another
prosecution for the offense charged, or for any
attempt to commit the same or a frustration
thereof, or for any offense which is necessarily
included in the offense charged in the former
complaint or information.[12]
The record shows that the court a
quo allowed
both
parties
an
exhaustive
presentation and offer of evidence and
submission of their respective memoranda. If
indeed public respondent has misappreciated
certain evidence, as argued by the petitioner in
this petition, such are not jurisdictional matters
that may be determined and ruled upon in a
certiorari proceeding. If at all, such alleged error
by the public respondent was merely an error of
judgment, but not an error of jurisdiction.

In the prayer[19] of this petition in the case at bar,


what is prayed for is that the decision of
respondent Judge dated November 28, 1994
acquitting
the
private
respondent
be
reversed. Nothing therein is mentioned about
damages or the civil aspect of the case.
In fine, the instant petition should be
dismissed not only for lack of merit but also for
lack of legal personality on the part of the
petitioner to appeal the public respondents
ruling on the criminal aspect of the case.
WHEREFORE, the petition is hereby
DISMISSED, with costs against the petitioner.

PEOPLE v ASIS
Ponente: Mendoza, J.
Date: August 25, 2010
Petition for review on certiorari of a resolution of
the Court of Appeals
RATIO DECIDENDI:

In this case, petitioners action does not concern


the civil aspect of the case but the validity of the
judgment itself. Indeed, petitioner does not
actually question the award of damages. What
he contends is that the trial court decided the
case outside the issues made out by the
pleadings and thereby deprived the prosecution
of due process.

A petition for certiorari under Rule 65,


not appeal, is the remedy to question a
verdict of acquittal whether at the trial
court or at the appellate level.

While certiorari may be availed of to


correct an erroneous acquittal, the
petitioner in such an extraordinary
proceeding must clearly demonstrate
16 | P a g e

JURISDICTION; FOR CASE


DIGEST

that the trial court blatantly abused its


authority to a point so grave as to
deprive it of its very power to dispense
justice.
3

An appellate court in a petition for


certiorari cannot review a trial courts
evaluation of the evidence and factual
findings. Errors of judgment cannot be
raised in a Rule 65 petition as a writ of
certiorari can only correct errors of
jurisdiction or those involving the
commission of grave abuse of discretion.

Respondents: Judge Asis of the RTC, and Abordo

Abordo was riding his motorcycle on his


way home when an altercation ensued
between him and the three offended
parties Montes, Calvez, and Majait. The
accused Abordo shot Majait in the leg
while Calvez was hit in the abdomen.
Montes escaped unhurt.

The CA dismissed the petition, saying


that the filing of the petition for certiorari
was the wrong remedy. It said that as
the State was questioning the verdict of
the acquittal and findings of lesser
offenses by the trial court, the remedy
should have been an appeal.

Abordo was charged with two counts of


attempted murder (Majait and Montes?)
and one count of frustrated murder
(Calvez?).

It said thus:

QUICK FACTS:
The RTC acquitted accused Abordo of the
attempted murder of Montes, and only held him
liable of Serious Physical Injuries for shooting
Calvez, and Less Serious Physical Injuries with
regard to Majait. The OSG filed a petition for
certiorari under Rule 65 before the Court of
Appeals, but the CA dismissed the petition for
being the wrong remedy. According to the CA,
the remedy should have been an appeal, not
petition for certiorari.

The RTC held Abordo liable only for


Serious Physical Injuries for shooting
Calvez and Less Serious Physical Injuries
with regard to Majait, when it found no
treachery and evident premeditation.
Four mitigating circumstances were
appreciated in favor of Abordo. Abordo
was acquitted with respect to the
complaint of Montes.

CA: (petition dismissed)

Name of Accused: Abordo

Petitioner: People of the Philippines

Where the error is not one of


jurisdiction but an error of law or
fact a mistake of judgment
appeal is the remedy.

Section 1, Rule 122 of the 2000


Rules of Criminal Procedure
provides that any party may
appeal from a judgment or final
order unless the accused will be
placed in double jeopardy.
In
filing the petition for certiorari,
the accused is thereby placed in
double jeopardy.

Such recourse is tantamount to


converting
the
petition
for
certiorari into an appeal, contrary
to the Constitution, the Rules of
Court
and
prevailing
jurisprudence
on
double
jeopardy.

RTC:

FACTS:

Name of Offended Parties: Montes, Calvez, and


Majait

holding him liable for Serious Physical


Injuries and Less Serious Physical Injuries
In the two other cases.

The OSG filed a petition for certiorari


under Rule 65 before the CA based on
the ground that Judge Asis of the RTC
acted with grave abuse of discretion
amounting to lack or excess of
jurisdiction in rendering its decision of
acquitting Abordo in one case, only

17 | P a g e

JURISDICTION; FOR CASE


DIGEST

The petition is dismissible not


only on the ground of wrong
remedy taken by the petitioner
to question an error of judgment
but also on the ground that such
action places the accused in
double jeopardy.

ISSUE: Whether or not the proper remedy to


question a verdict of acquittal is a petition for
certiorari.
DECISION: Petition of the People was partially
granted.
HELD:

acquittal is final and unappealable. The rule,


however, is not without exception. In several
cases, the Court has entertained petitions for
certiorari questioning the acquittal of the
accused in, or the dismissals of, criminal cases.
In People v Louel Uy, the Court said that petition
for certiorari under Rule 65 is appropriate upon
clear showing by the petitioner that the lower
court in acquitting the accused:
1

Committed reversible errors of judgment

Grave abuse of discretion amounting to


lack or excess of jurisdiction or denial of
due process.

Such commission of the lower court renders its


judgment void.

Yes.
No double jeopardy
Certiorari is the proper remedy
A petition for certiorari under Rule 65, not
appeal, is the remedy to question a verdict of
acquittal whether at the trial court or at the
appellate level.
Since appeal could not be taken without violating
Abordos
constitutionally
guaranteed
right
against double jeopardy, the OSG was correct in
pursuing its cause via a petition for certiorari
under Rule 65 before the appellate court.

When the order or dismissal is annulled or set


aside by an appellate court in an original special
civil action via certiorari, the right of the accused
against double jeopardy is not violated. Such
dismissal order, being considered void judgment,
does not result in jeopardy.
OSGs petition for certiorari before the CA,
however, is bereft of merit

Exception to Finality-of-Acquittal Doctrine

While the CA was erroneous of dismissing the


petition, the OSGs petition for certiorari if given
due course is bereft of merit.

In our jurisdiction, we adhere to the finality-ofacquittal doctrine, that is, a judgment of

While certiorari may be availed of to correct an


erroneous acquittal, the petitioner in such an

extraordinary
proceeding
must
clearly
demonstrate that the trial court blatantly abused
its authority to a point so grave as to deprive it
of its very power to dispense justice. A reading
of the OSG petition fails to show that the
prosecution was deprived of its right to due
process.
Also, what the OSG is questioning are errors of
judgment. This, however, cannot be resolved
without
violating
Abordos
constitutionally
guaranteed right against double jeopardy. An
appellate court in a petition for certiorari cannot
review a trial courts evaluation of the evidence
and factual findings. Errors of judgment cannot
be raised in a Rule 65 petition as a writ of
certiorari can only correct errors of jurisdiction or
those involving the commission of grave abuse
of discretion.
Error of Judgment v Error of Jurisdiction
Any error committed in the evaluation of
evidence is merely an error of judgment that
cannot be remedied by certiorari. An error of
judgment is one in which the court may commit
in the exercise of its jurisdiction.
An error of jurisdiction is one where the act
complained of was issued by the court without or
in excess of jurisdiction, or with grave abuse of
discretion which is tantamount to lack or in
excess of jurisdiction and which error is
correctible only by the extraordinary writ of
certiorari.
Certiorari will bot be issued to cure errors by the
trial court in its appreciation of the evidence of
18 | P a g e

JURISDICTION; FOR CASE


DIGEST

the parties, and its conclusions anchored on the


said findings and its conclusions of law.
[G.R. No. 130106. July 15, 2005]
PEOPLE OF THE PHILIPPINES, petitioner,
vs. HON. PERLITA J. TRIA-TIRONA, in
her capacity as Presiding Judge,
Branch 102, Regional Trial Court,
Quezon City and CHIEF INSPECTOR
RENATO A. MUYOT, respondents.
FACTS:
Can the government appeal from a
judgment acquitting the accused after trial on
the merits without violating the constitutional
precept against double jeopardy?
Armed with two search warrants, [2] members
of the National Bureau of Investigation (NBI) AntiOrganized
Crime
Division,
together
with
members of the NBI Special Investigation
Division and the Presidential Intelligence and
Counter-Intelligence Task Force Hammer Head
serving as security, conducted a search on the
house of accused-private respondent located on
Banawe, Quezon City. The alleged finding of
498.1094
grams
of
methamphetamine
hydrochloride (shabu) thereat led to the filing of
an information charging private respondent with
Violation of Section 16, Article III of Republic Act
No. 6425,[3] as amended by Rep. Act No. 7659.
The case was raffled to the sala of public
respondent -- Branch 102 of the Regional Trial
Court of Quezon City.

When arraigned on 27 November 1996,


private respondent, assisted by a counsel de
parte, pleaded not guilty to the crime charged.
[5]
After trial on the merits, public respondent
rendered
a
decision[6] acquitting
private
respondent on ground of reasonable doubt.
The decision, more particularly the acquittal
of private respondent, is being assailed via a
petition for certiorari under Rule 65 of the Rules
of Court.
Petitioner contends that public
respondent, in acquitting private respondent,
committed grave abuse of discretion by ignoring
material facts and evidence on record which,
when considered, would lead to the inevitable
conclusion of the latters guilt beyond reasonable
doubt. It added that the appealability of the trial
courts decision of acquittal in the context of the
constitutional guarantee against double jeopardy
should be resolved since it has two pending
petitions[7] before the court raising the same
question.
Petitioner informs the Court that in its prior
petition in People v. Velasco,[8] it has presented
and extensively discussed the now settled
constitutional doctrine in the United States that
the Double Jeopardy Clause does permit a review
of acquittals decreed by trial magistrates where,
as in this case, no retrial will be required even if
the judgment should be overturned. It thus
argues that appealing the acquittal of private
respondent would not be violative of the
constitutional right of the accused against double
jeopardy.
On 13 September 2000, the Court
promulgated its decision in People v. Velasco.
[13]
In said case, the government, by way of a

petition for certiorari under Rule 65 of the Rules


of Court, appealed the decision of Hon. Tirso D.C.
Velasco acquitting accused Honorato Galvez of
the charges of murder and double frustrated
murder due to insufficiency of evidence, and of
the charge of unauthorized carrying of firearm on
the ground that the act charged was not a
violation of law. This Court dismissed the
petition
On 10 November 2004, the Court gave due
course to the petition and required the parties to
submit their respective memoranda.[14]
Private
respondent
Muyot
filed
his
memorandum on 4 March 2005.[15] Invoking the
Rule of Double Jeopardy, he prays that the
petition be dismissed.
ISSUE:
WHETHER
OR
NOT
THE
DECISION
OF
RESPONDENT COURT ACQUITTING PRIVATE
RESPONDENT CAN BE REVIEWED ON A PETITION
FOR REVIEW ON CERTIORARI.
WHETHER OR NOT
SHOULD BE CONVICTED
DANGEROUS DRUG ACT
OF THE EVIDENCE
PROSECUTION.

PRIVATE RESPONDENT
FOR VIOLATION OF THE
OF 1972 ON THE BASIS
PRESENTED BY THE

On the first issue, petitioner argues that


notwithstanding our decision in People v.
Velasco,[16] the fact that we gave due course to
the petition means that the issue on the
sufficiency of the evidence in this case may be
reviewed.
It
added
that
a
petition
19 | P a g e

JURISDICTION; FOR CASE


DIGEST

for certiorari should be an available remedy to


question the acquittal of the accused.
On the second issue, petitioner maintains
that respondent court abused its discretion in
disregarding the testimonies of the NBI agents
on the discovery of the dangerous drug despite
the absence of any evidence to show that they
were impelled by any improper motive.
To settle the issue of whether or not an
acquittal can still be appealed, this Court
pronounced in People v. Velasco[17] that as
mandated by the Constitution, statutes and
jurisprudence, an acquittal is final and
unappealable on the ground of double jeopardy,
whether it happens at the trial court level or
before the Court of Appeals. In general, the rule
is that a remand to a trial court of a judgment of
acquittal brought before the Supreme Court
on certiorari cannot be had unless there is a
finding
of
mistrial,
as
in Galman
v.
Sandiganbayan.[18] Only when there is a finding
of a sham trial can the doctrine of double
jeopardy be not invoked because the people, as
represented by the prosecution, were denied due
process.
From the foregoing pronouncements, it is
clear in this jurisdiction that after trial on the
merits, an acquittal is immediately final and
cannot be appealed on the ground of double
jeopardy. The only exception where double
jeopardy cannot be invoked is where there is a
finding of mistrial resulting in a denial of due
process.
We find petitioners argument that, despite
our ruling in People v. Velasco,[19] since we gave

due course to the petition, the issue on the


sufficiency of the evidence may be reviewed, to
be untenable. The fact that the petition was
given due course does not necessarily mean we
have to look into the sufficiency of the evidence
since the issue to be resolved is the appealability
of an acquittal. We have categorically ruled
in People v. Velasco[20] that, except when there is
a finding of mistrial, no appeal will lie in case of
an acquittal. There being no mistrial in the case
before us, we find no need to reexamine the
evidence, because if we do so, we will be
allowing an appeal to be made on an acquittal
which would clearly be in violation of the
accuseds right against double jeopardy.
Petitioner, via a
petition
for
review
on certiorari, prays for the nullification and the
setting aside of the decision of public respondent
acquitting private respondent claiming that the
former abused her discretion in disregarding the
testimonies of the NBI agents on the discovery of
the illegal drugs. The petition smacks in the
heart of the lower courts appreciation of the
evidence of the parties. It is apparent from the
decision of public respondent that she
considered all the evidence adduced by the
parties. Even assuming arguendo that public
respondent may have improperly assessed the
evidence on hand, what is certain is that the
decision was arrived at only after all the
evidence was considered, weighed and passed
upon. In such a case, any error committed in the
evaluation of evidence is merely an error of
judgment that cannot be remedied bycertiorari.
An error of judgment is one in which the court
may commit in the exercise of its jurisdiction.
[21]
An error of jurisdiction is one where the act
complained of was issued by the court without or

in excess of jurisdiction, or with grave abuse of


discretion which is tantamount to lack or in
excess of jurisdiction and which error is
correctible only by the extraordinary writ
of certiorari.[22] Certiorari will not be issued to
cure errors by the trial court in its appreciation of
the evidence of the parties, and its conclusions
anchored on the said findings and its conclusions
of law.[23] Since no error of jurisdiction can be
attributed
to
public
respondent
in
her
assessment of the evidence, certiorari will not
lie.
WHEREFORE, the petition for certiorari is
hereby DISMISSED.
SO ORDERED.
[G.R. No. 159288. October 19, 2004]
JOHNSON LEE, petitioner, vs. PEOPLE OF
THE PHILIPPINES and NEUGENE
MARKETING, INC., respondents.
FACTS:
NEUGENE Marketing, Inc. (NMI) was incorporated
on January 27, 1978 with funds provided by the
Uy Family. It had an authorized capital stock
of P3 million divided into 30,000 shares with a
par value of P100 per share. The original
incorporators, with their corresponding number
of shares and the amounts thereof, are as
follows:
Johnson
600

Lee
P 60,000.00

20 | P a g e

JURISDICTION; FOR CASE


DIGEST

Lok
1,200
Charles
1,800
Eugenio
2,100
Arsenio
300

Chun

O.

Suen
120,000.00
Sy
180,000.00

Flores,
Jr.
210,000.00
Yang,
Jr.
30,000.00
T O T A
6,000
P600,000.00

=====
=====
====
There were two stock dividend declarations,
one on June 7, 1980 in the amount of P60,000.00
and another on May 2, 1981 for P40,000.00. On
May
15,
1986
Eugenio
Flores,
Jr.
assigned/divested himself of his shares in favor
of Sonny Moreno, 1,050 shares; Arsenio Yang, Jr.,
700 shares and Charles O. Sy, 700 shares. [1]
On June 11, 1987, the NMI sold and
delivered to the Victorias Milling Company, Inc.
(VMCI), in Victorias, Negros Occidental, 77,500
pieces of empty white bags for the price
of P565,750.00. NMI issued Charge Invoice No.
0809[2] dated June 11, 1987 to VMCI covering
said sale. On June 18, 1987, VMCI purchased
100,000 pieces of empty white bags from NMI
for P730,000.00 for which NMI issued Charge

Invoice No. 0810.[3] On June 25, 1987, VMCI again


purchased 28,000 pieces of empty white bags
from NMI for the price of P204,400.00 and the
latter issued Charge Invoice No. 0811 [4] dated
June 25, 1987. In payment of said purchases
from NMI, VMCI drew and issued two Bank of the
Philippine Islands (BPI) Checks: Check No.
068706 dated August 3, 1987 in the amount
of P565,750.00[5] and Check No. 068993 dated
August 19, 1987 in the amount of P934,400.00.
[6]
Both checks were payable to the order of NMI.

praying, among other things, for the annulment


or nullification of the Certification of Filing of
Resolution of Voluntary Dissolution of NMI for
being contrary to law and its by-laws.

On October 13, 1987, stockholders owning


two-thirds (2/3) of the subscribed capital stock of
NMI voted to call a stockholders meeting. One of
the items in the agenda was the dissolution of
the corporation.

A verified complaint for three (3) counts


of estafa was filed against the petitioner and
Sonny Moreno with the City Prosecutors Office.
Appended to the complaint were photocopies of
Charge Invoice Nos. 0809, 0810, and 0811,
issued by NMI to VMCI.

Pursuant thereto, a special stockholders


meeting was held on October 24, 1987 in
Bacolod City.
Accordingly, notices were again sent to all
stockholders of record, all of whom properly
acknowledged the said notices, that a meeting
was to be held on November 30, 1987 to
consider the dissolution of the corporation.
Again the stockholders who attended the
October 24, 1987 meeting were present. Upon
motion duly seconded, the dissolution was
approved. Per Resolution of the Board of
Directors, the law firm of Reyes, Treyes & Fudolin
Law Office was appointed as trustee to collect all
the receivables of the corporation.
On
Moreno,
petition
Clearing

March 22, 1988, Johnson Lee, Sonny


Leoncio Tan and Nicanor Martin filed a
with the Securities and Investigation
Department (SICD) of the Commission

In the meantime, the trustee wrote the


petitioner, Johnson Lee, on March 8, 1988
requesting
him
to
turn
over
to
it
the P1,500,150.00 he received in payment of the
empty bags sold by NMI to VCMI. However, he
failed to do so.[7]

During
the
requisite
preliminary
investigation,
the
petitioner
and
Moreno
submitted their counter-affidavits. The counteraffidavit of the petitioner consisted of five pages.
[8]
After the investigation, two (2) Amended
Informations were filed against the petitioner and
Moreno, with the Regional Trial Court (RTC) of
Negros Occidental.
To prove the loss, destruction or nonavailability of the original copies of the charge
invoices and checks, as well as the authenticity
and due execution thereof, the prosecution
presented Ban Hua Flores, who testified that she
saw the two checks in the office of the petitioner
at the Singson Building, Plaza Moraga, Sta. Cruz,
Manila. Sometime in 1987, she went to the
office of the VMCI and inquired if it still had
copies of the two checks and the clerk thereat
informed her that it would be difficult to locate
21 | P a g e

JURISDICTION; FOR CASE


DIGEST

the checks as they were stored in the bodega,


where many other checks were kept.[10] Flores
also testified that the signatures at the dorsal
portion of the checks were those of the
petitioner, the President of NMI, with whom she
had been working, and that he indorsed and
deposited the same on September 4, 1987 with
the Solidbank, instead of the BPI Plaza Cervantes
branch in Manila, the official depository bank of
NMI. According to Flores, she was able to secure
microfilm copies of the checks from Solidbank,
and was sure that the copies of the checks and
invoices were faithful reproductions of the
original copies thereof.[11]

that even if the contents of the checks were the


subject of inquiry, based on the proofs adduced
by the prosecution, such checks are admissible
in evidence. The Court of Appeals declared that,
in any event, the prosecution proved the loss or
destruction or non-availability of the checks and
charge invoices. The petitioners motion for
reconsideration of the decision suffered the same
fate.

Testifying for the prosecution in obedience


to a subpoena issued by the court, Merlita
Bayaban, Manager for Corporate Affairs of VMCI,
declared that the records section of VMCI, which
had custody of all checks and other corporate
records, was near her office. She testified that
the checks, including their other records, were
lost during the flood in 1985.[12] She also testified
on the Certification[13] issued by Carolina Diaz,
the Comptroller of VMCI, confirming the loss of
the two checks. She, however, admitted that she
did not see the original copies of the
checks[14] and that she was not a signatory
thereto.[15]

(b) whether or not the trial court committed a


grave abuse of its discretion amounting to
excess or lack of jurisdiction in admitting in
evidence the photocopies of the checks and
charge invoices in lieu of the original copies
thereof.

On March 14, 2003, the Court of Appeals


rendered judgment dismissing the petition for
lack of merit.20
The Court of Appeals ruled that the charge
invoices and the checks were not the best
evidence to prove receipt by the accused of the
amounts allegedly misappropriated; hence, the
best evidence rule does not apply. It also held

ISSUE:
(a) whether or not the petition at bar is the
proper remedy of the petitioner;

The Ruling of the Court:


The trial court acts without jurisdiction if it
does not have the legal power to determine the
case; there is excess of jurisdiction where the
respondent, being clothed with the power to
determine the case, oversteps its authority as
determined by law. There is grave abuse of
discretion where the public respondent acts in a
capricious, whimsical, arbitrary or despotic
manner in the exercise of its judgment as to be
said
to
be
equivalent
to
lack
of
jurisdiction.26 Mere abuse of discretion is not
enough. A remedy is plain, speedy and
adequate if it will promptly relieve the petitioner
from the injurious effects of that judgment and
the acts of the tribunal or inferior court. 27 A

petition for certiorari cannot co-exist with an


appeal or any other adequate remedy. The
existence and the availability of the right to
appeal are antithetical to the availment of the
special civil action for certiorari. These two
remedies are mutually exclusive.28
In a petition for certiorari, the jurisdiction of
the court is narrow in scope. It is limited to
resolving only errors of jurisdiction. It is not to
stray at will and resolve questions or issues
beyond its competence such as errors of
judgment. Errors of judgment of the trial court
are to be resolved by the appellate court in the
appeal by and of error or via a petition for review
on certiorari under Rule 45 of the Rules of Court,
as amended. Certiorari will issue only to correct
errors of jurisdiction. It is not a remedy to
correct errors of judgment.29 An error of
judgment is one in which the court may commit
in the exercise of its jurisdiction, and which error
is reversible only by an appeal. Error of
jurisdiction is one where the act complained of
was issued by the court without or in excess of
jurisdiction and which error is correctible only by
the extraordinary writ of certiorari. 30 Certiorari
will not be issued to cure errors made by the trial
court in its appreciation of the evidence of the
parties, its conclusions anchored on the said
findings and its conclusions of law thereon. 31 As
long as the court acts within its jurisdiction, any
alleged errors committed in the exercise of its
discretion will amount to nothing more than mere
errors of judgment, correctible by an appeal if
the aggrieved party raised factual and legal
issues; or a petition for review under Rule 45 of
the Rules of Court if only questions of law are
involved. 32
22 | P a g e

JURISDICTION; FOR CASE


DIGEST

In this case, there is no dispute that the RTC


had jurisdiction over the cases filed by the public
respondent against the petitioner for estafa. The
Order admitting in evidence the photocopies of
the charge invoices and checks was issued by
the RTC in the exercise of its jurisdiction. Even if
erroneous, the same is a mere error of judgment
and not of jurisdiction.
Additionally, the
admission of secondary evidence in lieu of the
original copies predicated on proof of the offeror
of the conditions sine qua non to the admission
of the said evidence is a factual issue addressed
to the sound discretion of the trial court. 33 Unless
grave abuse of discretion amounting to excess or
lack of jurisdiction is shown to have been
committed by the trial court, the resolution of
the trial court admitting secondary evidence
must be sustained.
The remedy of the
petitioner, after the admission of the photocopies
of the charge invoices and the checks, was to
adduce his evidence, and if after trial, he is
convicted, to appeal the decision to the
appropriate appellate court. Moreover, under
Rule 45 of the Rules of Court, as amended, only
questions of law may be properly raised.
With the admissions of the petitioner in his
counter-affidavit, the prosecution even no longer
needed to adduce evidence aliunde to prove the
existence, due execution and the authenticity of
the charge invoices and the checks.
All told then, the prosecution mustered the
requisite quantum of evidence to prove the
predicates to the admission of the photocopies of
the charge invoices and checks.
IN LIGHT OF ALL THE FOREGOING, the
petition is DENIED. The assailed decision of the

Court of Appeals is AFFIRMED.


ORDERED.

No costs.SO

WILLY TAN y CHUA vs. PEOPLE OF THE


PHILIPPINES
G.R. No. 148194: April 12, 2002
FACTS: Willy Tan was found guilty of
bigamy, and then he applied for probation which
was granted by the trial court but the release
was withheld in view of the filing by the
prosecution a motion for modification of penalty.
He later filed a notice of appeal.
ISSUE: Whether or not he is entitled to an
appeal after he has applied for probation.
HELD: In fine, petitioner had taken an
appropriate legal step in filing a notice of appeal
with the trial court. Ordinarily, the Court should
have the case remanded to the Court of Appeals
for further proceedings. The clear impingement
upon petitioners basic right against double
jeopardy, however, should here warrant the
exercise of the prerogative by this Court to relax
the stringent application of the rules on the
matter. When the trial court increased the
penalty on petitioner for his crime of bigamy
after it had already pronounced judgment and on
which basis he then, in fact, applied for
probation, the previous verdict could only be
deemed to have lapsed into finality.
Section 7, Rule 120, of the Rules on Criminal
Procedure that states

Sec. 7. Modification of judgment. A judgment of


conviction may, upon motion of the accused, be
modified or set aside before it becomes final or
before appeal is perfected. Except where the
death penalty is imposed, a judgment becomes
final after the lapse of the period for perfecting
an appeal, or when the sentence has been
partially or totally satisfied or served, or when
the accused has waived in writing his right to
appeal, or has applied for probationimplements a substantive provision of the
Probation Law which enunciates that the mere
filing of an application for probation forecloses
the right to appeal.
SEC. 4. Grant of Probation. Subject to the
provisions of this Decree, the trial court may,
after it shall have convicted and sentenced a
defendant, and upon application by said
defendant within the period for perfecting an
appeal, suspend the execution of the sentence
and place the defendant on probation for such
period and upon such terms and conditions as it
may deem best: Provided, That no application for
probation shall be entertained or granted if the
defendant has perfected the appeal from the
judgment or conviction.
Probation may be granted whether the
sentence imposes a term of imprisonment or a
fine only. An application for probation shall be
filed with the trial court. The filing of the
application shall be deemed a waiver of the right
to appeal.
An order granting or denying probation
shall not be appealable. Such a waiver amounts
to a voluntary compliance with the decision and
23 | P a g e

JURISDICTION; FOR CASE


DIGEST

writes finis to the jurisdiction of the trial court


over the judgment. There is no principle better
settled, or of more universal application, than
that no court can reverse or annul, reconsider or
amend, its own final decree or judgment. Any
attempt by the court to thereafter alter, amend
or modify the same, except in respect to correct
clerical errors, would be unwarranted.

PONCE VS PARANAQUE
FACTS: s "It is a settled rule that relief will not
be granted to a party x x x when the loss of the
remedy at law was due to his own negligence, or
to a mistaken mode of procedure. Before the
Court is a Petition for Review2 on Certiorari of the
March 23, 2007 Decision3 of the Court of Appeals
(CA), as well as its June 4, 2007 Resolution, 4 in
CA-G.R. SP No. 91791, which dismissed V.C.
Ponce Company, Inc.s (VCP) Petiton for
Certiorari. The CA held that VCPs resort to a
petition for certiorari under Rule 65 of the Rules
of Court was inappropriate and that the trial
courts rejection of the commissioners appraisal
report did not amount to a grave abuse of its
discretion. On October 5, 1987, respondent
Municipality (now City) of Paraaque
(municipality) filed a complaint7 against
petitioner VCP for the expropriation of its
property, which is located in the municipalitys
Barrio San Dionisio and covered by Transfer
Certificate of Title (TCT) No. 116554.8 The
municipality intended to develop the property for
its landless residents, in line with the Presidential
Commission on Urban Poors classification of the
site as an area of priority
development.9 Respondent Sampaguita Hills

Homeowners Association, Inc. (SHHAI), consisting


of the propertys actual occupants, who are also
the intended beneficiaries of the action,
intervened in the case.10
On August 23, 2002, the Regional Trial Court
(RTC) of Paraaque, Branch 274, sustained the
municipalitys right to expropriate the said
property11 and to a writ of possession.12 The trial
court also informed the parties in the same Order
of the reckoning period for the determination of
just compensation, thus:
The defenses having thus been ruled upon, the
Court hereby declares that the plaintiff has the
lawful right to take the property sought to be
expropriated for the public use or purpose
described in the complaint, upon the payment of
just compensation to be determined as of the
date of the taking of the property or the filing of
the complaint, whichever came first.13
Ruling of the Trial Court
On March 10, 2005, Judge Fortunito L. Madrona
(Judge Madrona) rendered his Decision rejecting
the report. The trial court explained that just
compensation, as Section 4 of Rule 67 of the
Rules of Court provides,27 must reflect the value
and character of the property sought to be
expropriated, at the time it was taken or at the
time the complaint for expropriation was filed,
whichever came first. Applying this rule to the
facts of the case, the reckoning period should
have been the time of filing of the complaint in
1987 because it took place before the taking of
the property in 2002. The report violated this
rule by using data from 1996 onwards.

The trial court then made an independent finding


based on the evidence already on hand. It
determined that there exists, on record, a
certification from the Office of the City Assessor,
that the propertys market value for the years
1985 to 1993 (which includes the year the
complaint was filed) was P1,366,400.00.28 This
value roughly translates to P75 per square
meter, for a total of P1,372,350.00. The
dispositive portion of the trial courts Decision
reads:
WHEREFORE, based then from [sic] the foregoing
considerations, considering that the land was
then a rawland in 1987 at the time of the filing of
the Amended Complaint for expropriation, it is
the determination of the Court that the just
compensation for the expropriation of the parcel
of land described as Lot No. 4598 of the Cad.
Survey of Paraaque, located in San Dionisio,
Paraaque City, containing an area of 18,298
square meters, registered under Transfer
Certificate of Title No. 116554 of the Registry of
Deeds of Paraaque City in the name of the
defendant V.C. Ponce Co., Inc., is hereby fixed at
P75.00 per square meter, or for an aggregate
valuation of P1,372,350.00.

Ruling of the Court of Appeals

At the outset, the CA observed that an ordinary


appeal under Rule 41 was available to petitioner
and would have constituted a plain, speedy and
adequate remedy to correct any perceived error
in the RTC Decision. VCP, for unknown reasons,
24 | P a g e

JURISDICTION; FOR CASE


DIGEST

failed to avail itself of the said remedy within the


reglementary period. Having lost its right to
appeal, VCP resorted to a Petition for Certiorari in
the hope that it could nevertheless, obtain a
reversal of the RTC Decision. The CA held that
certiorari is unavailing as a substitute for a lost
appeal. The CA brushed aside as unfounded
VCPs excuse that an appeal would be slow and
inadequate. Such excuse, it noted, would allow
any litigant to avail itself of extraordinary
remedies after they lose their right to appeal.41
The CA then held that, even if it were to rule that
certiorari is proper, it would still dismiss the
petition for certiorari. It held that grave abuse of
discretion was not attendant in the trial courts
rejection of the commissioners report. The CA
explained that the trial court has such authority
as long as it finds just cause. The reports
contravention of the principle regarding the
proper reckoning period for the determination of
just compensation is such a cause.42

Issues
1. Is petitioners lack of counsel a justifiable
excuse for the late filing of a Motion for
Reconsideration?
2. Is a Petition for Certiorari the proper remedy to
correct alleged errors in the trial courts
Decision?

RULING:

sThe petition has no merit.


Period for filing a Motion for
Reconsideration not extendible; failure
to file Motion for Reconsideration on
time renders the Decision final.
VCP received the CA Decision on April 10, 2007.
Based on Rule 52 of the Rules of Court57 and Rule
7 of the 2002 Internal Rules of the Court of
Appeals (IRCA),58 VCP had 15 days from its
receipt of the Decision, or until April 25, 2007, to
file a motion for reconsideration, an appeal, or a
motion for new trial. Failure to file the necessary
pleading within the reglementary period would
render the CA Decision final and executory.59
Instead of filing a Motion for Reconsideration on
April 25, 2007, VCP filed a MOTEX on the ground
that its lawyer had withdrawn from the case and
it was still in the process of retaining a new
counsel. The CA was correct in denying
petitioners MOTEX because the period to file a
Motion for Reconsideration is not
extendible.60 The Court has pronounced strict
adherence to the rule laid down in Habaluyas
Enterprises, Inc. v. Judge Japson61that "no motion
for extension of time to file a motion for new trial
or reconsideration may be filed with the
Metropolitan or Municipal Trial Courts, the
Regional Trial Courts, and the Intermediate
Appellate Court (now Court of Appeals)."62 Since
the period to file a Motion for Reconsideration is
not extendible, VCPs MOTEX did not toll the
reglementary period.63 Thus, there being no
Motion for Reconsideration as of April 25, 2007,
the Decision of the CA dated March 23, 2007
became final and executory by operation of
law.64 The CA was correct in denying the Motion
for Reconsideration that VCP had belatedly filed

on May 25, 2007 as its lateness had rendered it


moot.

There is no justification for the application of


equity and for the relaxation of the rules.

VCP urges the Court to relax the rules on the


reglementary period on the ground that it was
impossible for it to meet the deadline without the
aid of counsel.
The Court, in the interest of equity and justice,
sometimes allows a liberal reading of the rules,
so long as the petitioner is able to prove the
existence of cogent reasons to excuse its nonobservance.65 The Court, however, does not find
a justification to warrant such relaxation in this
instance.
It is incumbent upon the client to exert all efforts
to retain the services of new counsel.66 VCP knew
since August 29, 2006, seven months before the
CA rendered its Decision, that it had no counsel.
Despite its knowledge, it did not immediately
hire a lawyer to attend to its affairs. Instead, it
waited until the last minute, when it had already
received the adverse CA Decision on April 10,
2007, to search for a counsel; and even then,
VCP did not rush to meet the deadline. It asked
for an extension of 30 days to file a Motion for
Reconsideration.67 It finally retained the services
of a new counsel on May 24, 2007,68 nine months
from the time that its former counsel withdrew
her appearance. VCP did not even attempt to
25 | P a g e

JURISDICTION; FOR CASE


DIGEST

explain its inaction. The Court cannot grant


equity where it is clearly undeserved by a grossly
negligent party.69 As the Court pronounced in
another case:
x x x Both parties have a right to a speedy
resolution of their case. Not only petitioners, but
also the respondents, have a right to have the
case finally settled without delay.

Furthermore, the failure to file x x x on time was


due primarily to petitioners unwise choices x x
x. They hired their subsequent lawyers too late.
It must be pointed out that petitioners had a
choice of whether to continue the services of
their original lawyer or consent to let him go. x x
x They delayed in engaging their replacement
lawyer. Their poor choices and lack of sufficient
diligence x x x are the main culprits for the
situation they now find themselves in. It would
not be fair to pass on the bad consequences of
their choices to respondents. Petitioners low
regard for the rules or nonchalance toward
procedural requirements x x x has in fact
contributed much to the delay, and hence
frustration of justice, in the present case.70
This Court cannot ascribe good faith to
VCP as it had neglected reglementary
periods in the past.
Another reason that this Court is unable to
accept VCPs plea for indulgence is its
observation that VCP has a penchant for

disregarding procedural rules and the periods


allotted to it for its action.
It did not attend the meetings before the
commissioners for the initial and the final
valuation of its property despite notice. When the
commissioners were finalizing their report to
meet its deadline, VCP asked for an additional
four months to submit its independent valuation
of the property. While the commissioners denied
VCPs request, VCPs action betrays its lack of
consideration for deadlines.
Further, VCP did not file a timely appeal from the
RTC Order denying its Motion for
Reconsideration. VCP received the said Order on
August 24, 2005. Instead of appealing under Rule
41 of the Rules of Court, VCP filed, on the 58th
day from its receipt of the RTC Order, a MOTEX to
file a Petition for Certiorari. While the CA granted
VCPs MOTEX,71 it was correct in ultimately
denying VCPs Petition for Certiorari on the
ground that VCP cannot exploit the remedy of
certiorari after it had lost its right to appeal.
Appeal is a sufficient and adequate
remedy unless the party proves
otherwise.

Appeal is a speedy remedy, as an adverse party


can file its appeal from a final decision or order
immediately after receiving it. A party, who is
alleging that an appeal will not promptly relieve
it of the injurious effects of the judgment, should
establish facts to show how the appeal is not
speedy or adequate.73 VCPs empty
protestations, therefore, fail to impress. There is
no reason, and VCP cannot explain, why an
appeal would not be speedy and adequate to
address its assigned errors.74 VCP cannot
complain of delay because it was guilty of delay
itself, and it even waited until the 58th day of its
receipt of the CA Decision before taking action.
Clearly, petitioner resorted to certiorari as a
substitute for its lost appeal.75 The CA did not err
in dismissing the same.
In sum, VCPs continued negligence, and its
resort to the wrong remedy, placed all perceived
errors in the decisions below beyond the CAs
and this Courts grasp.
WHEREFORE, premises considered, the petition is
DENIED. The March 23, 2007 Decision of the
Court of Appeals in CA-G.R. SP No. 91791, as well
as its June 4, 2007 Resolution, are AFFIRMED.
SO ORDERED.

VCP attempts to extricate itself from the effects


of its negligence by alleging that an appeal
would not have been speedy and adequate for its
purpose. The Court, however, finds no merit in its
contention.1wphi1
A court with appellate jurisdiction can review
both the facts and the law, including questions of
jurisdiction.72 It can set aside an erroneous
decision and even nullify the same, if warranted.

BUSUEGO VS OMBUDSMAN

26 | P a g e

JURISDICTION; FOR CASE


DIGEST

FACTS : S Private respondent Rosa S. Busuego


(Rosa) filed a complaint for: (1) Concubinage
under Article 334 of the Revised Penal Code; (2)
violation of Republic Act No. 9262 (Anti-Violence
Against Women and Their Children); and (3)
Grave Threats under Article 282 of the Revised
Penal Code, before the Office of the Ombudsman
against her husband, Alfredo, with designation
Chief of Hospital, Davao Regional Hospital,
Apokon, Tagum City.
In her complaint, Rosa painted a picture of a
marriage in disarray.

She and Alfredo were married on 12 July 1975 at


the Assumption Church, Davao City. Their union
was blessed with two (2) sons, Alfred and Robert,
born in 1976 and 1978, respectively. Sometime
in 1983, their marriage turned sour. At this time,
Rosa unearthed photographs of, and love letters
addressed to Alfredo from, other women. Rosa
confronted Alfredo who claimed ignorance of the
existence of these letters and innocence of any
wrongdoing.
Purportedly, Alfredo very rarely stayed at home
to spend time with his family. He would come
home late at night on weekdays and head early
to work the next day; his weekends were spent
with his friends, instead of with his family. Rosa
considered herself lucky if their family was able
to spend a solid hour with Alfredo.
Around this time, an opportunity to work as
nurse in New York City, United States of America
(US) opened up for Rosa. Rosa informed Alfredo,
who vehemently opposed Rosas plan to work

abroad. Nonetheless, Rosa completed the


necessary requirements to work in the US and
was scheduled to depart the Philippines in March
1985.

Before leaving, Rosa took up the matter again


with Alfredo, who remained opposed to her
working abroad. Furious with Rosas pressing,
Alfredo took his loaded gun and pointed it at
Rosas right temple, threatening and taunting
Rosa to attempt to leave him and their family.
Alfredo was only staved off because Rosas
mother arrived at the couples house. Alfredo left
the house in a rage: Rosa and her mother heard
gun shots fired outside.
Because of that incident, Rosa acted up to her
plan and left for the US. While in the US, Rosa
became homesick and was subsequently joined
by her children who were brought to the US by
Alfredo. Rosa singularly reared them: Alfred, from
grade school to university, while Robert, upon
finishing high school, went back to Davao City to
study medicine and lived with Alfredo.

During that time his entire family was in the US,


Alfredo never sent financial support. In fact, it
was Rosa who would remit money to Alfredo from
time to time, believing that Alfredo had stopped
womanizing. Rosa continued to spend her annual
vacation in Davao City.
Sometime in 1997, Rosa learned that a certain
Emy Sia (Sia) was living at their conjugal home.

When Rosa asked Alfredo, the latter explained


that Sia was a nurse working at the Regional
Hospital in Tagum who was in a sorry plight as
she was allegedly being raped by Rosas brotherin-law. To get her out of the situation, Alfredo
allowed Sia to live in their house and sleep in the
maids quarters. At that time, Rosa gave Alfredo
the benefit of the doubt.
In October 2005, Rosa finally learned of Alfredos
extra-marital relationships. Robert, who was
already living in Davao City, called Rosa to
complain of Alfredos illicit affairs and shabby
treatment of him. Rosa then rang up Alfredo
which, not surprisingly, resulted in an altercation.
Robert executed an affidavit, corroborating his
mothers story and confirming his fathers illicit
affairs
On 24 June 2008, the Ombudsman issued a Joint
Order4 impleading Sia and de Leon as partyrespondents in the complaint for Concubinage
and directing them to submit their respective
counter-affidavits within a period of time. Copies
of the Joint Order were mailed to Sias and de
Leons last known addresses, as provided by
Rosa to the Ombudsman.
Sia and de Leon did not submit their respective
counter-affidavits: a copy of the Joint Order sent
to Sias last known address was returned to the
Ombudsman with the notation on the Registry
Return Receipt No. 1624 "Return to Sender;
removed," while a copy thereof to de Leon was
received on 3 September 2008 by Ananias de
Leon.5
Apparently still opposed to the Ombudsmans
ruling to simply amend the complaint and
27 | P a g e

JURISDICTION; FOR CASE


DIGEST

implead therein Alfredos alleged mistresses,


Alfredo filed his Comment to the 24 June 2008
Order with Motion to Dismiss and/or Refer the
charges to the Appropriate Provincial/City
Prosecutor6 praying for dismissal of the
complaint for: (1) failure to implead the two
mistresses in violation of Article 344 of the
Revised Penal Code; and in the alternative, (2)
referral of the complaint to the Office of the City
Prosecutor as provided in OMB-DOJ Circular No.
95-001.
Rosa filed a Reply to that latest pleading of
Alfredo.
On 17 April 2009, the Ombudsman issued the
herein assailed Resolution, disposing of the
procedural issues:
Before dwelling into the merits of the case, this
Office finds an urgent need to resolve the
ancillary issues raised by petitioner Dr. Busuego
on: 1.) the alleged legal infirmity of Rosass
initiatory pleading by resorting to a procedural
short cut which would result to the delay in the
disposition of this case; and 2.) the criminal
charges imputed are not in relation to office,
hence, the Office of the Provincial/City Prosecutor
shall investigate and prosecute this case
pursuant to OMB-DOJ Joint Circular No. 95-001,
Series of 1995.
On the first issue, this Office observed that
Busuego had already pointed out in his counterAffidavit the alleged deficiency in the complaint.
Rosa also explained in her Reply that the names
of the mistresses were categorically mentioned
in the complaint. She averred that this Office is
empowered to investigate and prosecute any act

or omission of a public official or employee to the


exclusion of non-government employees. She
stated that the inclusion of the alleged
concubines in the Information to be filed in court
is a matter of procedure, within the competence
of the investigating prosecutor.

WHEREFORE, in view of the foregoing, this Office


finds a prima facie case for violation of Article
334 of the Revised Penal Code (concubinage)
and that petitioner ALFREDO ROMULO BUSUEGO
y ABRIO, and EMY SIA, are probably guilty
thereof.

In order to clarify some matters, including the


said issue, with the parties, the clarificatory
hearing was conducted. It was explained in the
said hearing the need to implead the alleged
concubines in this case pursuant to Article 344 of
the Revised Penal Code and to obviate the
proceedings, Rosa was directed to submit the
addresses of the alleged concubines. Busuegos
position that the said short cut procedure would
delay the proceedings is misplaced. If the case
will be dismissed based on procedural infirmity,
Rosa could still amend her complaint and re-file
this case since the doctrine of res judicata does
not apply in the preliminary investigation stage
of the proceedings.

Let the herewith Information be filed in the


appropriate court.

On the second issue, the motion of Busuego to


refer this case to the Office of the City Prosecutor
was belatedly filed. Record would show that the
motion praying for the referral of this case to the
Office of the City Prosecutor was filed on 17 July
2008, after the parties have already filed all their
pleadings and the case is now ripe for resolution.
Further, referral to the said office is not
mandatory as cited in the said Joint Circular.7
In the same Resolution, the Ombudsman,
ultimately, found probable cause to indict only
Alfredo and Sia of Concubinage and directed the
filing of an Information against them in the
appropriate court:

The charges for: 1.) Concubinage against Alfredo


Romulo Busuego y Abrio and Julie de Leon; 2.)
Grave Threats against Alfredo Romulo y Abrio;
and 3.) violation of RA 9262 (Anti-Violence
Against Women and Children Act), are hereby
DISMISSED for lack of merit.8
Alfredo filed a Partial Motion for Reconsideration
excepting to the Ombudsmans ruling on the
automatic inclusion of Sia as respondent in the
complaint and their indictment for the crime of
Concubinage. Alfredo is adamant that Rosas
complaint should have, at the outset, impleaded
his alleged concubines. Failing such, the
Ombudsman cannot resort to automatic inclusion
of party-respondents, erroneously finding him
and Sia prima facie culpable for Concubinage.
For good measure, Alfredo pointed out that from
Rosas own allegations, she had condoned or
pardoned Alfredos supposed concubinage.
Alfredo likewise submitted Liza S. Diambangans
affidavit, recanting her previous affidavit
corroborating Rosas charges.
Nonetheless, the Ombudsman stood pat on its
ruling, declared that the Partial Motion for
Reconsideration was filed out of time, and gave
scant attention to Liza S. Diambangans affidavit
of recantation:
28 | P a g e

JURISDICTION; FOR CASE


DIGEST

WHEREFORE, all the foregoing considered, this


instant Motion for Reconsideration is hereby
DENIED. The findings in the questioned
Resolution hereby remains undisturbed. Let the
Information for Concubinage be filed in the
proper court against herein Busuego.9
Alfredo now comes to us on petition for certiorari
alleging grave abuse of discretion in the
Ombudsmans finding of probable cause to indict
him and Sia for Concubinage. Alfredos badges of
grave abuse of discretion are the following:
ISSUE: WON OMBUDSMAN ABUSE ITS
DISCRETION
RULING: We sustain the Ombudsman.
The Ombudsman has full discretionary
authority in the determination of probable
cause during a preliminary
investigation.10 This is the reason why
judicial review of the resolution of the
Ombudsman in the exercise of its power
and duty to investigate and prosecute
felonies and/or offenses of public officers is
limited to a determination of whether there
has been a grave abuse of discretion
amounting to lack or excess of jurisdiction.
Courts are not empowered to substitute
their judgment for that of the
Ombudsman.11
By grave abuse of discretion is meant such
capricious and whimsical exercise of
judgment tantamount to lack of
jurisdiction.12 The abuse of discretion must
be so patent and gross as to amount to an
evasion of a positive duty or a virtual

refusal to perform a duty enjoined by law,


or to act at all in contemplation of law, as
where the power is exercised in an
arbitrary and despotic manner by reason of
passion or hostility.13 In this regard,
petitioner failed to demonstrate the
Ombudsman's abuse, much less grave
abuse, of discretion.

the offended party has consented to the


offense or pardoned the offenders.

First. Alfredo insists that the Ombudsmans


automatic inclusion, over his vehement
objections of Sia and de Leon as partyrespondents, violates Article 344 of the
Revised Penal Code and Section 5, Rule 110
of the Rules of Court, which respectively
provide:

S Rule II
PROCEDURE IN CRIMINAL CASES

Art. 344. Prosecution of the crimes of


adultery, concubinage, seduction,
abduction, rape and acts of lasciviousness.
The crimes of adultery and concubinage
shall not be prosecuted except upon a
complaint filed by the offended spouse.
The offended party cannot institute
criminal prosecution without including both
the guilty parties, if they are both alive,
nor, in any case, if he shall have consented
or pardoned the offenders.

We do not agree.
The submission of Alfredo is belied by the
fact that the Ombudsman merely followed
the provisions of its Rules of Procedure.

xxxx
Section 2. Evaluation Upon evaluating the
complaint, the investigating officer shall
recommend whether it may be:
a) dismissed outright for want of palpable merit;
b) referred to respondent for comment;
c) indorsed to the proper government office or
agency which has jurisdiction over the case;
d) forwarded to the appropriate office or official
for fact-finding investigation;
e) referred for administrative adjudication; or

Section 5. Who must prosecute criminal


action. xxx.
The crimes of adultery and concubinage
shall not be prosecuted except upon a
complaint filed by the offended spouse. The
offended party cannot institute criminal
prosecution without including the guilty
parties, if both are alive, nor, in any case, if

f) subjected to a preliminary investigation.


xxxx
Section 4. Procedure The preliminary
investigation of cases falling under the
jurisdiction of the Sandiganbayan and Regional
29 | P a g e

JURISDICTION; FOR CASE


DIGEST

Trial Courts shall be conducted in the manner


prescribed in Section 3, Rule 112 of the Rules of
Court, subject to the following provisions:

the complaint shall be deemed submitted for


resolution on the basis of the evidence on the
record.

a) x x x

f) If, after the filing of the requisite affidavits and


their supporting evidences, there are facts
material to the case which the investigating
officer may need to be clarified on, he may
conduct a clarificatory hearing during which the
parties shall be afforded the opportunity to be
present but without the right to examine or
cross-examine the witness being questioned.
Where the appearance of the parties or
witnesses is impracticable, the clarificatory
questioning may be conducted in writing,
whereby the questions desired to be asked by
the investigating officer or a party shall be
reduced into writing and served on the witness
concerned who shall be required to answer the
same in writing and under oath.

b) After such affidavits have been secured, the


investigating officer shall issue an order,
attaching thereto a copy of the affidavits and
other supporting documents, directing the
respondents to submit, within ten (10) days from
receipt thereof, his counter-affidavits and
controverting evidence with proof of service
thereof on the complainant. The complainant
may file reply affidavits within ten (10) days after
service of the counter-affidavits.
c) If the respondents does not file a counteraffidavit, the investigating officer may consider
the comment filed by him, if any, as his answer
to the complaint. In any event, the respondent
shall have access to the evidence on record.
d) No motion to dismiss shall be allowed except
for lack of jurisdiction.
Neither may a motion for a bill of particulars be
entertained.
If respondent desires any matter in the
complainants affidavit to be clarified, the
particularization thereof may be done at the time
of the clarificatory questioning in the manner
provided in paragraph (f) of this section.
e) If the respondents cannot be served with the
order mentioned in paragraph 6 hereof, or
having been served, does not comply therewith,

g) Upon the termination of the preliminary


investigation, the investigating officer shall
forward the records of the case together with his
resolution to the designated authorities for their
appropriate action thereon.
No information may be filed and no complaint
may be dismissed without the written authority
or approval of the ombudsman in cases falling
within the jurisdiction of the Sandiganbyan, or of
the proper Deputy Ombudsman in all other
cases. (Emphasis supplied).
Notably, Rosas complaint contained not just the
Concubinage charge, but other charges: violation
of Republic Act No. 9262 and Grave Threats.
Upon the Ombudsmans perusal, the complaint
was supported by affidavits corroborating Rosas

accusations. Thus, at that stage, the


Ombudsman properly referred the complaint to
Alfredo for comment. Nonetheless, while the
Ombudsman found no reason for outright
dismissal, it deemed it fit to hold a clarificatory
hearing to discuss the applicability of Article 344
of the Revised Penal Code, the issue having been
insisted upon by Alfredo.
Surely the procedural sequence of referral of the
complaint to respondent for comment and
thereafter the holding of a clarificatory hearing is
provided for in paragraph b, Section 2 and
paragraphs d and f, Section 4 of Rule II, which we
have at the outset underscored. Thus did the
Ombudsman rule:
In order to clarify some matters, including the
said issue, with the parties, the clarificatory
hearing was conducted. It was explained in the
said hearing the need to implead the alleged
concubines in this case pursuant to Article 344 of
the Revised Penal Code and to obviate the
proceedings, Rosa was directed to submit the
addresses of the alleged concubines. Busuegos
position that the said short cut procedure would
delay the proceedings is misplaced. If the case
will be dismissed based on procedural infirmity,
Rosa could still amend her complaint and re-file
this case since the doctrine of res judicata does
not apply in the preliminary investigation stage
of the proceedings.14
The Ombudsman merely facilitated the
amendment of the complaint to cure the defect
pointed out by Alfredo. We agree with the
Ombudsman that it would be superfluous to
dismiss the complaint when amendment thereof
30 | P a g e

JURISDICTION; FOR CASE


DIGEST

is allowed by its Rules of Procedure15 and the


Rules of Court.16

respondent Ombudsman may, in the exercise of


its primary jurisdiction take over at any stage.

Second. Alfredo claims that the Ombudsman


should have referred Rosas complaint to the
Department of Justice (DOJ), since the crime of
Concubinage is not committed in relation to his
being a public officer. This is not a new
argument.

Thus, with the jurisprudential declarations that


the Ombudsman and the DOJ have concurrent
jurisdiction to conduct preliminary investigation,
the respective heads of said offices came up with
OMB-DOJ Joint Circular No. 95-001 for the proper
guidelines of their respective prosecutors in the
conduct of their investigations, to wit:

The Ombudsmans primary jurisdiction, albeit


concurrent with the DOJ, to conduct preliminary
investigation of crimes involving public officers,
without regard to its commission in relation to
office, had long been settled in Sen. Honasan II v.
The Panel of Investigating Prosecutors of
DOJ,17 and affirmed in subsequent cases:
The Constitution, Section 15 of the Ombudsman
Act of 1989 and Section 4 of the Sandiganbayan
Law, as amended, do not give to the
Ombudsman exclusive jurisdiction to investigate
offenses committed by public officers or
employees. The authority of the Ombudsman to
investigate offenses involving public officers or
employees is concurrent with other government
investigating agencies such as provincial, city
and state prosecutors. However, the
Ombudsman, in the exercise of its primary
jurisdiction over cases cognizable by the
Sandiganbayan, may take over, at any stage,
from any investigating agency of the
government, the investigation of such cases.
In other words, respondent DOJ Panel is not
precluded from conducting any investigation of
cases against public officers involving violations
of penal laws but if the cases fall under the
exclusive jurisdiction of the Sandiganbayan, the

OMB-DOJ JOINT CIRCULAR NO. 95-001


Series of 1995
ALL GRAFT INVESTIGATION/SPECIAL
PROSECUTIONOFFICERS OF THE OFFICE OF THE
OMBUDSMAN
TO: ALL REGIONAL STATE PROSECUTORS AND
THEIR ASSISTANTS, PROVINCIAL/CITY
PROSECUTORS AND THEIR ASSISTANTS, STATE
PROSECUTORS ANDPROSECUTING ATTORNEYS
OF THE DEPARTMENT OFJUSTICE.
SUBJECT: HANDLING COMPLAINTS FILED AGAINST
PUBLICOFFICERS AND EMPLOYEES, THE
CONDUCT OFPRELIMINARY INVESTIGATION,
PREPARATION OFRESOLUTIONS AND
INFORMATIONS AND PROSECUTION OF CASES BY
PROVINCIAL AND CITY PROSECUTORS AND THEIR
ASSISTANTS.
x--------------------------------------------------------------------------------------x
In a recent dialogue between the OFFICE OF THE
OMBUDSMAN and the DEPARTMENT OF JUSTICE,
discussion centered around the latest

pronouncement of the SUPREME COURT on the


extent to which the OMBUDSMAN may call upon
the government prosecutors for assistance in the
investigation and prosecution of criminal cases
cognizable by his office and the conditions under
which he may do so. Also discussed was Republic
Act No. 7975 otherwise known as "AN ACT TO
STRENGTHEN THE FUNCTIONAL AND
STRUCTURAL ORGANIZATION OF THE
SANDIGANBAYAN, AMENDING FOR THE PURPOSE
PRESIDENTIAL DECREE NO. 1606, AS AMENDED"
and its implications on the jurisdiction of the
office of the Ombudsman on criminal offenses
committed by public officers and employees.
Concerns were expressed on unnecessary delays
that could be caused by discussions on
jurisdiction between the OFFICE OF THE
OMBUDSMAN and the DEPARTMENT OF JUSTICE,
and by procedural conflicts in the filing of
complaints against public officers and
employees, the conduct of preliminary
investigations, the preparation of resolutions and
informations, and the prosecution of cases by
provincial and city prosecutors and their
assistants as DEPUTIZED PROSECUTORS OF THE
OMBUDSMAN.
Recognizing the concerns, the OFFICE OF THE
OMBUDSMAN and the DEPARTMENT OF JUSTICE,
in a series of consultations, have agreed on the
following guidelines to be observed in the
investigation and prosecution of cases against
public officers and employees:
1. Preliminary investigation and prosecution of
offenses committed by public officers and
employees IN RELATION TO OFFICE whether
cognizable by the SANDIGANBAYAN or the
31 | P a g e

JURISDICTION; FOR CASE


DIGEST

REGULAR COURTS, and whether filed with the


OFFICE OF THE OMBUDSMAN or with the OFFICE
OF THE PROVINCIAL/CITY PROSECUTOR shall be
under the control and supervision of the office of
the OMBUDSMAN.
2. Unless the Ombudsman under its
Constitutional mandate finds reason to believe
otherwise, offenses NOT IN RELATION TO OFFICE
and cognizable by the REGULAR COURTS shall be
investigated and prosecuted by the OFFICE OF
THE PROVINCIAL/CITY PROSECUTOR, which shall
rule thereon with finality.
3. Preparation of criminal information shall be the
responsibility of the investigating officer who
conducted the preliminary investigation.
Resolutions recommending prosecution together
with the duly accomplished criminal informations
shall be forwarded to the appropriate approving
authority.
4. Considering that the OFFICE OF THE
OMBUDSMAN has jurisdiction over public officers
and employees and for effective monitoring of all
investigations and prosecutions of cases
involving public officers and employees, the
OFFICE OF THE PROVINCIAL/CITY PROSECUTOR
shall submit to the OFFICE OF THE OMBUDSMAN
a monthly list of complaints filed with their
respective offices against public officers and
employees.
xxxx
A close examination of the circular supports the
view of the respondent Ombudsman that it is just
an internal agreement between the Ombudsman
and the DOJ.

Sections 2 and 4, Rule 112 of the Revised Rules


on Criminal Procedure on Preliminary
Investigation, effective December 1, 2000, to wit:

opportunity to submit controverting evidence.


Otherwise, he shall recommend the dismissal of
the complaint.

SEC. 2. Officers authorized to conduct


preliminary investigations

Within five (5) days from his resolution, he shall


forward the record of the case to the provincial
or city prosecutor or chief state prosecutor, or to
the Ombudsman or his deputy in cases of
offenses cognizable by the Sandiganbayan in the
exercise of its original jurisdiction. They shall act
on the resolution within ten (10) days from their
receipt thereof and shall immediately inform the
parties of such action.

The following may conduct preliminary


investigations:
(a) Provincial or City Prosecutors and their
assistants;
(b) Judges of the Municipal Trial Courts and
Municipal Circuit Trial Courts;
(c) National and Regional State Prosecutors; and
(d) Other officers as may be authorized by law.
Their authority to conduct preliminary
investigation shall include all crimes cognizable
by the proper court in their respective territorial
jurisdictions.
SEC. 4. Resolution of investigating prosecutor
and its review. - If the investigating prosecutor
finds cause to hold the respondent for trial, he
shall prepare the resolution and information. He
shall certify under oath in the information that
he, or as shown by the record, an authorized
officer, has personally examined the complainant
and his witnesses; that there is reasonable
ground to believe that a crime has been
committed and that the accused is probably
guilty thereof; that the accused was informed of
the complaint and of the evidence submitted
against him; and that he was given an

No complaint or information may be filed or


dismissed by an investigating prosecutor without
the prior written authority or approval of the
provincial or city prosecutor or chief state
prosecutor or the Ombudsman or his deputy.
Where the investigating prosecutor recommends
the dismissal of the complaint but his
recommendation is disapproved by the provincial
or city prosecutor or chief state prosecutor or the
Ombudsman or his deputy on the ground that a
probable cause exists, the latter may, by himself
file the information against the respondent, or
direct another assistant prosecutor or state
prosecutor to do so without conducting another
preliminary investigation.
If upon petition by a proper party under such
rules as the Department of Justice may prescribe
or motu proprio, the Secretary of Justice reverses
or modifies the resolution of the provincial or city
prosecutor or chief state prosecutor, he shall
direct the prosecutor concerned either to file the
corresponding information without conducting
another preliminary investigation, or to dismiss
32 | P a g e

JURISDICTION; FOR CASE


DIGEST

or move for dismissal of the complaint or


information with notice to the parties. The same
Rule shall apply in preliminary investigations
conducted by the officers of the Office of the
Ombudsman.
confirm the authority of the DOJ prosecutors to
conduct preliminary investigation of criminal
complaints filed with them for offenses
cognizable by the proper court within their
respective territorial jurisdictions, including those
offenses which come within the original
jurisdiction of the Sandiganbayan; but with the
qualification that in offenses falling within the
original jurisdiction of the Sandiganbayan, the
prosecutor shall, after their investigation,
transmit the records and their resolutions to the
Ombudsman or his deputy for appropriate action.
Also, the prosecutor cannot dismiss the
complaint without the prior written authority of
the Ombudsman or his deputy, nor can the
prosecutor file an Information with the
Sandiganbayan without being deputized by, and
without prior written authority of the
Ombudsman or his deputy.
xxxx
To reiterate for emphasis, the power to
investigate or conduct preliminary investigation
on charges against any public officers or
employees may be exercised by an investigator
or by any provincial or city prosecutor or their
assistants, either in their regular capacities or as
deputized Ombudsman prosecutors. The fact
that all prosecutors are in effect deputized
Ombudsman prosecutors under the OMB-DOJ
circular is a mere superfluity. The DOJ Panel need
not be authorized nor deputized by the

Ombudsman to conduct the preliminary


investigation for complaints filed with it because
the DOJs authority to act as the principal law
agency of the government and investigate the
commission of crimes under the Revised Penal
Code is derived from the Revised Administrative
Code which had been held in the Natividad case
citation omitted as not being contrary to the
Constitution. Thus, there is not even a need to
delegate the conduct of the preliminary
investigation to an agency which has the
jurisdiction to do so in the first place. However,
the Ombudsman may assert its primary
jurisdiction at any stage of the investigation.
(Emphasis supplied).
In Honasan II, although Senator Gregorio
"Gringo" Honasan was a public officer who was
charged with coup detat for the occupation of
Oakwood on 27 July 2003, the preliminary
investigation therefor was conducted by the DOJ.
Honasan questioned the jurisdiction of the DOJ to
do so, proferring that it was the Ombudsman
which had jurisdiction since the imputed acts
were committed in relation to his public office.
We clarified that the DOJ and the Ombudsman
have concurrent jurisdiction to investigate
offenses involving public officers or employees.
Nonetheless, we pointed out that the
Ombudsman, in the exercise of its primary
jurisdiction over cases cognizable by the
Sandiganbayan, may take over, at any stage,
from any investigating agency of the
government, the investigation of such cases.
Plainly, applying that ruling in this case, the
Ombudsman has primary jurisdiction, albeit
concurrent with the DOJ, over Rosas complaint,
and after choosing to exercise such jurisdiction,
need not defer to the dictates of a respondent in

a complaint, such as Alfredo. In other words, the


Ombudsman may exercise jurisdiction to the
exclusion of the DOJ.
SMART VS ALDECOA
FACTS: Before the Court is a Petition for Review
on Certiorari under Rule 45 of the Rules of Court
filed by petitioner Smart Communications, Inc.,
seeking the reversal of the Decision1 dated July
16, 2004 and Resolution2 dated December 9,
2004 of the Court of Appeals in CA-G.R. CV No.
71337. The appellate court (I) reversed and set
aside the Order3 dated January 16, 2001 of the
Regional Trial Court (RTC), Branch 23, of Roxas,
Isabela, in Civil Case No. Br. 23-632-2000
dismissing the complaint for abatement of
nuisance and injunction against petitioner, and
(2) entered a new judgment declaring
petitioner's cellular base station located in
Barangay Vira, Municipality of Roxas, Province of
Isabela, a nuisance and ordering petitioner to
cease and desist from operating the said cellular
base station.
The instant Petition arose from the following
facts:
Petitioner is a domestic corporation engaged in
the telecommunications business. On March 9,
2000, petitioner entered into a contract of
lease4 with Florentino Sebastian in which the
latter agreed to lease to the former a piece of
vacant lot, measuring around 300 square meters,
located in Barangay Vira, Roxas, Isabela (leased
property).Petitioner, through its contractor,
Allarilla Construction, immediately constructed
and installed a cellular base station on the leased
property. Inside the cellular base station is a
33 | P a g e

JURISDICTION; FOR CASE


DIGEST

communications tower, rising as high as150 feet,


with antennas and transmitters; as well as a
power house open on three sides containing a
25KVA diesel power generator. Around and close
to the cellular base station are houses, hospitals,
clinics, and establishments, including the
properties of respondents Arsenio Aldecoa, Jose
B. Torre, Conrado U. Pua, Gregorio V. Mansano,
Jerry Corpuz, and Estelita Acosta.
Civil Case No. Br. 23-632-2000 was set for pretrial on September 28, 2000.9
On September 11, 2000, petitioner filed its PreTrial Brief in which it identified the following
issues:
4.1. Whether respondents have a cause of action
against the petitioner SMART for this Honorable
Court to issue a Preliminary Mandatory Injunction
over the SMART tower in Roxas, Isabela as it
allegedly poses a threat to the lives and safety of
the residents within the area and if respondents
are entitled to moral and exemplary damages as
well as attorneys fees and expenses of litigation.
4.2 Whether the complaint should be dismissed
in that the claim or demand set forth in the
Complaint is fictitious, imaginary, sham and
without any real basis.
4.3. What petitioner SMART is entitled under its
compulsory counterclaim against respondents for
moral and exemplary damages, attorneys fees,
and other expenses of litigation.10
On even date, petitioner filed a Motion for
Summary Judgment that reads:

Petitioner SMART Communications Inc., thru


counsel, respectfully manifests that:
1. There is no need for a full-blown trial as the
causes of action and issues have already been
identified in all the pleadings submitted to this
Honorable court by both respondents and
petitioner
2. There is clearly no genuine issue as to any
material fact or cause in the action.
3. There is no extreme urgency to issue a
Preliminary Mandatory Injunction as stated in an
affidavit executed by SMART Senior Supervisor
Andres V. Romero in an affidavit hereto attached
as Annex "A"
4. Petitioner seeks immediate declaratory relief
from respondents contrived allegations as set
forth in their complaint;
Wherefore, it is most respectfully prayed of this
Honorable Court that summary judgment be
rendered pursuant to Rule 35 of the Revised
Rules of Court.11
Respondents filed their Pre-Trial Brief on
September 21, 2000, proposing to limit the
issues,
viz:
- Whether petitioners communications tower is a
nuisance per se/per accidens and together with
its standby generator maybe abated for posing
danger to the property and life and limb of the
residents of Vira, Roxas, Isabela more particularly

the respondents and those whose houses are


clustered around or in the periphery of the cell
site.
- Damages, attorneys fees, litigation expenses
and other claims.12
Respondents likewise filed on September 21,
2000 their Opposition to petitioners Motion for
Summary Judgment, maintaining that there were
several genuine issues relating to the cause of
action and material facts of their Complaint.
They asserted that there was a need for a full
blown trial to prove the allegations in their
Complaint, as well as the defenses put up by
petitioner.13
In its Order14 dated September 28, 2000, the RTC
indefinitely postponed the pre-trial until it has
resolved petitioners Motion for Summary
Judgment. In the same Order, the RTC directed
the counsels of both parties to submit their
memoranda, including supporting affidavits and
other documents within 30 days.
Petitioner submitted its Memorandum15 on
October 26, 2000; while respondents, following
several motions for extension of time, filed their
Memorandum16 on November 22, 2000. In their
Memorandum, respondents additionally alleged
that:
The cellsite base station is powered by a roaring
25 KVA power generator. Operated 24 hours
since it started more than a month ago, it has
sent "jackhammers into the brains" of all the
inhabitants nearby. Everyone is going crazy. A
resident just recently operated for breast cancer
is complaining that the noise emanating from the
34 | P a g e

JURISDICTION; FOR CASE


DIGEST

generator is fast tracking her appointment with


death. She can no longer bear the unceasing and
irritating roar of the power generator.
For this, the residents, led by the respondents,
sought a noise emission test of the power
generator of petitioner SMART Communications
with the DENR. The test was conducted on
November 14 and 15, 2000 and the result shows
that the petitioners power generator failed the
noise emission test, day and night time. Result of
this test was furnished the Municipal Mayor of
Roxas, Isabela (See Communication of DENR
Regional Director Lorenzo C. Aguiluz to Mayor
Benedicto Calderon dated November 16, 2000
and the Inspection Monitoring Report).
With these findings, the power generator is also
a nuisance. It must also be abated.17
On January 16, 2001, the RTC issued its Order
granting petitioners Motion for Summary
Judgment and dismissing respondents
Complaint. The RTC ruled as follows:
What is of prime importance is the fact that
contrary to the respondents speculation, the
radio frequency radiation as found out by the
Department of Health is much lower compared to
that of TV and radio broadcast. The respondents
counter to this claim is that the Department of
Health was misled. This is a mere conclusion of
the respondents.
The respondents in opposing the Smarts
construction of their cellsite is anchored on the
supposition that the operation of said cellsite
tower would pose a great hazard to the health of
the alleged cluster of residents nearby and the

perceived danger that the said tower might also


collapse in case of a strong typhoon that fell the
Mobiline Cellsite tower of Mobiline (sic). The
structured built of the Smarts Cellsite tower is
similar to that of the Mobiline.
Now, as to the Courts assessment of the
circumstances obtaining, we find the claim of the
respondents to be highly speculative, if not an
isolated one. Elsewhere, we find several cellsite
towers scattered (sic) allover, both of the Smart,
Globe, and others, nay even in thickly populated
areas like in Metro Manila and also in key cities
nationwide, yet they have not been outlawed or
declared nuisance as the respondents now want
this Court to heed. To the thinking of the Court,
the respondents are harping imagined perils to
their health for reason only known to them
perhaps especially were we to consider that the
Brgy. Captain of Vira earlier gave its imprimatur
to this project. Noteworthy is the fact that the
alleged cluster of residential houses that abut
the cellsite tower in question might be
endangered thereby, the respondents are but a
few of those residents. If indeed, all those
residents in Vira were adversely affected for the
perceived hazards posed by the tower in
question, they should also have been joined in as
respondents in a class suit. The sinister motive is
perhaps obvious.
All the foregoing reasons impel this Court to
grant the petitioners motion for the dismissal of
the complaint, the perceived dangers being
highly speculative without any bases in fact.
Allegations in the complaint being more
imaginary than real, do not constitute factual
bases to require further proceeding or a trial. As
to the claim that there is no certification or

clearance from the DENR for the petitioner to lay


in wait before the construction, suffice it to say
that no action as yet has been taken by said
office to stop the ongoing operation of said
cellsite now in operation. There has been no hue
and cry from among the greater majority of the
people of Roxas, Isabela, against it. Al contrario,
it is most welcome to them as this is another
landmark towards the progress of this town.18
The dispositive portion of the RTC Order reads:
WHEREFORE, in view of the foregoing
considerations, the Court hereby renders
judgment dismissing the complaint as the
allegations therein are purely speculative and
hence no basis in fact to warrant further
proceedings of this case.
The Court finds no compelling grounds to award
damages.
Without costs.19
In another Order20 dated February 27, 2001, the
RTC denied respondents Motion for
Reconsideration.
Respondents filed an appeal with the Court of
Appeals, docketed as CA-G.R. CV No. 71337.
The Court of Appeals rendered its Decision on
July 16, 2004. The appellate court declared the
cellular base station of petitioner a nuisance that
endangered the health and safety of the
residents of Barangay Vira, Roxas, Isabela
because: (1) the locational clearance granted to
petitioner was a nullity due to the lack of
35 | P a g e

JURISDICTION; FOR CASE


DIGEST

approval by majority of the actual residents of


the
barangay and a barangay resolution endorsing
the construction of the cellular base station; and
(2) the sound emission of the generator at the
cellular base station exceeded the Department of
Environment and Natural Resources (DENR)
standards. Consequently, the Court of Appeals
decreed:
WHEREFORE, the appealed decision is hereby
REVERSED and SET ASIDE. A new one is entered
declaring the communications tower or base
station of petitioner Smart Communications, Inc.
located at Brigido Pascual Street in Vira,
Municipality of Roxas, Province of Isabela, a
nuisance. Petitioner is ordered to cease and
desist from operating the said tower or station.21
Petitioner filed its Motion for Reconsideration
arguing that: (1) the basis for the judgment of
the appellate court that the cellular base station
was a nuisance had been extinguished as the
generator subject of the Complaint was already
removed; and (2) there had been substantial
compliance in securing all required permits for
the cellular base station.22
The Court of Appeals, in a Resolution dated
December 9, 2004,refused to reconsider its
earlier Decision, reasoning that:
Petitioner principally anchors its pleas for
reconsideration on the Certification issued by
Roxas, Isabela Municipal Engineer Virgilio
Batucal, declaring that upon actual inspection,
no Denyo Generator Set has been found in the
companys cell site in Roxas, Isabela. We hold,

however, that the certification dated August 12,


2004, taken on its own, does not prove Smarts
allegation that it has abandoned using dieselpowered generators since January 2002.
Respondents current photographs of the cell site
clearly shows (sic) that Smart continues to use a
mobile generator emitting high level of noise and
fumes.
ISSUE: 21.0 The Court of Appeals erred
when it encroached upon an executive
function of determining the validity of a
locational clearance when it declared,
contrary to the administrative findings of
the Housing Land Use and Regulatory
Board ("HLURB"), that the locational
clearance of Petitioner was void.
22.0 The Court of Appeals erred when it
resolved an issue that was not submitted
to it for resolution and in the process had
usurped a purely executive function.
23.0 The Court of Appeals erred in
declaring Petitioners entire base station a
nuisance considering that it was only a
small part of the base station, a generator
that initially powered the base station, that
was reportedly producing unacceptable
levels of noise.
24.0 The Court of Appeals erred in not
considering that the supervening event of
shut down and pull out of the generator in
the base station, the source of the
perceived nuisance, made the complaint for
abatement of nuisance academic.24
RULING:

S The Petition is partly meritorious. While the


Court agrees that the Court of Appeals should
not have taken cognizance of the issue of
whether the locational clearance for petitioners
cellular base station is valid, the Court will still
not reinstate the RTC Order dated January 16,
2001 granting petitioners Motion for Summary
Judgment and entirely dismissing Civil Case No.
Br. 23-632-2000. The issues of (1) whether
petitioners cellular base station is a nuisance,
and (2) whether the generator at petitioners
cellular base station is, by itself, also a nuisance,
ultimately involve disputed or contested factual
matters that call for the presentation of evidence
at a full-blown trial.
On the finding of the Court of
Appeals that petitioners locational
clearance for its cellular base station
is a nullity
Based on the principle of exhaustion of
administrative remedies and its corollary
doctrine of primary jurisdiction, it was premature
for the Court of Appeals to take cognizance of
and rule upon the issue of the validity or nullity
of petitioners locational clearance for its cellular
base station.
The principle of exhaustion of administrative
remedies and the doctrine of primary jurisdiction
were explained at length by the Court in Province
of Zamboanga del Norte v. Court of Appeals,25 as
follows:
The Court in a long line of cases has held that
before a party is allowed to seek the intervention
of the courts, it is a pre-condition that he avail
himself of all administrative processes afforded
36 | P a g e

JURISDICTION; FOR CASE


DIGEST

him. Hence, if a remedy within the administrative


machinery can be resorted to by giving the
administrative officer every opportunity to
decide on a matter that comes within his
jurisdiction, then such remedy must be
exhausted first before the court's power of
judicial review can be sought. The premature
resort to the court is fatal to one's cause of
action. Accordingly, absent any finding of waiver
or estoppel, the case may be dismissed for lack
of cause of action.
The doctrine of exhaustion of administrative
remedies is not without its practical and legal
reasons. Indeed, resort to administrative
remedies entails lesser expenses and provides
for speedier disposition of controversies. Our
courts of justice for reason of comity and
convenience will shy away from a dispute until
the system of administrative redress has been
completed and complied with so as to give the
administrative agency every opportunity to
correct its error and to dispose of the case.
xxxx
The doctrine of primary jurisdiction does not
warrant a court to arrogate unto itself the
authority to resolve a controversy the jurisdiction
over which is initially lodged with an
administrative body of special competence.
We have held that while the administration
grapples with the complex and multifarious
problems caused by unbridled exploitation of our
resources, the judiciary will stand clear. A long
line of cases establishes the basic rule that the
court will not interfere in matters which are
addressed to the sound discretion of government

agencies entrusted with the regulation of


activities coming under the special technical
knowledge and training of such agencies.
In fact, a party with an administrative remedy
must not merely initiate the prescribed
administrative procedure to obtain relief, but also
pursue it to its appropriate conclusion before
seeking judicial intervention. The underlying
principle of the rule on exhaustion of
administrative remedies rests on the
presumption that when the administrative body,
or grievance machinery, is afforded a chance to
pass upon the matter, it will decide the same
correctly. (Citations omitted.)
The Court again discussed the said principle and
doctrine in Addition Hills Mandaluyong Civic &
Social Organization, Inc. v. Megaworld Properties
& Holdings, Inc., et al.,26 citing Republic v.
Lacap,27 to wit:
We have consistently declared that the doctrine
of exhaustion of administrative remedies is a
cornerstone of our judicial system. The thrust of
the rule is that courts must allow administrative
agencies to carry out their functions and
discharge their responsibilities within the
specialized areas of their respective competence.
The rationale for this doctrine is obvious. It
entails lesser expenses and provides for the
speedier resolution of controversies. Comity and
convenience also impel courts of justice to shy
away from a dispute until the system of
administrative redress has been completed.
In the case of Republic v. Lacap, we expounded
on the doctrine of exhaustion of administrative

remedies and the related doctrine of primary


jurisdiction in this wise:
The general rule is that before a party may seek
the intervention of the court, he should first avail
of all the means afforded him by administrative
processes. The issues which administrative
agencies are authorized to decide should not be
summarily taken from them and submitted to a
court without first giving such administrative
agency the opportunity to dispose of the same
after due deliberation.
Corollary to the doctrine of exhaustion of
administrative remedies is the doctrine of
primary jurisdiction; that is, courts cannot or will
not determine a controversy involving a question
which is within the jurisdiction of the
administrative tribunal prior to the resolution of
that question by the administrative tribunal,
where the question demands the exercise of
sound administrative discretion requiring the
special knowledge, experience and services of
the administrative tribunal to determine
technical and intricate matters of fact. (Citations
omitted.)
The Housing and Land Use Regulatory Board
(HLURB)28 is the planning, regulatory, and quasijudicial instrumentality of government for land
use development.29 In the exercise of its
mandate to ensure rational land use by
regulating land development, it issued HLURB
Resolution No.R-626, series of 1998, Approving
the Locational Guidelines for Base Stations of
Cellular Mobile Telephone Service, Paging
Service, Trunking Service, Wireless Loop Service
and Other Wireless Communication Services
(HLURB Guidelines). Said HLURB Guidelines aim
37 | P a g e

JURISDICTION; FOR CASE


DIGEST

to protect" providers and users, as well as the


public in general while ensuring efficient and
responsive communication services."

Correlatively, the HLURB provides administrative


remedies for non-compliance with its
requirements.

Indeed, the HLURB Guidelines require the


submission of several documents for the
issuance of a locational clearance for a cellular
base station, including:

In 2000, when factual precedents to the instant


case began to take place, HLURB Resolution No.
R-586, series of 1996, otherwise known as the
1996 HLURB Rules of Procedure, as amended,
was in effect. The original 1996 HLURB Rules of
Procedure was precisely amended by HLURB
Resolution No. R-655, series of 1999, "so as to
afford oppositors with the proper channel and
expeditious means to ventilate their objections
and oppositions to applications for permits,
clearances and licenses, as well as to protect the
rights of applicants against frivolous oppositions
that may cause undue delay to their projects.
"Under the 1996 HLURB Rules of Procedure, as
amended, an opposition to an application for a
locational clearance for a cellular base station or
a complaint for the revocation of a locational
clearance for a cellular base station already
issued, is within the original jurisdiction of the
HLURB Executive Committee. Relevant provisions
read:

IV. Requirements and Procedures in Securing


Locational Clearance
A. The following documents shall be submitted in
duplicate:
xxxx
g. Written Consent:
g.1 Subdivisions
xxxx
g. 1.2 In the absence of an established
Homeowners Association, consent/affidavit of
non-objection from majority of actual occupants
and owners of properties within a radial distance
equivalent to the height of the proposed base
station measured from its base, including all
those whose properties is adjoining the proposed
site of the base station.(Refer to Figure 2)
xxxx
h. Barangay Council Resolution endorsing the
base station.

RULE III
Commencement of Action, Summons and Answer
xxxx
SECTION 2. Opposition to Application for
Permit/License/ Clearance. When an opposition
is filed to an application for a license, permit or
clearance with the Board or any of its Regional
Field Office, the Regional Officer shall make a
preliminary evaluation and determination
whether the case is impressed with significant

economic, social, environmental or national


policy implications. If he/she determines that the
case is so impressed with significant economic,
social, environmental or national policy
implications, such as, but not limited to:
1) Projects of national significance, for purposes
of this rule, a project is of national significance if
it is one or falls under any of those enumerated
in Rule III, Section 3 of these Rules, as amended;
2) Those involving zoning variances and
exceptions;
3) Those involving significant public interest or
policy issues;
4) Those endorsed by the zoning administrators
of local government units.
The Regional Officer shall cause the records of
the case to be transmitted to the Executive
Committee which shall assume original
jurisdiction over the case, otherwise, the
Regional Officer shall act on and resolve the
Opposition.
SECTION 3. A project is of national significance if
it involves any of the following:
a) Power generating plants (e.g., coal-fired
thermal plants)and related facilities (e.g.,
transmission lines);
b) Airport/seaports; dumping sites/sanitary
landfills; reclamation projects;
c) Large-scale piggery and poultry projects;
38 | P a g e

JURISDICTION; FOR CASE


DIGEST

d) Mining/quarrying projects;
e) National government centers;
f) Golf courses;
g) Fish ponds and aqua culture projects;
h) Cell sites and telecommunication facilities;
i) Economic zones, regional industrial centers,
regional agro-industrial centers, provincial
industrial centers;
j) All other industrial activities classified as highintensity uses (1-3 Projects).
SECTION 4. Any party aggrieved, by reason of
the elevation or non-elevation of any contested
application by the Regional Officer, may file a
verified petition for review thereof within thirty
(30) days from receipt of the notice of elevation
or non-elevation of the contested application
with the Executive Committee which shall
resolve whether it shall assume jurisdiction
thereon.
The contested application for clearance, permit
or license shall be treated as a complaint and all
other provisions of these rules on complaints not
inconsistent with the preceding section shall, as
far as practicable, be made applicable to
oppositions except that the decision of the Board
en banc on such contested applications shall be
final and executory as provided in Rule XIX,
Section 2 of these Rules, as amended.

The Rules pertaining to contested applications


for license, permit or clearance shall, by analogy,
apply to cases filed primarily for the revocation
thereof.

appeal, also provided in the 1996 HLURB Rules of


Procedure, as amended:
RULE XII
Petition for Review

xxxx
RULE XVII
Proceedings Before the Board of Commissioners
xxxx
SECTION 15. The Executive Committee. The
Executive Committee shall be composed of the
four regular Commissioners and the Ex-Officio
Commissioner from the Department of Justice.
xxxx
The Executive Committee shall act for the Board
on policy matters, measures or proposals
concerning the management and substantive
administrative operations of the Board subject to
ratification by the Board en banc, and shall
assume original jurisdiction over cases involving
opposition to an application for license, permit or
clearance for projects or cases impressed with
significant economic, social, environmental or
national policy implications or issues in
accordance with Section 2, Rule II of these Rules,
as amended. It shall also approve the proposed
agenda of the meetings of the Board en banc.
(Emphases supplied.)
After the HLURB Executive Committee had
rendered its Decision, the aggrieved party could
still avail itself of a system of administrative

SECTION 1. Petition for Review. Any party


aggrieved by the Decision of the Regional
Officer, on any legal ground and upon payment
of the review fee may file with the Regional
Office a verified Petition for Review of such
decision within thirty (30) calendar days from
receipt thereof.
In cases decided by the Executive Committee
pursuant to Rule II, Section 2 of these Rules, as
amended, the verified Petition shall be filed with
the Executive Committee within thirty (30)
calendar days from receipt of the Committees
Decision. Copy of such petition shall be furnished
the other party and the Board of Commissioners.
No motion for reconsideration or mere notice of
petition for review of the decision shall be
entertained.
Within ten (10) calendar days from receipt of the
petition, the Regional Officer, or the Executive
Committee, as the case may be, shall elevate the
records to the Board of Commissioner together
with the summary of proceedings before the
Regional Office. The Petition for Review of a
decision rendered by the Executive Committee
shall betaken cognizance of by the Board en
banc.
RULE XVIII
Appeal from Board Decisions

39 | P a g e

JURISDICTION; FOR CASE


DIGEST

SECTION 1.
Motion for Reconsideration. Within the period
for filing an appeal from a Board decision, order
or ruling of the Board of Commissioners, any
aggrieved party may file a motion for
reconsideration with the Board only on the
following grounds: (1) serious errors of law which
would result in grave injustice if not corrected;
and (2) newly discovered evidence.
Only one (1) motion for reconsideration shall be
entertained.
Motions for reconsideration shall be assigned to
the division from which the decision, order or
ruling originated.
SECTION 2. Appeal. Any party may upon notice
to the Board and the other party appeal a
decision rendered by the Board of
Commissioners en banc or by one of its divisions
to the Office of the President within fifteen (15)
calendar days from receipt thereof, in
accordance with P.D. No. 1344 and A.O. No. 18
Series of 1987.
RULE XIX
Entry of Judgment
xxxx
SECTION 2. Rules on Finality. For purposes of
determining when a decision or order has
become final and executory for purposes of entry
in the Book of Judgment, the following shall be
observed:

a. Unless otherwise provided in a decision or


resolution rendered by the Regional Officer, the
Executive Committee, or the Board of
Commissioners, as the case may be, the orders
contained therein shall become final as regards a
party thirty (30) calendar days after the date of
receipt thereof and no petition for review or
appeal therefrom has been filed within the said
period. (Emphases supplied.)
There is no showing that respondents availed
themselves of the afore-mentioned
administrative remedies prior to instituting Civil
Case No. Br. 23-632-2000 before the RTC. While
there are accepted exceptions to the principle of
exhaustion of administrative remedies and the
doctrine of primary jurisdiction,30 respondents
never asserted nor argued any of them. Thus,
there is no cogent reason for the Court to apply
the exceptions instead of the general rule to this
case.
Ordinarily, failure to comply with the principle of
exhaustion of administrative remedies and the
doctrine of primary jurisdiction will result in the
dismissal of the case for lack of cause of action.
However, the Court herein will not go to the
extent of entirely dismissing Civil Case No. Br.
23-632-2000. The Court does not lose sight of
the fact that respondents Complaint in Civil Case
No. Br. 23-632-2000 is primarily for abatement of
nuisance; and respondents alleged the lack of
HLURB requirements for the cellular base station,
not to seek nullification of petitioners locational
clearance, but to support their chief argument
that said cellular base station is a nuisance which
needs to be abated. The issue of whether or not
the locational clearance for said cellular base
station is valid is actually separate and distinct

from the issue of whether or not the cellular base


station is a nuisance; one is not necessarily
determinative of the other. While the first is
within the primary jurisdiction of the HLURB and,
therefore, premature for the courts to rule upon
in the present case, the latter is within the
jurisdiction of the courts to determine but only
after trial proper.
On the declaration of the Court of
Appeals that petitioners cellular
base station is a nuisance that must
be abated
Article 694 of the Civil Code defines nuisance as:
ART. 694. A nuisance is any act, omission,
establishment, business, condition of property, or
anything else which:
(1) Injures or endangers the health or safety of
others; or
(2) Annoys or offends the senses; or
(3) Shocks, defies or disregards decency or
morality; or
(4) Obstructs or interferes with the free passage
of any public highway or street, or any body of
water; or
(5) Hinders or impairs the use of property.
The term "nuisance" is so comprehensive that it
has been applied to almost all ways which have
interfered with the rights of the citizens, either in
40 | P a g e

JURISDICTION; FOR CASE


DIGEST

person, property, the enjoyment of his property,


or his comfort.31
The Court, in AC Enterprises, Inc. v. Frabelle
Properties Corporation,32 settled that a simple
suit for abatement of nuisance, being incapable
of pecuniary estimation, is within the exclusive
jurisdiction of the RTC. Although respondents
also prayed for judgment for moral and
exemplary damages, attorneys fees, and
litigation expenses, such claims are merely
incidental to or as a consequence of, their
principal relief.
Nonetheless, while jurisdiction over respondents
Complaint for abatement of nuisance lies with
the courts, the respective judgments of the RTC
and the Court of Appeals cannot be upheld.
At the outset, the RTC erred in granting
petitioners Motion for Summary Judgment and
ordering the dismissal of respondents Complaint
in Civil Case No. Br. 23-632-2000.
Summary judgments are governed by Rule 35 of
the Rules of Court, pertinent provisions of which
state:
SEC. 2. Summary judgment for defending party.
A party against whom a claim, counterclaim, or
cross-claim is asserted or a declaratory relief is
sought may, at any time, move with supporting
affidavits, depositions or admissions for a
summary judgment in his favor as to all or any
part thereof.
SEC. 3. Motion and proceedings thereon. The
motion shall be served at least ten (10) days

before the time specified for the hearing. The


adverse party may serve opposing affidavits,
depositions, or admissions at least three (3) days
before the hearing. After the hearing, the
judgment sought shall be rendered forthwith if
the pleadings, supporting affidavits, depositions,
and admissions on file, show that, except as to
the amount of damages, there is no genuine
issue as to any material fact and that the moving
party is entitled to a judgment as a matter of law.
(Emphases supplied.)

pleaded appear uncontested or undisputed, then


there is no real or genuine issue or question as to
any fact and summary judgment called for. On
the other hand, where the facts pleaded by the
parties are disputed or contested, proceedings
for a summary judgment cannot take the place
of a trial. The evidence on record must be viewed
in light most favorable to the party opposing the
motion who must be given the benefit of all
favorable inferences as can reasonably be drawn
from the evidence.

In Rivera v. Solidbank Corporation,33 the Court


discussed extensively when a summary
judgment is proper:

Courts must be critical of the papers presented


by the moving party and not of the
papers/documents in opposition thereto.
Conclusory assertions are insufficient to raise an
issue of material fact. A party cannot create a
genuine dispute of material fact through mere
speculations or compilation of differences. He
may not create an issue of fact through bald
assertions, unsupported contentions and
conclusory statements. He must do more than
rely upon allegations but must come forward
with specific facts in support of a claim. Where
the factual context makes his claim implausible,
he must come forward with more persuasive
evidence demonstrating a genuine issue for trial.
(Emphases supplied; citations omitted.)

For a summary judgment to be proper, the


movant must establish two requisites: (a) there
must be no genuine issue as to any material fact,
except for the amount of damages; and (b) the
party presenting the motion for summary
judgment must be entitled to a judgment as a
matter of law. Where, on the basis of the
pleadings of a moving party, including
documents appended thereto, no genuine issue
as to a material fact exists, the burden to
produce a genuine issue shifts to the opposing
party. If the opposing party fails, the moving
party is entitled to a summary judgment.
A genuine issue is an issue of fact which requires
the presentation of evidence as distinguished
from an issue which is a sham, fictitious,
contrived or a false claim.

Judging by the aforequoted standards, summary


judgment cannot be rendered in this case as
there are clearly factual issues disputed or
contested by the parties. As respondents
correctly argued in their Opposition to
petitioners Motion for Summary Judgment:

The trial court can determine a genuine issue on


the basis of the pleadings, admissions,
documents, affidavits or counter affidavits
submitted by the parties. When the facts as

1. Contrary to the claim of petitioner, there are


several genuine issues as to the cause of action
and material facts related to the complaint. For
41 | P a g e

JURISDICTION; FOR CASE


DIGEST

one there is an issue on the structural integrity of


the tower, the ultra high frequency (UHF) radio
wave emission radiated by the communications
tower affecting the life, health and well being of
the[respondents] and the barangay residents,
especially their children. Also, the
noxious/deleterious fumes and the noise
produce[d] by the standby generator and the
danger posted by the tower if it collapses in
regard to life and limb as well as the property of
the [respondents] particularly those whose
houses abut, or are near/within the periphery of
the communications tower. x x x34
Likewise constituting real or genuine issues for
trial, which arose from subsequent events, are
the following: whether the generator subject of
respondents Complaint had been removed;
whether said generator had been replaced by
another that produces as much or even more
noise and fumes; and whether the generator is a
nuisance that can be abated separately from the
rest of the cellular base station.
Furthermore, the Court demonstrated in AC
Enterprises, Inc. the extensive factual
considerations of a court before it can arrive at a
judgment in an action for abatement of nuisance:
Whether or not noise emanating from a blower of
the air conditioning units of the Feliza Building is
nuisance is to be resolved only by the court in
due course of proceedings.1wphi1 The plaintiff
must prove that the noise is a nuisance and the
consequences thereof. Noise is not a nuisance
per se. It may be of such a character as to
constitute a nuisance, even though it arises from
the operation of a lawful business, only if it
affects injuriously the health or comfort of

ordinary people in the vicinity to an


unreasonable extent. Injury to a particular
person in a peculiar position or of especially
sensitive characteristics will not render the noise
an actionable nuisance. In the conditions of
present living, noise seems inseparable from the
conduct of many necessary occupations. Its
presence is a nuisance in the popular sense in
which that word is used, but in the absence of
statute, noise becomes actionable only when it
passes the limits of reasonable adjustment to the
conditions of the locality and of the needs of the
maker to the needs of the listener. What those
limits are cannot be fixed by any definite
measure of quantity or quality; they depend
upon the circumstances of the particular case.
They may be affected, but are not controlled, by
zoning ordinances. The delimitation of
designated areas to use for manufacturing,
industry or general business is not a license to
emit every noise profitably attending the conduct
of any one of them.
The test is whether rights of property, of health
or of comfort are so injuriously affected by the
noise in question that the sufferer is subjected to
a loss which goes beyond the reasonable limit
imposed upon him by the condition of living, or
of holding property, in a particular locality in fact
devoted to uses which involve the emission of
noise although ordinary care is taken to confine it
within reasonable bounds; or in the vicinity of
property of another owner who, though creating
a noise, is acting with reasonable regard for the
rights of those affected by it.
Commercial and industrial activities which are
lawful in themselves may become nuisances if
they are so offensive to the senses that they

render the enjoyment of life and property


uncomfortable. The fact that the cause of the
complaint must be substantial has often led to
expressions in the opinions that to be a nuisance
the noise must be deafening or loud or excessive
and unreasonable. The determining factor when
noise alone is the cause of complaint is not its
intensity or volume. It is that the noise is of such
character as to produce actual physical
discomfort and annoyance to a person of
ordinary sensibilities, rendering adjacent
property less comfortable and valuable. If the
noise does that it can well be said to be
substantial and unreasonable in degree, and
reasonableness is a question of fact dependent
upon all the circumstances and conditions. There
can be no fixed standard as to what kind of noise
constitutes a nuisance.
The courts have made it clear that in every case
the question is one of reasonableness. What is a
reasonable use of ones property and whether a
particular use is an unreasonable invasion of
anothers use and enjoyment of his property so
as to constitute a nuisance cannot be
determined by exact rules, but must necessarily
depend upon the circumstances of each case,
such as locality and the character of the
surroundings, the nature, utility and social value
of the use, the extent and nature of the harm
involved, the nature, utility and social value of
the use or enjoyment invaded, and the like.
Persons who live or work in thickly populated
business districts must necessarily endure the
usual annoyances and of those trades and
businesses which are properly located and
carried on in the neighborhood where they live or
work. But these annoyances and discomforts
42 | P a g e

JURISDICTION; FOR CASE


DIGEST

must not be more than those ordinarily to be


expected in the community or district, and which
are incident to the lawful conduct of such trades
and businesses. If they exceed what might be
reasonably expected and cause unnecessary
harm, then the court will grant relief.

For the same reasons cited above, without


presentation by the parties of evidence on the
contested or disputed facts, there was no factual
basis for declaring petitioner's cellular base
station a nuisance and ordering petitioner to
cease and desist from operating the same.

A finding by the LGU that the noise quality


standards under the law have not been complied
with is not a prerequisite nor constitutes
indispensable evidence to prove that the
defendant is or is not liable for a nuisance and
for damages. Such finding is merely
corroborative to the testimonial and/or other
evidence to be presented by the parties. The
exercise of due care by the owner of a business
in its operation does not constitute a defense
where, notwithstanding the same, the business
as conducted, seriously affects the rights of
those in its vicinity.35(Citations omitted.)

Given the equally important interests of the


parties in this case, i.e., on one hand,
respondents' health, safety, and property, and on
the other, petitioner's business interest and the
public's need for accessible and better cellular
mobile telephone services, the wise and prudent
course to take is to remand the case to the RTC
for trial and give the parties the opportunity to
prove their respective factual claims.

A reading of the RTC Order dated January 16,


2001 readily shows that the trial court did not
take into account any of the foregoing
considerations or tests before summarily
dismissing Civil Case No. Br. 23-632-2000. The
reasoning of the RTC that similar cellular base
stations are scattered in heavily populated areas
nationwide and are not declared nuisances is
unacceptable. As to whether or not this specific
cellular base station of petitioner is a nuisance to
respondents is largely dependent on the
particular factual circumstances involved in the
instant case, which is exactly why a trial for
threshing out disputed or contested factual
issues is indispensable. Evidently, it was the RTC
which engaged in speculations and
unsubstantiated conclusions.

WHEREFORE, premises considered, the instant


Petition is PARTIALLY GRANTED. The Decision
dated July 16, 2004 and Resolution dated
December 9, 2004 of the Court of Appeals in CAG.R. CV No. 71337 are REVERSED and SET ASIDE.
Let the records of the case be REMANDED to the
Regional Trial Court, Branch 23, of Roxas, Isabela,
which is DIRECTED to reinstate Civil Case No. Br.
23-632-2000 to its docket and proceed with the
trial and adjudication thereof with appropriate
dispatch in accordance with this Decision.
SO ORDERED.
SAN MIGUEL VS PEREZ
FACTS: S The pendency of an administrative
case for specific performance brought by the
buyer of residential subdivision lots in the
Housing and Land Use Regulatory Board (HLURB)
to compel the seller to deliver the transfer
certificates of title (TCTs) of the fully paid lots is

properly considered a ground to suspend a


criminal prosecution for violation of Section 25 of
Presidential Decree No. 9571 on the ground of a
prejudicial question. The administrative
determination is a logical antecedent of the
resolution of the criminal charges based on nondelivery of the TCTs.
Antecedents
Petitioner San Miguel Properties Inc. (San Miguel
Properties), a domestic corporation engaged in
the real estate business, purchased in 1992,
1993 and April 1993 from B.F. Homes, Inc. (BF
Homes), then represented by Atty. Florencio B.
Orendain (Orendain) as its duly authorized
rehabilitation receiver appointed by the
Securities and Exchange Commission (SEC),2 130
residential lots situated in its subdivision BF
Homes Paraaque, containing a total area of
44,345 square meters for the aggregate price
of P106,248,000.00. The transactions were
embodied in three separate deeds of sale.3 The
TCTs covering the lots bought under the first and
second deeds were fully delivered to San Miguel
Properties, but 20 TCTs covering 20 of the 41
parcels of land with a total area of 15,565 square
meters purchased under the third deed of sale,
executed in April 1993 and for which San Miguel
Properties paid the full price of P39,122,627.00,
were not delivered to San Miguel Properties.
On its part, BF Homes claimed that it withheld
the delivery of the 20 TCTs for parcels of land
purchased under the third deed of sale because
Atty. Orendain had ceased to be its rehabilitation
receiver at the time of the transactions after
being meanwhile replaced as receiver by FBO
43 | P a g e

JURISDICTION; FOR CASE


DIGEST

Network Management, Inc. on May 17, 1989


pursuant to an order from the SEC.4
BF Homes refused to deliver the 20 TCTs despite
demands. Thus, on August 15, 2000, San Miguel
Properties filed a complaint-affidavit in the Office
of the City Prosecutor of Las Pias City (OCP Las
Pias) charging respondent directors and officers
of BF Homes with non-delivery of titles in
violation of Section 25, in relation to Section 39,
both of Presidential Decree No. 957 (I.S. No. 002256).5
At the same time, San Miguel Properties sued BF
Homes for specific performance in the HLURB
(HLURB Case No. REM-082400-11183),6 praying
to compel BF Homes to release the 20 TCTs in its
favor.
In their joint counter-affidavit submitted in I.S.
No. 00-2256,7 respondent directors and officers
of BF Homes refuted San Miguel Properties
assertions by contending that: (a) San Miguel
Properties claim was not legally demandable
because Atty. Orendain did not have the
authority to sell the 130 lots in 1992 and 1993
due to his having been replaced as BF Homes
rehabilitation receiver by the SEC on May 17,
1989; (b) the deeds of sale conveying the lots
were irregular for being undated and
unnotarized; (c) the claim should have been
brought to the SEC because BF Homes was under
receivership; (d) in receivership cases, it was
essential to suspend all claims against a
distressed corporation in order to enable the
receiver to effectively exercise its powers free
from judicial and extra-judicial interference that
could unduly hinder the rescue of the distressed
company; and (e) the lots involved were under

custodia legis in view of the pending receivership


proceedings, necessarily stripping the OCP Las
Pias of the jurisdiction to proceed in the action.
On October 10, 2000, San Miguel Properties filed
a motion to suspend proceedings in the OCP Las
Pias,8 citing the pendency of BF Homes
receivership case in the SEC. In its
comment/opposition, BF Homes opposed the
motion to suspend. In the meantime, however,
the SEC terminated BF Homes receivership on
September 12, 2000, prompting San Miguel
Properties to file on October 27, 2000 a reply to
BF Homes comment/opposition coupled with a
motion to withdraw the sought suspension of
proceedings due to the intervening termination
of the receivership.9
On October 23, 2000, the OCP Las Pias
rendered its resolution,10 dismissing San Miguel
Properties criminal complaint for violation of
Presidential Decree No. 957 on the ground that
no action could be filed by or against a receiver
without leave from the SEC that had appointed
him; that the implementation of the provisions of
Presidential Decree No. 957 exclusively pertained
under the jurisdiction of the HLURB; that there
existed a prejudicial question necessitating the
suspension of the criminal action until after the
issue on the liability of the distressed BF Homes
was first determined by the SEC en banc or by
the HLURB; and that no prior resort to
administrative jurisdiction had been made; that
there appeared to be no probable cause to indict
respondents for not being the actual signatories
in the three deeds of sale.
On February 20, 2001, the OCP Las Pias denied
San Miguel Properties motion for reconsideration

filed on November 28, 2000, holding that BF


Homes directors and officers could not be held
liable for the non-delivery of the TCTs under
Presidential Decree No. 957 without a definite
ruling on the legality of Atty. Orendains actions;
and that the criminal liability would attach only
after BF Homes did not comply with a directive of
the HLURB directing it to deliver the titles.11
San Miguel Properties appealed the resolutions of
the OCP Las Pias to the Department of Justice
(DOJ), but the DOJ Secretary denied the appeal
on October 15, 2001, holding:
After a careful review of the evidence on record,
we find no cogent reason to disturb the ruling of
the City Prosecutor of Las Pias City. Established
jurisprudence supports the position taken by the
City Prosecutor concerned.
There is no dispute that aside from the instant
complaint for violation of PD 957, there is still
pending with the Housing and Land Use
Resulatory Board (HLURB, for short) a complaint
for specific performance where the HLURB is
called upon to inquire into, and rule on, the
validity of the sales transactions involving the
lots in question and entered into by Atty.
Orendain for and in behalf of BF Homes.
As early as in the case of Solid Homes, Inc. vs.
Payawal, 177 SCRA 72, the Supreme Court had
ruled that the HLURB has exclusive jurisdiction
over cases involving real estate business and
practices under PD 957. This is reiterated in the
subsequent cases of Union Bank of the
Philippines versus HLURB, G.R. [No.] 953364,
June 29, 1992 and C.T. Torres Enterprises vs.
Hilionada, 191 SCRA 286.
44 | P a g e

JURISDICTION; FOR CASE


DIGEST

The said ruling simply means that unless and


until the HLURB rules on the validity of the
transactions involving the lands in question with
specific reference to the capacity of Atty.
Orendain to bind BF Homes in the said
transactions, there is as yet no basis to charge
criminally respondents for non-delivery of the
subject land titles. In other words, complainant
cannot invoke the penal provision of PD 957 until
such time that the HLURB shall have ruled and
decided on the validity of the transactions
involving the lots in question.
WHEREFORE, the appeal is hereby DENIED.
Ruling of the CA
Undaunted, San Miguel Properties elevated the
DOJs resolutions to the CA on certiorari and
mandamus (C.A.-G.R. SP No. 73008), contending
that respondent DOJ Secretary had acted with
grave abuse in denying their appeal and in
refusing to charge the directors and officers of BF
Homes with the violation of Presidential Decree
No. 957. San Miguel Properties submitted the
issue of whether or not HLURB Case No. REM082400-11183 presented a prejudicial question
that called for the suspension of the criminal
action for violation of Presidential Decree No.
957.

generally applies to civil and criminal actions


only.
However, an exception to this rule is provided in
Quiambao vs. Osorio cited by the respondents. In
this case, an issue in an administrative case was
considered a prejudicial question to the
resolution of a civil case which, consequently,
warranted the suspension of the latter until after
termination of the administrative proceedings.
Quiambao vs. Osorio is not the only instance
when the Supreme Court relaxed the application
of the rule on prejudicial question.
In Tamin vs. CA involving two (2) civil actions, the
Highest Court similarly applied the rule on
prejudicial question when it directed petitioner
therein to put up a bond for just compensation
should the demolition of private respondents
building proved to be illegal as a result of a
pending cadastral suit in another tribunal.
City of Pasig vs. COMELEC is yet another
exception where a civil action involving a
boundary dispute was considered a prejudicial
question which must be resolved prior to an
administrative proceeding for the holding of a
plebiscite on the affected areas.

In its assailed decision promulgated on February


24, 2004 in C.A.-G.R. SP No. 73008, 14 the CA
dismissed San Miguel Properties petition,
holding and ruling as follows:

In fact, in Vidad vs. RTC of Negros Oriental, Br.


42, it was ruled that in the interest of good order,
courts can suspend action in one case pending
determination of another case closely
interrelated or interlinked with it.

From the foregoing, the conclusion that may be


drawn is that the rule on prejudicial question

It thus appears that public respondent did not act


with grave abuse of discretion x x x when he

applied the rule on prejudicial question to the


instant proceedings considering that the issue on
the validity of the sale transactions x x x by x x x
Orendain in behalf of BF Homes, Inc., is closely
intertwined with the purported criminal
culpability of private respondents, as
officers/directors of BF Homes, Inc., arising from
their failure to deliver the titles of the parcels of
land included in the questioned conveyance.
All told, to sustain the petitioners theory that the
result of the HLURB proceedings is not
determinative of the criminal liability of private
respondents under PD 957 would be to espouse
an absurdity. If we were to assume that the
HLURB finds BFHI under no obligation to delve
the subject titles, it would be highly irregular and
contrary to the ends of justice to pursue a
criminal case against private respondents for the
non-delivery of certificates of title which they are
not under any legal obligation to turn over in the
first place. (Bold emphasis supplied)
On a final note, absent grave abuse of discretion
on the part of the prosecutorial arm of the
government as represented by herein public
respondent, courts will not interfere with the
discretion of a public prosecutor in prosecuting or
dismissing a complaint filed before him. A public
prosecutor, by the nature of his office, is under
no compulsion to file a criminal information
where no clear legal justification has been
shown, and no sufficient evidence of guilt nor
prima facie case has been established by the
complaining party.
WHEREFORE, premises considered, the instant
Petition for Certiorari and Mandamus is hereby
DENIED. The Resolutions dated 15 October 2001
45 | P a g e

JURISDICTION; FOR CASE


DIGEST

and 12 July 2002 of the Department of Justice are


AFFIRMED.

outcome of the administrative proceeding in the


interest of good order.18

SO ORDERED.

Not content with the outcome, San Miguel


Properties appealed to the Office of the President
(OP), arguing that the HLURB erred in suspending
the proceedings. On January 27, 2004, the OP
reversed the HLURB Boards ruling, holding
thusly:

15

The CA denied San Miguel Properties motion for


reconsideration on January 18, 2005.1
ISSUE: THE COURT OF APPEALS
COMMITTED GRAVE, SERIOUS AND
REVERSIBLE ERRORS WHEN IT DISMISSED
PETITIONERS CERTIORARI AND MANDAMUS
PETITION TO ORDER AND DIRECT
RESPONDENT SECRETARY TO INDICT
RESPONDENTS FOR VIOLATION OF SECTION
25, PD. 957 .
RULING: S It is relevant at this juncture to
mention the outcome of the action for specific
performance and damages that San Miguel
Properties instituted in the HLURB simultaneously
with its filing of the complaint for violation of
Presidential Decree No. 957. On January 25,
2002, the HLURB Arbiter ruled that the HLURB
was inclined to suspend the proceedings until the
SEC resolved the issue of Atty. Orendains
authority to enter into the transactions in BF
Homes behalf, because the final resolution by
the SEC was a logical antecedent to the
determination of the issue involved in the
complaint before the HLURB. Upon appeal, the
HLURB Board of Commissioners (HLURB Board),
citing the doctrine of primary jurisdiction,
affirmed the HLURB Arbiters decision, holding
that although no prejudicial question could arise,
strictly speaking, if one case was civil and the
other administrative, it nonetheless opted to
suspend its action on the cases pending the final

The basic complaint in this case is one for


specific performance under Section 25 of the
Presidential Decree (PD) 957 "The Subdivision
and Condominium Buyers Protective."
As early as August 1987, the Supreme Court
already recognized the authority of the HLURB,
as successor agency of the National Housing
Authority (NHA), to regulate, pursuant to PD 957,
in relation to PD 1344, the real estate trade, with
exclusive original jurisdiction to hear and decide
cases "involving specific performance of
contractual and statutory obligation filed by
buyers of subdivision lots against the owner,
developer, dealer, broker or salesman," the
HLURB, in the exercise of its adjudicatory powers
and functions, "must interpret and apply
contracts, determine the rights of the parties
under these contracts and award[s] damages
whenever appropriate."
Given its clear statutory mandate, the HLURBs
decision to await for some forum to decide if
ever one is forthcoming the issue on the
authority of Orendain to dispose of subject lots
before it peremptorily resolves the basic
complaint is unwarranted, the issues thereon
having been joined and the respective position
papers and the evidence of the parties having

been submitted. To us, it behooved the HLURB to


adjudicate, with the usual dispatch, the right and
obligation of the parties in line with its own
appreciation of the obtaining facts and applicable
law. To borrow from Mabubha Textile Mills
Corporation vs. Ongpin, it does not have to rely
on the finding of others to discharge this
adjudicatory functions.19
After its motion for reconsideration was denied,
BF Homes appealed to the CA (C.A.-G.R. SP No.
83631), raising as issues: (a) whether or not the
HLURB had the jurisdiction to decide with finality
the question of Atty. Orendains authority to
enter into the transaction with San Miguel
Properties in BF Homes behalf, and rule on the
rights and obligations of the parties to the
contract; and (b) whether or not the HLURB
properly suspended the proceedings until the
SEC resolved with finality the matter regarding
such authority of Atty. Orendain.
The CA promulgated its decision in C.A.-G.R. SP
No. 83631,20 decreeing that the HLURB, not the
SEC, had jurisdiction over San Miguel Properties
complaint. It affirmed the OPs decision and
ordered the remand of the case to the HLURB for
further proceedings on the ground that the case
involved matters within the HLURBs competence
and expertise pursuant to the doctrine of primary
jurisdiction, viz:
[T]he High Court has consistently ruled that the
NHA or the HLURB has jurisdiction over
complaints arising from contracts between the
subdivision developer and the lot buyer or those
aimed at compelling the subdivision developer to
comply with its contractual and statutory
obligations.
46 | P a g e

JURISDICTION; FOR CASE


DIGEST

Hence, the HLURB should take jurisdiction over


respondents complaint because it pertains to
matters within the HLURBs competence and
expertise. The proceedings before the HLURB
should not be suspended.
While We sustain the Office of the President, the
case must be remanded to the HLURB. This is in
recognition of the doctrine of primary
jurisdiction. The fairest and most equitable
course to take under the circumstances is to
remand the case to the HLURB for the proper
presentation of evidence.21
Did the Secretary of Justice commit grave abuse
of discretion in upholding the dismissal of San
Miguel Properties criminal complaint for violation
of Presidential Decree No. 957 for lack of
probable cause and for reason of a prejudicial
question?
The question boils down to whether the HLURB
administrative case brought to compel the
delivery of the TCTs could be a reason to suspend
the proceedings on the criminal complaint for the
violation of Section 25 of Presidential Decree No.
957 on the ground of a prejudicial question.
Ruling of the Court
The petition has no merit.
1.
Action for specific performance, even if pending
in the HLURB, an administrative agency, raises a
prejudicial question BF Homes posture that the
administrative case for specific performance in

the HLURB posed a prejudicial question that must


first be determined before the criminal case for
violation of Section 25 of Presidential Decree No.
957 could be resolved is correct.

civil in nature but could not be instituted


elsewhere except in the HLURB, whose
jurisdiction over the action was exclusive and
original.25

A prejudicial question is understood in law to be


that which arises in a case the resolution of
which is a logical antecedent of the issue
involved in the criminal case, and the cognizance
of which pertains to another tribunal. It is
determinative of the criminal case, but the
jurisdiction to try and resolve it is lodged in
another court or tribunal. It is based on a fact
distinct and separate from the crime but is so
intimately connected with the crime that it
determines the guilt or innocence of the
accused.22 The rationale behind the principle of
prejudicial question is to avoid conflicting
decisions.23 The essential elements of a
prejudicial question are provided in Section 7,
Rule 111 of the Rules of Court, to wit: (a) the
previously instituted civil action involves an issue
similar or intimately related to the issue raised in
the subsequent criminal action, and (b) the
resolution of such issue determines whether or
not the criminal action may proceed.

The determination of whether the proceedings


ought to be suspended because of a prejudicial
question rested on whether the facts and issues
raised in the pleadings in the specific
performance case were so related with the issues
raised in the criminal complaint for the violation
of Presidential Decree No. 957, such that the
resolution of the issues in the former would be
determinative of the question of guilt in the
criminal case. An examination of the nature of
the two cases involved is thus necessary.

The concept of a prejudicial question involves a


civil action and a criminal case. Yet, contrary to
San Miguel Properties submission that there
could be no prejudicial question to speak of
because no civil action where the prejudicial
question arose was pending, the action for
specific performance in the HLURB raises a
prejudicial question that sufficed to suspend the
proceedings determining the charge for the
criminal violation of Section 2524 of Presidential
Decree No. 957. This is true simply because the
action for specific performance was an action

Article 1191. The power to rescind obligations is


implied in reciprocal ones, in case one of the
obligors should not comply with what is
incumbent upon him.

An action for specific performance is the remedy


to demand the exact performance of a contract
in the specific form in which it was made, or
according to the precise terms agreed upon by a
party bound to fulfill it.26 Evidently, before the
remedy of specific performance is availed of,
there must first be a breach of the
contract.27 The remedy has its roots in Article
1191 of the Civil Code, which reads:

The injured party may choose between the


fulfillment and the rescission of the obligation,
with the payment of damages in either case. He
may also seek rescission, even after he has
chosen fulfillment, if the latter should become
impossible. x x x (Emphasis supplied)
47 | P a g e

JURISDICTION; FOR CASE


DIGEST

Accordingly, the injured party may choose


between specific performance or rescission with
damages. As presently worded, Article 1191
speaks of the remedy of rescission in reciprocal
obligations within the context of Article 1124 of
the former Civil Code which used the term
resolution. The remedy of resolution applied only
to reciprocal obligations, such that a partys
breach of the contract equated to a tacit
resolutory condition that entitled the injured
party to rescission. The present article, as in the
former one, contemplates alternative remedies
for the injured party who is granted the option to
pursue, as principal actions, either the rescission
or the specific performance of the obligation,
with payment of damages in either case. 28
On the other hand, Presidential Decree No. 957 is
a law that regulates the sale of subdivision lots
and condominiums in view of the increasing
number of incidents wherein "real estate
subdivision owners, developers, operators,
and/or sellers have reneged on their
representations and obligations to provide and
maintain properly" the basic requirements and
amenities, as well as of reports of alarming
magnitude of swindling and fraudulent
manipulations perpetrated by unscrupulous
subdivision and condominium sellers and
operators,29 such as failure to deliver titles to the
buyers or titles free from liens and
encumbrances. Presidential Decree No. 957
authorizes the suspension and revocation of the
registration and license of the real estate
subdivision owners, developers, operators,
and/or sellers in certain instances, as well as
provides the procedure to be observed in such
instances; it prescribes administrative fines and

other penalties in case of violation of, or noncompliance with its provisions.

2.
Doctrine of primary jurisdiction is applicable

Conformably with the foregoing, the action for


specific performance in the HLURB would
determine whether or not San Miguel Properties
was legally entitled to demand the delivery of
the remaining 20 TCTs, while the criminal action
would decide whether or not BF Homes directors
and officers were criminally liable for withholding
the 20 TCTs. The resolution of the former must
obviously precede that of the latter, for should
the HLURB hold San Miguel Properties to be not
entitled to the delivery of the 20 TCTs because
Atty. Orendain did not have the authority to
represent BF Homes in the sale due to his
receivership having been terminated by the SEC,
the basis for the criminal liability for the violation
of Section 25 of Presidential Decree No. 957
would evaporate, thereby negating the need to
proceed with the criminal case.
Worthy to note at this juncture is that a
prejudicial question need not conclusively
resolve the guilt or innocence of the accused. It
is enough for the prejudicial question to simply
test the sufficiency of the allegations in the
information in order to sustain the further
prosecution of the criminal case. A party who
raises a prejudicial question is deemed to have
hypothetically admitted that all the essential
elements of the crime have been adequately
alleged in the information, considering that the
Prosecution has not yet presented a single piece
of evidence on the indictment or may not have
rested its case. A challenge to the allegations in
the information on the ground of prejudicial
question is in effect a question on the merits of
the criminal charge through a non-criminal suit.30

That the action for specific performance was an


administrative case pending in the HLURB,
instead of in a court of law, was of no
consequence at all. As earlier mentioned, the
action for specific performance, although civil in
nature, could be brought only in the HLURB. This
situation conforms to the doctrine of primary
jurisdiction. There has been of late a proliferation
of administrative agencies, mostly regulatory in
function. It is in favor of these agencies that the
doctrine of primary jurisdiction is frequently
invoked, not to defeat the resort to the judicial
adjudication of controversies but to rely on the
expertise, specialized skills, and knowledge of
such agencies in their resolution. The Court has
observed that one thrust of the proliferation is
that the interpretation of contracts and the
determination of private rights under contracts
are no longer a uniquely judicial function
exercisable only by the regular courts.31
The doctrine of primary jurisdiction has been
increasingly called into play on matters
demanding the special competence of
administrative agencies even if such matters are
at the same time within the jurisdiction of the
courts. A case that requires for its determination
the expertise, specialized skills, and knowledge
of some administrative board or commission
because it involves technical matters or intricate
questions of fact, relief must first be obtained in
an appropriate administrative proceeding before
a remedy will be supplied by the courts although
the matter comes within the jurisdiction of the
courts. The application of the doctrine does not
48 | P a g e

JURISDICTION; FOR CASE


DIGEST

call for the dismissal of the case in the court but


only for its suspension until after the matters
within the competence of the administrative
body are threshed out and determined.32
To accord with the doctrine of primary
jurisdiction, the courts cannot and will not
determine a controversy involving a question
within the competence of an administrative
tribunal, the controversy having been so placed
within the special competence of the
administrative tribunal under a regulatory
scheme. In that instance, the judicial process is
suspended pending referral to the administrative
body for its view on the matter in dispute.
Consequently, if the courts cannot resolve a
question that is within the legal competence of
an administrative body prior to the resolution of
that question by the latter, especially where the
question demands the exercise of sound
administrative discretion requiring the special
knowledge, experience, and services of the
administrative agency to ascertain technical and
intricate matters of fact, and a uniformity of
ruling is essential to comply with the purposes of
the regulatory statute administered, suspension
or dismissal of the action is proper.33
3.

an act or omission was malum prohibitum did not


do away with the initiative inherent in every
court to avoid an absurd result by means of
rendering a reasonable interpretation and
application of the procedural law. Indeed, the
procedural law must always be given a
reasonable construction to preclude absurdity in
its application.35 Hence, a literal application of
the principle governing prejudicial questions is to
be eschewed if such application would produce
unjust and absurd results or unreasonable
consequences.
San Miguel Properties further submits that
respondents could not validly raise the
prejudicial question as a reason to suspend the
criminal proceedings because respondents had
not themselves initiated either the action for
specific performance or the criminal
action.1wphi1 It contends that the defense of a
prejudicial question arising from the filing of a
related case could only be raised by the party
who filed or initiated said related case.
The submission is unfounded. The rule on
prejudicial question makes no distinction as to
who is allowed to raise the defense. Ubi lex non
distinguit nec nos distinguere debemos. When
the law makes no distinction, we ought not to
distinguish.36

FACTS: s This is a petition for review


on certiorari under Rule 45 of the 1997 Rules of
Civil Procedure of the Decision[1] dated May 16,
2006 as well as the Resolution[2] dated October 5,
2006 of the Court of Appeals in CA-G.R. CV No.
63439, entitled ADDITION HILLS MANDALUYONG
CIVIC & SOCIAL ORGANIZATION INC. vs.
MEGAWORLD PROPERTIES & HOLDINGS, INC.,
WILFREDO I. IMPERIAL in his capacity as Director,
NCR, and HOUSING AND LAND USE REGULATORY
BOARD, DEPARTMENT OF ENVIRONMENT AND
NATURAL RESOURCES. In effect, the appellate
courts issuances reversed and set aside the
Decision[3] dated September 10, 1998 rendered
by the Regional Trial Court (RTC) of Pasig City,
Branch 158 in Civil Case No. 65171.

The facts of this case, as narrated in the assailed


May 16, 2006 Decision of the Court of Appeals,
are as follows:
[Private respondent] MEGAWORLD was the
registered owner of a parcel of land located
along Lee Street, Barangay Addition Hills,
Mandaluyong City with an area of 6,148 square
meters, more or less, covered by Transfer
Certificate of Title (TCT) No. 12768, issued by the
Register of Deeds for Mandaluyong City.

Other submissions of petitioner are unwarranted


It is not tenable for San Miguel Properties to
argue that the character of a violation of Section
25 of Presidential Decree No. 957 as malum
prohibitum, by which criminal liability attached to
BF Homes directors and officers by the mere
failure to deliver the TCTs, already rendered the
suspension unsustainable.34 The mere fact that

WHEREFORE, the Court AFFIRMS the decision


promulgated on February 24, 2004 by the Court
of Appeals in CA-G.R. SP NO. 73008; and ORDERS
petitioner to pay the costs of suit.
SO ORDERED.

Sometime in 1994, [private respondent]


MEGAWORLD conceptualized the construction of
a residential condominium complex on the said
parcel of land called the Wack-Wack Heights
Condominiumconsisting of a cluster of six (6)

ADDITION HILLS VS MEGA WORLD


49 | P a g e

JURISDICTION; FOR CASE


DIGEST

four-storey buildings and one (1) seventeen (17)


storey tower.
On July 24, 1994, the RTC denied the
motion to dismiss filed by [private respondent]
MEGAWORLD.
[Private respondent] MEGAWORLD
thereafter secured the necessary clearances,
licenses and permits for the condominium
project, including: (1) a CLV, issued on October
25, 1994, and a Development Permit, issued on
November 11, 1994, both by the [public
respondent] HLURB; (2) an ECC, issued on March
15, 1995, by the Department of Environment and
Natural Resources (DENR); (3) a Building Permit,
issued on February 3, 1995, by the Office of the
Building Official of Mandaluyong City; and (4) a
Barangay Clearance dated September 29, 1994,
from the office of the Barangay Chairman of
Addition Hills.
Thereafter, construction of the
condominium project began, but on June 30,
1995, the plaintiff-appellee AHMCSO filed a
complaint before the Regional Trial Court of Pasig
City, Branch 158, docketed as Civil Case No.
65171, for yo (sic) annul the Building Permit,
CLV, ECC and Development Permit granted to
MEGAWORLD; to prohibit the issuance to
MEGAWORLD of Certificate of Registration and
License to Sell Condominium Units; and to
permanently enjoin local and national building
officials from issuing licenses and permits to
MEGAWORLD.
On July 20, 1995, [private respondent]
MEGAWORLD filed a Motion to Dismiss the case
for lack of cause of action and that jurisdiction
over the case was with the [public respondent]
HLURB and not with the regular courts.

On August 3, 1995, [private respondent]


MEGAWORLD filed its Answer.
On November 15, 1995, pre-trial was
commenced.
Thereafter, trial on the merits ensued.[4]
The trial court rendered a Decision dated
September 10, 1998 in favor of petitioner, the
dispositive portion of which reads:
WHEREFORE, in view of the foregoing, the
Certificate of Locational Viability, the
Development Permit and the Certificate of
Registration and License to Sell Condominium
Units, all issued by defendant Wilfredo I.
Imperial, National Capital Region Director of the
Housing and Land Use Regulatory Boad (HLURBNCR) are all declared void and of no effect. The
same goes for the Building Permit issued by
defendant Francisco Mapalo of Mandaluyong City.
In turn, defendant Megaworld Properties and
Holdings Inc. is directed to rectify its Wack Wack
Heights Project for it to conform to the
requirements of an R-2 zone of Mandaluyong City
and of the Metro Manila Zoning Ordinance 81-01.

Costs against these defendants.[5]

Private respondent appealed to the Court of


Appeals which issued the assailed May 16, 2006
Decision which reversed and set aside the
aforementioned trial court ruling, the dispositive
portion of which reads:
WHEREFORE, premises considered, the
September 10, 1998 Decision of the Regional
Trial Court of Pasig City, Branch 158, rendered in
Civil Case No. 65171 is hereby REVERSED and
SET ASIDE and a new one entered DISMISSING
the complaint.[6]
As can be expected, petitioner moved for
reconsideration; however, the Court of Appeals
denied the motion in its assailed October 5, 2006
Resolution.
ISSUE: WHETHER OR NOT THE COURT OF
APPEALS ERRED WHEN IT FOUND THAT
PETITIONER FAILED TO EXHAUST
ADMINISTRATIVE REMEDIES BEFORE
SEEKING JUDICIAL INTERVENTION FROM
THE COURTS.
RULING: S We find the petition to be without
merit.
At the outset, the parties in their various
pleadings discuss issues, although ostensibly
legal, actually require the Court to make findings
of fact. It is long settled, by law and
jurisprudence, that the Court is not a trier of
facts.[10] Therefore, the only relevant issue to be
resolved in this case is whether or not the
remedy sought by the petitioner in the trial court
50 | P a g e

JURISDICTION; FOR CASE


DIGEST

is in violation of the legal principle of the


exhaustion of administrative remedies.
We have consistently declared that the doctrine
of exhaustion of administrative remedies is a
cornerstone of our judicial system. The thrust of
the rule is that courts must allow administrative
agencies to carry out their functions and
discharge their responsibilities within the
specialized areas of their respective
competence. The rationale for this doctrine is
obvious. It entails lesser expenses and provides
for the speedier resolution of
controversies. Comity and convenience also
impel courts of justice to shy away from a
dispute until the system of administrative
redress has been completed.[11]
In the case of Republic v. Lacap,[12] we
expounded on the doctrine of exhaustion of
administrative remedies and the related doctrine
of primary jurisdiction in this wise:
The general rule is that before a party may seek
the intervention of the court, he should first avail
of all the means afforded him by administrative
processes. The issues which administrative
agencies are authorized to decide should not be
summarily taken from them and submitted to a
court without first giving such administrative
agency the opportunity to dispose of the same
after due deliberation.
Corollary to the doctrine of exhaustion of
administrative remedies is the doctrine of
primary jurisdiction; that is, courts cannot or will
not determine a controversy involving a question
which is within the jurisdiction of the
administrative tribunal prior to the resolution of

that question by the administrative tribunal,


where the question demands the exercise of
sound administrative discretion requiring the
special knowledge, experience and services of
the administrative tribunal to determine
technical and intricate matters of fact.[13]
It is true that the foregoing doctrine admits of
exceptions, such that in Lacap, we also held:
Nonetheless, the doctrine of exhaustion of
administrative remedies and the corollary
doctrine of primary jurisdiction, which are based
on sound public policy and practical
considerations, are not inflexible rules. There are
many accepted exceptions, such as: (a) where
there is estoppel on the part of the party
invoking the doctrine; (b) where the challenged
administrative act is patently illegal, amounting
to lack of jurisdiction; (c) where there is
unreasonable delay or official inaction that will
irretrievably prejudice the complainant; (d)
where the amount involved is relatively small so
as to make the rule impractical and oppressive;
(e) where the question involved is purely legal
and will ultimately have to be decided by the
courts of justice; (f) where judicial intervention is
urgent; (g) when its application may cause great
and irreparable damage; (h) where the
controverted acts violate due process; (i) when
the issue of non-exhaustion of administrative
remedies has been rendered moot; (j) when
there is no other plain, speedy and adequate
remedy; (k) when strong public interest is
involved; and, (l) in quo warranto proceedings. x
x x.[14]

aforementioned exceptions exist in the case at


bar.
What is apparent, however, is that petitioner
unjustifiably failed to exhaust the administrative
remedies available with the Housing and Land
Use Regulatory Board (HLURB) before seeking
recourse with the trial court. Under the rules of
the HLURB which were then in effect, particularly
Sections 4 and 6 of HLURB Resolution No. R-391,
Series of 1987 (Adopting the 1987 Rules of
Procedure of the Housing and Land Use
Regulatory Board),[15] a complaint to annul any
permit issued by the HLURB may be filed before
the Housing and Land Use Arbiter
(HLA). Therefore, petitioners action to annul the
Certificate of Locational Viability (CLV) and the
Development Permit issued by the HLURB on
October 25, 1994 and November 11, 1994,
respectively, in favor of private respondent for its
Wack-Wack Heights Condominium Project should
have been properly filed before the HLURB
instead of the trial court.
We quote with approval the Court of Appeals
discussion of this matter:
In the case at bar, plaintiff-appellee AHMCSO
failed to exhaust the available administrative
remedies before seeking judicial
intervention via a petition for annulment. The
power to act as appellate body over decisions
and actions of local and regional planning and
zoning bodies and deputized official of the board
was retained by the HLURB and remained
unaffected by the devolution under the Local
Government Code.

Upon careful consideration of the parties


contentions, we find that none of the
51 | P a g e

JURISDICTION; FOR CASE


DIGEST

Under Section 5 of Executive Order No. 648,


series of 1981, the Human Settlement Regulatory
Commission (HSRC) later renamed as Housing
and Land Use Regulatory Board (HLURB),
pursuant to Section 1(c) of Executive Order No.
90, series of 1986, has the power to:
f) Act as the appellate body on decisions and
actions of local and regional planning and zoning
bodies of the deputized officials of the
Commission, on matters arising from the
performance of these functions.

In fact, Section 4 of E.O. No. 71 affirms the power


of the HLURB to review actions of local
government units on the issuance of permits

Sec. 4. If in the course of evaluation of


application for registration and licensing of
projects within its jurisdiction, HLURB finds that a
local government unit has overlooked or
mistakenly applied a certain law, rule or standard
in issuing a development permit, it shall suspend
action with a corresponding advice to the local
government concerned, so as to afford it an
opportunity to take appropriate action thereon.
Such return and advice must likewise be effected
within a period of thirty (30) days from receipt by
HLURB of the application.
Moreover, Section 18 and 19 of HSRC
Administrative Order No. 20 provides:

Section 18. Oppossition to


Application. Opposition to application shall be
considered as a complaint, the resolution of
which shall be a prerequisite to any action on the
application. Complaints and other legal
processes shall be governed by the Rules of
Procedure of the Commission, and shall have the
effect of suspending the application.
Section 19. Complaints/Opposition Filed After the
Issuance of Locational Clearance. Temporary
issuance of locational permit or land transaction
approval shall be acted upon by the Office that
issued the same. Such complaint shall not
automatically suspend the locational clearance,
temporary use permit, development permit or
land transaction approval unless an order issued
by the commission to that effect.
The appropriate provisions of the Rules of
Procedure governing hearings before the
Commission shall be applied in the resolution of
said complaint as well as any motion for
reconsideration that may be filed thereto,
provided that if the complaint is directed against
the certificate of zoning compliance issued by
the deputized zoning administrator, the same
shall be acted upon the Commissioner in Charge
for adjudication.

Under the rules of the HLURB then prevailing at


the time this case was filed, a complaint to
annul any permit issued by the HLURB may
be filed before the Housing and Land Use
Arbiter (HLA). The decision of the HLA may
be brought to the Board of Commissioners
by Petition for Certiorari and the decision

of the Board of Commissioners [is]


appealable to the Office of the President.[16]
(Citations omitted; emphases supplied.)
It does not escape the attention of the Court that
in its Reply, petitioner admitted that it had a
pending complaint with the HLURB involving
private respondents the Development Permit,
the Certificate of Registration and License to Sell
Condominium Units, aside from complaints with
the Building Official of the Municipality (now City)
of Mandaluyong and the MMDA, when it
instituted its action with the trial court. As
discussed earlier, a litigant cannot go around the
authority of the concerned administrative agency
and directly seek redress from the courts. Thus,
when the law provides for a remedy against a
certain action of an administrative board, body,
or officer, relief to the courts can be made only
after exhausting all remedies provided therein. It
is settled that the non-observance of the doctrine
of exhaustion of administrative remedies results
in lack of cause of action, which is one of the
grounds in the Rules of Court justifying the
dismissal of the complaint.[17]

In view of the foregoing discussion, we find it


unnecessary to resolve the other issues raised by
the parties.
To conclude, it is our view that the Court of
Appeals committed no reversible error in setting
aside the trial court decision and dismissing said
complaint.
WHEREFORE, premises considered, the petition
is hereby DENIED. The assailed Decision dated
52 | P a g e

JURISDICTION; FOR CASE


DIGEST

May 16, 2006 and the Resolution dated October


5, 2006 of the Court of Appeals in CA-G.R. CV No.
63439 are AFFIRMED.

As prayed for, the respondent SEC issued a


Temporary Restraining Order on 23 November
1987.

SO ORDERED.

On 2 December 1987, petitioners filed a Motion


to Dismiss, alleging lack of jurisdiction over the
case on the part of the SEC. Private respondents
opposed said Motion to Dismiss.

SAAVEDRA VS SEC

On 11 December 1987, the SEC issued an order


denying the Motion to Dismiss.

FACTS:

HELD:

Private respondents case with the SEC, alleging


in then amended complaint that, private
respondents sold all their stocks, lights and
interests in Philippine Inc. to petitioners for the
sum of P12 million payable in installments; that
the sale was evidenced by a Memorandum of
Agreement and a Deed of Assignment, under the
Memorandum, the parties agreed that the sale
agreement would automatically be rescinded
upon failure on the part of petitioners to pay any
amount due; the petitioners failed to pay the last
sum due on the scheduled date, so that private
respondents rescinded the sale under an
instrument, Rescission of Memorandum of
Agreement. Private respondents prayed, among
others, that said instrument of rescission be
declared as having been made and executed -in
accordance with law and that a Temporary
Restraining Order be issued to enjoin petitioners
from ... "committing acts of disposal of the
Company assets, merchandise stocks,
equipment's, machineries and other company
paraphernalia."

In cases involving specialized disputes, the trend


has been to refer the same to an administrative
agency of special competence. As early as 1954,
the Court in Pambujan Sur United Mine Workers
vs. Samar Mining Co., Inc. held that under the
sense-making and expeditious doctrine of
primary jurisdiction . . . the courts cannot or will
not determine a controversy involving question
which is within the jurisdiction of an
administrative tribunal prior to the decision of
that question by the administrative tribunal,
where the question demands the exercise of
sound administrative discretion requiring the
special knowledge, experience, and services of
the administrative tribunal to determine
technical and intricate matters of fact, and a
uniformity of ruling is essential to comply with
the purposes of the regulatory statute
administered.

Blue Bar Coconut Phils vs Tantuico

FACTS:

Sometime in 1976, the respondent Acting


Chairman of the Commission on Audit initiated a
special audit of coconut end-user companies,
which include herein petitioners, with respect to
their Coconut Consumers Stabilization Fund levy
collections and the subsidies they had received.
As a result of the initial findings of the
Performance Audit Office with respect only to the
petitioners, respondent Acting COA Chairman
directed the Chairman, the Administrator, and
the Military Supervisor of PCA and the Manager
of the Coconut Consumers Stabilization Fund, in
various letters to them to collect the short levies
and overpaid subsidies, and to apply subsidy
claims to the settlement of short levies should
the petitioners fail to remit the amount due. The
petitioners contend that PCA auditor has no
jurisdiction over the subject matter.

HELD:
53 | P a g e

JURISDICTION; FOR CASE


DIGEST

Princlple of primary jurisdiction.The courts


cannot or will not determine a controversy
involving a question which is within the
jurisdiction of an administrative tribunal prior to
the decision of that question by the
administrative tribunal, where the question
demands the exercise of sound administrative
discretion requiring the special knowledge,
experience, and services of the administrative
tribunal, to determine technical and intricate
matters of fact, and a uniformity of ruling is
essential to comply with the purposes of the
r.egulatory statute administered. In this era of
clogged court dockets, the need for specialized
administrative boards or commissions with the
special knowledge, experience and capability to
hear and determine promptly disputes on
technical matters or essentially factual matters,
subject to judicial review in case of grave abuse
of discretion, has become well nigh
indispensable.

Province of Aklan vs Jody Kind Construction


FACTS:
The Province of Aklan (Aklan) and Jody King
Construction and Development Corp. (JKCDC)
entered into a contract for the design and
construction of the Caticlan Jetty Port and
Terminal (Phase I) in Malay, Aklan. In the course
of construction, Aklan issued variation/change
orders for additional works, which agreed upon
by the parties.

Aklan entered into a negotiated contract with


JKCDC for the construction of Passenger Terminal
Building (Phase II) also at Caticlan Jetty Port in
Malay, Aklan. JKCDC made a demand for the total
amount of P22,419,112.96 covering the items
which Aklan allegedly failed to settle. JKCDC then
filed a civil case with the RTC of Marikina City
(RTC) against Aklan for the collection of said
amount. The RTC issued a writ of preliminary
attachment against Aklan. The RTC later ruled in
favor of JKCDC (RTC Decision). Since Aklans
motion for reconsideration of the RTC Decision
was filed out of time, a writ of execution was
later issued. The sheriff served notices of
garnishment on Land Bank of the Philippines,
Philippine National Bank and Development Bank
of the Philippines at their branches in Kalibo,
Aklan for the satisfaction of the judgment debt
from the funds deposited under Aklans account.
Said banks, however, refused to give due course
to the court order, citing the relevant provisions
of statutes, circulars and jurisprudence on the
determination of government monetary
liabilities, their enforcement and satisfaction.

ISSUE:
Does the COA have primary jurisdiction over
JKCDCs money claims against Aklan?

where there is estoppel on the part of the party


invoking the doctrine; (b) where the challenged
administrative act is patently illegal, amounting
to lack of jurisdiction; (c) where there is
unreasonable delay or official inaction that will
irretrievably prejudice the complainant; (d)
where the amount involved is relatively small so
as to make the rule impractical and oppressive;
(e) where the question involved is purely legal
and will ultimately have to be decided by the
courts of justice; (f) where judicial intervention is
urgent; (g) when its application may cause great
and irreparable damage; (h) where the
controverted acts violate due process; (i) when
the issue of non-exhaustion of administrative
remedies has been rendered moot; (j) when
there is no other plain, speedy and adequate
remedy; (k) when strong public interest is
involved; and, (l) in quo warranto
proceedings.31 However, none of the foregoing
circumstances is applicable
in the present case.
The doctrine of primary jurisdiction does not
warrant a court to
arrogate unto itself authority to resolve a
controversy the jurisdiction over
which is initially lodged with an administrative
body of special
competence.32 All the proceedings of the court
in violation of the doctrine
and all orders and decisions rendered thereby
are null and void.33"

HELD:
YES. "There are established exceptions to the
doctrine of primary jurisdiction, such as: (a)

Garcia vs Executive Secreatary


54 | P a g e

JURISDICTION; FOR CASE


DIGEST

FACTS:
Provost Martial General of the Armed Forces of
the Philippines (AFP), Col. Henry A. Galarpe, by
command of Vice-Admiral De Los Reyes, issued a
Restriction to Quarters. Thereafter, a Charge
Sheet dated October 27, 2004 was filed with the
Special General Court Martial NR 2 presided by
Maj. Gen. Emmanuel R. Teodosio, AFP, (Ret.),
enumerating the following violations allegedly
committed by petitioner After six (6) years and
two (2) months of preventive confinement, on
December 16, 2010, petitioner was released
from the Camp Crame Detention Center.8
The Office of the President, or the President as
Commander-in-Chief of the AFP and acting as the
Confirming Authority under the Articles of War,
confirmed the sentence imposed by the Court
Martial against petitioner. The petitioners argued
that THE JURISDICTION OF THE GENERAL COURT
MARTIAL CEASED IPSO FACTO UPON THE
RETIREMENT OF PETITIONER, FOR WHICH
REASON THE OFFICE OF THE PRESIDENT ACTED
WITHOUT JURISDICTION IN ISSUING THE
CONFIRMATION OF SENTENCE, AND
PETITIONER'S ARREST AND CONFINEMENT
PURSUANT THERETO IS ILLEGAL, THUS
WARRANTING THE WRIT OF HABEAS CORPUS.

parties but continues until the case is


terminated.It is indisputable that petitioner
was an officer in the active service of the AFP in
March 2003 and 2004, when the alleged
violations were committed. The charges were
filed on October 27, 2004 and he was arraigned
on November 16, 2004. Clearly, from the time
the violations were committed until the time
petitioner was arraigned, the General Court
Martial had jurisdiction over the case. Wellsettled is the rule that jurisdiction once acquired
is not lost upon the instance of the parties but
continues until the case is terminated. Therefore,
petitioners retirement on November 18, 2004
did not divest the General Court Martial of its
jurisdiction.

Kulayan vs Tan

FACTS:
Three members from the International
Committee of the Red Cross
(ICRC) were kidnapped in the vicinity of the
Provincial Capitol in Patikul, Sulu.
Andres Notter, Eugenio Vagni, and Marie Jean
Lacaba, were purportedly inspecting

HELD:
Well-settled is the rule that jurisdiction once
acquired is not lost upon the instance of the

a water sanitation project for the Sulu Provincial


Jail when they were seized by

three armed men who were later confirmed to be


members of the Abu Sayyaf
Group (ASG). A Local Crisis Committee, later
renamed Sulu Crisis Management Committee
(Committee) was then formed to investigate the
kidnapping incident.
The Committee convened under the leadership
of respondent Abdusakur Mahail
Tan, the Provincial Governor of Sulu. Governor
Tan issued Proclamation No. 1, Series of 2009,
declaring a state of emergency in the province of
Sulu. The Proclamation cited the kidnapping
incident as a ground for the said declaration,
describing it as a terrorist act pursuant to the
Human Security Act (R.A. 9372). It also invoked
Section 465 of the Local
Government Code of 1991 (R.A. 7160), which
bestows on the Provincial Governorthe power to
carry out emergency measures during manmade and natural disasters and calamities, and
to call upon the appropriate national law
enforcement agencies to suppress disorder and
lawless violence. In the Proclamation, Tan called
upon the PNP and the Civilian Emergency Force
(CEF) to set up checkpoints and chokepoints,
conduct general search and seizures including
arrests, and other actions necessary to ensure
public safety. Petitioners, Jamar Kulayan, et al.
claimed that Proclamation No. 1-09 was issued
ultra vires, and thus null and void, for violating
Sections 1 and 18, Article VII of the Constitution,
which grants the President sole authority to
exercise emergency powers and calling-out
powers as the chief executive of the Republic
and commander-in-chief of the armed forces.
55 | P a g e

JURISDICTION; FOR CASE


DIGEST

HELD:
Hierarchy of Courts; The doctrine of hierarchy of
courts provides that where the issuance of an
extraordinary writ is also within the competence
of the Court of Appeals (CA) or the Regional Trial
Court (RTC), it is in either of these courts and not
in the Supreme Court, that the specific action for
the issuance of such writ must be sought unless
special and important laws are clearly and
specifically set forth in the petition.We first
dispose of respondents invocation of the
doctrine of hierarchy of courts which allegedly
prevents judicial review by this Court in the
present case, citing for this specific purpose,
Montes v. Court of Appeals, 489 SCRA 382
(2006), and Purok Bagong Silang Association,
Inc. v. Yuipco, 384 SCRA 152 (2002), Simply put,
the doctrine provides that where the issuance of
an extraordinary writ is also within the
competence of the CA or the RTC, it is in either of
these courts and not in the Supreme Court, that
the specific action for the issuance of such writ
must be sought unless special and important
laws are clearly and specifically set forth in the
petition. The reason for this is that this Court is a
court of last resort and must so remain if it is to
perform the functions assigned to it by the
Constitution and immemorial tradition. It cannot
be burdened with deciding cases in the first
instance. x x x The instant case stems from a
petition for certiorari and prohibition, over which
the Supreme Court possesses original
jurisdiction. More crucially, this case involves
acts of a public official which pertain to

restrictive custody, and is thus impressed with


transcendental public importance that would
warrant the relaxation of the general rule. The
Court would be remiss in its constitutional duties
were it to dismiss the present petition solely due
to claims of judicial hierarchy.

G.R. No. 190710

June 6, 2011

FACTS:

JESSE U. LUCAS vs. JESUS S. LUCAS,


This is a petition for review on certiorari towards
the decision of the Court of Appeals reversing the
decision of the Regional Trial Court regarding the
claim of petitioner Jesse Lucas, in his Petition to
Establish Illegitimate Filiation with Motion for the
submission of Parties to DNA Testing.

The CA held that the RTC did not acquire


jurisdiction over the person of respondent, as no
summons had been served on him. Respondents
special appearance could not be considered as
voluntary appearance because it was filed only
for the purpose of questioning the jurisdiction of
the court over respondent. Although respondent
likewise questioned the courts jurisdiction over
the subject matter of the petition, the same is
not equivalent to a waiver of his right to object to
the jurisdiction of the court over his person.

56 | P a g e

JURISDICTION; FOR CASE


DIGEST

The CA remarked that petitioner filed the petition


to establish illegitimate filiation, specifically
seeking a DNA testing order to abbreviate the
proceedings. It noted that petitioner failed to
show that the four significant procedural aspects
of a traditional paternity action had been met.
The CA further held that a DNA testing should
not be allowed when the petitioner has failed to
establish a prima facie case.

ISSUE:

Whether or not the Court of Appeals erred in


reversing the decision of the RTC due to lack of
jurisdiction

HELD:

The Court held in the affirmative because the


herein petition to establish illegitimate filiation is
an action in rem. By the simple filing of the
petition to establish illegitimate filiation before
the RTC, which undoubtedly had jurisdiction over
the subject matter of the petition, the latter
thereby acquired jurisdiction over the case. An in
rem proceeding is validated essentially through
publication. Publication is notice to the whole
world that the proceeding has for its object to
bar indefinitely all who might be minded to make
an objection of any sort to the right sought to be
established.[24] Through publication, all interested
parties are deemed notified of the petition. The
service of summons or notice is made to the
defendant, it is not for the purpose of vesting the
court with jurisdiction, but merely for satisfying
the due process requirements

An action in personam is lodged against a person


based on personal liability; an action in rem is
directed against the thing itself instead of the
person; while an action quasi in rem names a
person as defendant, but its object is to subject
that person's interest in a property to a
corresponding lien or obligation. A petition
directed
against
the
"thing"
itself
or
the res, which concerns the status of a person,
like a petition for adoption, annulment of
marriage, or correction of entries in the birth
certificate, is an action in rem.[22]

In an action in personam, jurisdiction over the


person of the defendant is necessary for the
court to validly try and decide the case. In a
proceeding in rem or quasi in rem, jurisdiction
over the person of the defendant is not a
prerequisite to confer jurisdiction on the court,
provided that the latter has jurisdiction over
the res. Jurisdiction over the resis acquired
either (a) by the seizure of the property under
legal process, whereby it is brought into actual
custody of the law, or (b) as a result of the
institution of legal proceedings, in which the
power of the court is recognized and made
effective.

REPUBLIC OF THE PHILIPPINES vs. MERLYN


MERCADERA
G.R. No. 186027

December 8, 2010

FACTS:

On June 6, 2005, Merlyn Mercadera (Mercadera),


represented by her sister and duly constituted
Attorney-in-Fact, Evelyn M. Oga (Oga), sought the
correction of her given name as it appeared in
her Certificate of Live Birth from Marilyn L.
Mercadera to Merlyn L. Mercadera before the
Office of the Local Civil Registrar of Dipolog City
pursuant to Republic Act No. 9048.

57 | P a g e

JURISDICTION; FOR CASE


DIGEST

Under R.A. No. 9048, the city or municipal civil


registrar or consul general is now authorized to
effect the change of first name or nickname and
the correction of clerical or typographical errors
in civil registry entries. However, Civil Registrar
of Dipolog refused to correct unless a court order
was obtained "because the Civil Registrar therein
is not yet equipped with a permanent
appointment before he can validly act on
petitions for corrections filed before their office
as mandated by R.A. No. 9048."

Mercadera then filed a Petition For Correction of


Some Entries as Appearing in the Certificate of
Live Birth under Rule 108 before the Regional
Trial Court of Dipolog City (RTC). Upon receipt of
the petition for correction of entry, the RTC
issued an order, dated June 10, 2005, for the
hearing of said petition. The Office of the
Solicitor General (OSG) deputized the Office of
the City Prosecutor to assist in the case. Without
any objection from the City Prosecutor, the
testimony of Oga and several photocopies of
documents were formally offered and marked as
evidence to prove that Mercadera never used the
name "Marilyn" in any of her public or private
transactions.

In its September 28, 2005 Decision, the RTC


granted the petition and ruled that the
documentary evidence presented by Mercadera
sufficiently supported the circumstances alleged
in her petition. Considering that she had used
"Merlyn" as her given name since childhood until
she discovered the discrepancy in her Certificate

of Live Birth, the RTC was convinced that the


correction was justified.

The OSG timely appealed praying for the reversal


and setting aside of the RTC decision. For the
OSG, the correction in the spelling of
Mercaderas given name "is in truth a material
correction as it would modify or increase
substantive rights", which would have been
proper had she filed a petition under Rule 103
and proved any of the grounds therefor.

The CA was not persuaded. In its December 9,


2008 Decision, the appellate court affirmed the
questioned RTC order.

On March 6, 2009, the OSG filed the present


petition. On behalf of Mercadera, the Public
Attorneys Office (PAO) filed its Comment on July
3, 2009.

ISSUES:

WHETHER OR NOT THE COURT OF APPEALS


ERRED ON A QUESTION OF LAW IN GRANTING

THE CHANGE IN RESPONDENTS NAME UNDER


RULE 103.

HELD:

The proceeding under Rule 103 is an action in


rem which requires publication of the order
issued by the court to afford the State and all
other interested parties to oppose the
petition. When complied with, the decision binds
not only the parties impleaded but the whole
world. As notice to all, publication serves to
indefinitely bar all who might make an
objection. It is the publication of such notice
that brings in the whole world as a party in the
case and vests the court with jurisdiction to hear
and decide it..

Rule 108, on the other hand, implements judicial


proceedings for the correction or cancellation of
entries in the civil registry pursuant to Article
412 of the Civil Code. Entries in the civil register
refer to "acts, events and judicial decrees
concerning the civil status of persons," also as
enumerated in Article 408 of the same law.

In the case at bench, the OSG posits that the


conversion from "MARILYN" to "MERLYN" is not a
58 | P a g e

JURISDICTION; FOR CASE


DIGEST

correction of an innocuous error but a material


correction tantamount to a change of name
which entails a modification or increase in
substantive rights. For the OSG, this is a
substantial error that requires compliance with
the procedure under Rule 103, and not Rule 108.

SPOUSES
FERNANDO
TORRES andIRMA
TORRES VS. AMPARO MEDINA and the EXOFFICIO SHERIFF of the RTC of Quezon City
G.R. No. 166730

A change of ones name under Rule 103 can be


granted, only on grounds provided by law, there
must be a proper and compelling reason for the
change and proof that the person requesting will
be prejudiced by the use of his official name. In
petitions for correction, only clerical, spelling,
typographical and other innocuous errors in the
civil registry may be raised. Considering that the
enumeration in Section 2, Rule 108 also includes
"changes of name," the correction of a patently
misspelled name is covered by Rule 108. Suffice
it to say, not all alterations allowed in ones
name are confined under Rule 103. Corrections
for clerical errors may be set right under Rule
108.
Mercadera complied with the requirement for an
adversarial proceeding before the lower court.
The publication and posting of the notice of
hearing in a newspaper of general circulation and
the notices sent to the OSG and the Local Civil
Registry are sufficient indicia of an adverse
proceeding. Considering that the OSG did not
oppose the petition and the motion to present its
evidence ex parte when it had the opportunity to
do so, it cannot now complain that the
proceedings in the lower court were procedurally
defective. Wherefore, the December 9, 2008
Decision of the Court of Appeals is AFFIRMED.

Quezon City for the declaration of nullity of the


extrajudicial foreclosure of mortgage conducted
by the Ex-Officio Sheriff. The same was docketed
as Civil Case No. Q-99-38781.

March 10, 2010

FACTS:

On July 28, 1994, respondent Amparo


Medina (Medina) wrote a letter[4] to the Office of
the Sheriff, Regional Trial Court (RTC) of Quezon
City, applying for the extrajudicial foreclosure of
mortgage of the property of petitioner spouses
Fernando and Irma Torres (Spouses Torres) which
was covered by Transfer Certificate of Title No.
RT-61056 (354973) and which is subject of a
Deed of Mortgage[5] dated December 20, 1993.

However the complaint was dismissed by


the RTC due to the motion to dismiss by Medina
and the Court ruled that res judicata was present
and that the spouses torres is guilty of forum
shopping.

The Spouses Torres then filed a Motion


for Reconsideration dated August 30, 2004,
which was, however, denied by the CA in the
Resolution dated January 18, 2005.

Hence, herein petition.


ISSUE:

On May 27, 1997, the Office of the ExOfficio Sheriff


issued
a
Notice
of
Sheriffs Sale[6] and, on June 30, 1997, sold at
public
auction
the
subject
property
to Medinabeing the highest bidder thereof. A
Certificate of Sale[7] was thereafter issued
to Medina.

Whether or not there is res judicata in this case

HELD:

On September 21, 1999, the Spouses


Torres filed a Complaint[8] before the RTC of
59 | P a g e

JURISDICTION; FOR CASE


DIGEST

jurisdiction over the


matter and the parties;

subject

The court held in the affirmative.


(3) the disposition of the case
must be a judgment on the
merits; and
As borne from the records of the case, the
Spouses Torres first instituted Civil Case No. Q94-18962 before the RTC of Quezon City and an
Entry of Judgment was already rendered. Yet,
they sought remedy to this court.

Res judicata literally means "a matter


adjudged; a thing judicially acted upon or
decided; a thing or matter settled by
judgment." Res judicata lays the rule that an
existing final judgment or decree rendered on
the merits, and without fraud or collusion, by a
court of competent jurisdiction, upon any matter
within its jurisdiction, is conclusive of the rights
of the parties or their privies, in all other actions
or suits in the same or any other judicial tribunal
of concurrent jurisdiction on the points and
matters in issue in the first suit.

The elements of res judicata are:

(1) the judgment sought to bar


the new action must be final;
(2) the decision must have been
rendered by a court having

(4) there must be as between the


first and second action identity of
parties, subject matter, and
causes of action.[29]

This Court has previously employed


various tests in determining whether or not there
is identity of causes of action as to warrant the
application of the principle of res judicata. One
test of identity is the "absence of inconsistency
test" where it is determined whether the
judgment sought will be inconsistent with the
prior judgment. If no inconsistency is shown, the
prior judgment shall not constitute a bar to
subsequent actions.[32]

It bears stressing that the doctrine of res


judicata actually
embraces
two
different
concepts: (1) bar by former judgment and (b)
conclusiveness of judgment.

of competent jurisdiction, is conclusively settled


by the judgment therein as far as the parties to
that action and persons in privity with them are
concerned and cannot be again litigated in any
future action between such parties or their
privies, in the same court or any other court of
concurrent jurisdiction on either the same
or different cause of action, while the judgment
remains unreversed by proper authority. It has
been held that in order that a judgment in one
action can be conclusive as to a particular matter
in another action between the same parties or
their privies, it is essential that the issue be
identical. If a particular point or question is in
issue in the second action, and the judgment will
depend on the determination of that particular
point or question, a former judgment between
the same parties or their privies will be final and
conclusive in the second if that same point or
question was in issue and adjudicated in the first
suit. Identity of cause of action is not required,
but merely identity of issues.[35]

ZENAIDA ACOSTA, EDUARDO ACOSTA, et. al.


vs.
TRINIDAD SALAZAR
AND
ANICETA
SALAZAR

The second concept conclusiveness of


judgment states that a fact or question which
was in issue in a former suit and was there
judicially passed upon and determined by a court
60 | P a g e

JURISDICTION; FOR CASE


DIGEST

G.R. No. 161034

June 30, 2009

FACTS:

On November 19, 1985, respondents Trinidad


and Aniceta Salazar (hereinafter, Salazars), filed
a petition for the cancellation of the entries
annotated at the back of Original Certificate of
Title (OCT) No. 40287 registered in the names of
spouses Juan Soriano and Vicenta Macaraeg, who
died without issue.[4] The Salazars claim that two
of the entries Entry Nos. 19756 and 20102
annotated at the back of the aforesaid title are
void since no consolidation of rights appear in
the Registry of Deeds (RD) of Tarlac to support
the entries; and that Transfer Certificate of Title
(TCT) No. 9297, which supposedly cancelled OCT
No. 40287, is non-existent according to a
certification issued by the RD.[5] On October 21,
1986, RTC Branch 63 of Tarlac resolved to grant
the petition and ordered the cancellation of Entry
No. 20102.[6] No respondent was impleaded in
the said petition.

Other motions were filed by the Salazarss and in


one motion they submitted, they prayed the
owners of the affected property to appear in
court.

It was at this stage of the proceedings that


herein
petitioners
together
with
other
subsequent purchasers for value of the disputed
property twenty-seven (27) titleholders in
all[11] filed their formal written comment dated
April 17, 1989.[12] In their comment, the
oppositors contended, among others, that they
had acquired their titles in good faith and for
value, and that the lower court, acting as a land
registration court, had no jurisdiction over issues
of ownership.

ISSUE:
Whether or not the previous order was null and
void due to lack of jurisdiction

HELD:
It is true that the registration of land under
the Torrens system is a proceeding in rem and
not in personam. Such a proceeding in rem,
dealing with a tangible res, may be instituted
and carried to judgment without personal service
upon the claimants within the state or notice by
mail to those outside of it. Jurisdiction is acquired
by virtue of the power of the court over the res.
Such a proceeding would be impossible were this
not so, for it would hardly do to make a
distinction between constitutional rights of
claimants who were known and those who were
not known to the plaintiff, when the proceeding
is to bar all.[30]

Interestingly, however, the proceedings


instituted by the Salazars both in Branch 63 of
the RTC of Tarlac for the cancellation of entries in
OCT No. 40287 and later in Branch 64 of the RTC
of Tarlac for quieting of title can hardly be
classified as actions in rem. The petition for
cancellation of entries annotated at the back of
OCT No. 40287 ought to have been directed
against specific persons: namely, the heirs of
Juan Soriano as appearing in Entry No. 20102
and, indubitably, against their successors-ininterest who have acquired different portions of
the property over the years because it is in the
nature of an action quasi in rem. Accordingly, the
Salazars should have impleaded as party
defendants the heirs of Juan Soriano and/or
Vicenta Macaraeg as well as those claiming
ownership over the property under their names
because they are indispensable parties. This was
not done in this case. [31] Since no indispensable
party was ever impleaded by the Salazars in
their petition for cancellation of entry filed before
Branch 63 of the RTC of Tarlac, herein petitioners
are not bound by the dispositions of the said
court.[32] Consequently, the judgment or order of
the said court never even acquired finality.

Apparently realizing their mistake, the


Salazars later on filed an action for quieting of
title, also an action quasi in rem, albeit this time
before Branch 64 of the RTC of Tarlac. Because
the Salazars miserably failed to prove the basis
for their claim, the RTC dismissed the complaint.

61 | P a g e

JURISDICTION; FOR CASE


DIGEST

Needless to say, the failure of the Salazars to


implead indispensable party defendants in the
petition for cancellation of entries in OCT No.
40287 should have been a ground for the RTC to
dismiss, or at least suspend, the proceedings of
the case.[35] Yet, although the action proceeded,
any judgment or order issued by the court
thereon is still null and void for want of authority
on the part of the court to act with respect to the
parties never impleaded in the action. [36] Thus,
the orders issued by said court dated October
21, 1986 and November 7, 1986 never acquired
finality

wife Alanga, evidenced by a deed of sale in favor


of Salzos signed solely by Alanga, on behalf of
Datto Darondon.
HELD:

MANOTOK

Facts: The consolidated seven cases have for


their common genesis the 1914 case of Cacho v.
Government of the United States[16] (1914 Cacho
case).
The 1914 Cacho Case

REPUBLIC
OF
THE
PHILIPPINES,
represented
by
the
ANTI-MONEY
LAUNDERING
COUNCIL
vs.
GLASGOW
CREDIT AND COLLECTION SERVICES, INC.
and
CITYSTATE
SAVINGS
BANK,
INC., respondents.

G.R. No. 170281

FACTS:

ISSUE:

January 18, 2008

Sometime in the early 1900s, the late Doa


Demetria Cacho (Doa Demetria) applied for the
registration of two parcels of land: (1) Lot 1 of
Plan II-3732, the smaller parcel with an area
of 3,635 square meters or 0.36 hectares (Lot 1);
and (2) Lot 2 of Plan II-3732, the larger parcel
with an area of 378,707 square meters or 37.87
hectares(Lot 2). Both parcels are situated in
what was
then the Municipality of Iligan, Moro Province,
which later became Sitio Nunucan, then Brgy.
Suarez, in Iligan City, Lanao del Norte. Doa
Demetrias applications for registration were
docketed as GLRO Record Nos. 6908 and 6909.
The application in GLRO Record No.
6908 covered Lot 1, the smaller parcel of
land. Doa Demetria allegedly acquired Lot 1 by
purchase from Gabriel Salzos (Salzos). Salzos, in
turn, bought Lot 1 from Datto Darondon and his

The application in GLRO Record No.


6909 involved Lot 2, the bigger parcel of
land. Doa Demetria purportedly
purchased Lot 2 from Datto Bunglay. Datto
Bunglay claimed to have inherited Lot 2 from his
uncle, Datto Anandog, who died without issue.
Only the Government opposed Doa Demetrias
applications for registration on the ground that
the two parcels of land were the property of
the United States and formed part of a military
reservation, generally known as Camp Overton.
On December 10, 1912, the land registration
court (LRC) rendered its Decision in GLRO Record
Nos. 6908 and 6909.
Based on the evidence, the LRC made the
following findings in GLRO Record No. 6908:
6th. The court is convinced from the proofs that
the small parcel of land sold by the Moro woman
Alanga was the home of herself and her
husband, Darondon, and was their conjugal
property; and the court so finds.
xxxx
As we have seen, the deed on which applicants
title to the small parcel rests, is executed only by
the Moro woman Alanga, wife of Datto Darondon,
which is not permitted either by the Moro laws or
the Civil Code of the Philippine Islands. It
appears that the husband of Alanga, Datto
62 | P a g e

JURISDICTION; FOR CASE


DIGEST

Darondon, is alive yet, and before admitting this


parcel to registration it is ordered that a deed
from Datto Darondon, husband of Alanga, be
presented, renouncing all his rights in the small
parcel of land object of Case No. 6908, in favor of
the applicant.[17] (Emphases supplied.)
In GLRO Record No. 6909, the LRC observed and
concluded that:
A tract of land 37 hectares in area, which is the
extent of the land under discussion, is larger
than is cultivated ordinarily by the Christian
Filipinos. In the Zamboanga cadastral case of
thousands of parcels now on trial before this
court, the average size of the parcels is not
above 3 or 4 hectares, and the court doubts very
much if a Moro with all his family could cultivate
as extensive a parcel of land as the one in
question. x x x x x x
The court is also convinced from the proofs that
the small portion in the southern part of the
larger parcel, where, according to the proofs,
Datto Anandog had his house and where there
still exist some cocos and fruit trees, was the
home of the said Moro Datto Anandog; and the
court so finds. As to the rest of the large parcel
the court does not find the title of Datto Bunglay
established. According to his own declaration his
residence on this land commenced only a few
days before the sale. He admitted that the coco
trees he is supposed to have planted had not yet
begun to bear fruit at the time of the sale, and
were very small. Datto Duroc positively denies
that Bunglay lived on the land, and it clearly
appears that he was not on the land when it was
first occupied by the military. Nor does Datto
Bunglay claim to have planted the three mango

trees by the roadside near point 25 of the


plan. The court believes that all the rest of this
parcel, not occupied nor cultivated by Datto
Anandog, was land claimed by Datto Duroc and
also by Datto Anandog and possibly by other
dattos as a part of their general jurisdiction, and
that it is the class of land that Act No. 718
prohibits the sale of, by the dattos, without the
express approval of the Government.
It is also found that Datto Bunglay is the nephew
of Dato Anandog, and that the Moro woman
Alanga, grantor of the small parcel, is the sister
of Datto Anandog, and that he died without
issue.
xxxx
It appears also that according to the provisions
of the Civil Code as also the provisions of the
Luwaran Code of the Moros, the Moro woman
Alanga has an interest in the portion of land left
by her deceased brother, Datto Anandog. By
article LXXXV, section 3, of the Luwaran Code, it
will be seen that the brothers and sisters of a
deceased Moro inherit his property to the
exclusion of the more distant
relatives. Therefore Datto Bunglay had no legal
interest whatever in the land to sell to the
applicant, Doa Demetria Cacho. But the Moro
woman, Alanga, having appeared as a witness
for the applicant without having made any claim
to the land, the court finds from this fact that she
has ratified the sale made by her nephew.
The court therefore finds that the applicant
Doa Demetria Cacho is owner of the portion of
land occupied and planted by the deceased
Datto Anandog in the southern part of the large

parcel object of expediente No. 6909 only; and


her application as to all the rest of the land
solicited in said case is denied. And it is ordered
that a new survey of the land be made and a
corrected plan be presented, excluding all the
land not occupied and cultivated by Datto
Anandog; that said survey be made and the
corrected plan presented on or before the
30th day of March, 1913, with previous notice to
the commanding general of the Division of the
Philippines.
On the 8th day of December, the court was
at Camp Overton and had another ocular
inspection of the land for the purpose of fixing
the limits of the part cultivated by Datto
Anandog, so often mentioned herein, with
previous notice to the applicant and her husband
and representative, Seor Dionisio Vidal. Having
arrived late, Seor Vidal did not assist in the
ocular inspection, which was fixed for 3 oclock,
p.m. of the day mentioned. But the court,
nevertheless, set stakes marking the N.E., S.E.,
and S.W. corners of the land found to have been
cultivated by the deceased Anandog. The N.E.
limit of said land is a brook, and the N.W. corner
is the point where the brook intersects the shore
line of the sea, the other corners mentioned
being marked with pine stakes. And it is ordered
that the new survey be made in accordance with
the points mentioned, by tracing four straight
lines connecting these four points. Between the
portion cultivated by Datto Anandog and the
mouth of the River Agus there is a high steep hill
and the court does not believe it possible to
cultivate said hill, it being covered with rocks and
forest.[18] (Emphases supplied.)

63 | P a g e

JURISDICTION; FOR CASE


DIGEST

The LRC additionally decreed at the end of its


December 10, 1912 Decision:
It is further ordered that one-half of the costs of
the new survey be paid by the applicant and the
other half by the Government of the United
States, and that the applicant present the
corresponding deed from Datto Darondon on or
before the above-mentioned 30th day of March,
1913. Final decision in these cases is reserved
until the presentation of the said deed and the
new plan.[19]
Apparently dissatisfied with the foregoing LRC
judgment, Doa Demetria appealed to this
Court. In its Decision dated December 10, 1914,
the Court affirmed in toto the LRC Decision of
December 10, 1912, well satisfied that the
findings of fact of the court below were fully
sustained by the evidence adduced during trial.
Eighty-three years later, in 1997, the
Court was again called upon to settle a matter
concerning the registration of Lots 1 and 2 in the
case of Cacho v. Court of Appeals[20](1997 Cacho
case).
The 1997 Cacho Case
On June 29, 1978, Teofilo Cacho (Teofilo),
claiming to be the late Doa Demetrias son and
sole heir, filed before the RTC a petition for
reconstitution of two original certificates of title
(OCTs), docketed under the original GLRO Record
Nos. 6908 and 6909.

Teofilos petition was opposed by the Republic,


National Steel Corporation (NSC), and the City
of Iligan.
Acting on the motion for judgment on demurrer
to evidence filed by the Republic and NSC, the
RTC initially dismissed Teofilos petition for
reconstitution of titles because there was
inadequate evidence to show the prior existence
of the titles sought to be restored. According to
the RTC, the proper remedy was a petition for the
reconstitution of decrees since it is undisputed
that in Cases No. 6908 and 6909, Decrees No.
10364 and 18969, respectively, were
issued. Teofilo sought leave of court for the
filing and admission of his amended petition, but
the RTC refused. When elevated to this Court
in Cacho v. Mangotara, docketed as G.R. No.
85495, the Court resolved to remand the case to
the RTC, with an order to the said trial court to
accept Teofilos amended petition and to hear it
as one for re-issuance of decrees.

Record No. 6909 on July 8, 1915 was sufficiently


established by the certifications and testimonies
of concerned officials. The original issuance of
these decrees presupposed a prior judgment that
had become final.
On appeal, the Court of Appeals reversed the
RTC Decision dated June 9, 1993 and dismissed
the petition for re-issuance of Decree Nos. 10364
and 18969 because: (1) re-issuance of Decree
No. 18969 in GLRO Record No. 6909 could not be
made in the absence of the new survey ordered
by this Court in the 1914 Cacho case; (2) the heir
of a registered owner may lose his right to
recover possession of the property and title
thereto by laches; and (3) Teofilo failed to
establish his identity and existence and that he
was a real party-in-interest.

In opposing Teofilos petition, the Republic and


NSC argued that the same suffered from
jurisdictional infirmities; that Teofilo was not the
real party-in-interest; that Teofilo was guilty of
laches; that Doa Demetria was not the
registered owner of the subject parcels of land;
that no decrees were ever issued in Doa
Demetrias name; and that the issuance of the
decrees was dubious and irregular.

Teofilo then sought recourse from this Court in


the 1997 Cacho case. The Court reversed the
judgment of the Court of Appeals and reinstated
the decision of the RTC approving the re-issuance
of Decree Nos. 10364 and 18969. The Court
found that such decrees had in fact been issued
and had attained finality, as certified by the
Acting Commissioner, Deputy Clerk of Court III,
Geodetic Engineer, and Chief of Registration of
the then Land Registration Commission, now
National Land Titles and Deeds Registration
Administration (NALTDRA). The Court further
reasoned that:

After trial, on June 9, 1993, the RTC rendered its


Decision granting Teofilos petition and ordering
the reconstitution and re-issuance of Decree Nos.
10364 and 18969. The RTC held that the
issuance of Decree No. 10364 in GLRO No. 6908
on May 9, 1913 and Decree No. 18969 in GLRO

[T]o sustain the Court of Appeals ruling as


regards requiring petitioners to fulfill the
conditions set forth in Cacho vs. U.S. would
constitute a derogation of the doctrine of res
judicata. Significantly, the issuance of the subject
decrees presupposes a prior final judgment
64 | P a g e

JURISDICTION; FOR CASE


DIGEST

because the issuance of such decrees is a mere


ministerial act on part of the Land Registration
Commission (now the NALTDRA), upon
presentation of a final judgment. It is also worth
noting that the judgment in Cacho vs. U.S. could
not have acquired finality without the prior
fulfillment of the conditions in GLRO Record No.
6908, the presentation of the corresponding
deed of sale from Datto Dorondon on or before
March 30, 1913 (upon which Decree No. 10364
was issued on May 9, 1913); and in GLRO Record
No. 6909, the presentation of a new survey per
decision of Judge Jorge on December 10, 1912
and affirmed by this Court on December 10,
1914 (upon which Decree No. 18969 was issued
on July 8, 1915).
Requiring the submission of a new plan
as a condition for the re-issuance of the decree
would render the finality attained by the Cacho
vs. U.S. case nugatory, thus, violating the
fundamental rule regarding res judicata. It must
be stressed that the judgment and the resulting
decree are res judicata, and these are binding
upon the whole world, the proceedings being in
the nature of proceedings in rem. Besides, such
a requirement is an impermissible assault upon
the integrity and stability of the Torrens System
of registration because it also effectively renders
the decree inconclusive.[21]
As to the issue of laches, the Court referred to
the settled doctrine that laches cannot bar the
issuance of a decree. A final decision in land
registration cases can neither be rendered
inefficacious by the statute of limitations nor by
laches.

Anent the issue of the identity and existence of


Teofilo and he being a real party-in-interest, the
Court found that these were sufficiently
established by the records. The Court relied on
Teofilos Affidavit of Adjudication as Doa
Demetrias sole heir, which he executed before
the Philippine Consulate General
in Chicago, United States of America (U.S.A.); as
well as the publication in the Times Journal of the
fact of adjudication of Doa Demetrias
estate. Teofilo also appeared personally before
the Vice Consul of the Philippine Consulate
General in Chicago to execute a Special Power of
Attorney in favor of Atty. Godofredo Cabildo (Atty.
Cabildo) who represented him in this case. The
Court stressed that the execution of public
documents is entitled to the presumption of
regularity and proof is required to assail and
controvert the same.

In the Resolution dated July 28, 1997,[22] the


Court denied the Motions for Reconsideration of
the Republic and NSC.

As a result of the 1997 Cacho case, the decrees


of registration were re-issued bearing new
numbers and OCTs were issued for the two
parcels of land in Doa Demetrias name. OCT
No. 0-1200 (a.f.) was based on re-issued Decree
No. N-219464 in GLRO Record No. 6908,
while OCT No. 0-1201 (a.f.) was based on reissued Decree No. N-219465 in GLRO Record No.
6909.

II
THE ANTECENT FACTS
OF THE PETITIONS AT BAR

The dispute over Lots 1 and 2 did not end with


the termination of the 1997 Cacho case. Another
four cases involving the same parcels of land
were instituted before the trial courts during and
after the pendency of the 1997 Cacho
case. These cases are: (1) the Expropriation
Case, G.R. No. 170375; (2) the Quieting of Title
Case, G.R. Nos. 178779 and 178894; (3) the
Ejectment or Unlawful Detainer Case, G.R.
No. 170505 (execution pending appeal before
the RTC) and G.R. Nos. 173355-56 and 17356364 (execution pending appeal before the Court of
Appeals); and (4) the Cancellation of Titles and
Reversion Case, G.R. No. 173401. These cases
proceeded independently of each other in the
courts a quo until they reached this Court via the
present Petitions. In the Resolution[23] dated
October 3, 2007, the Court consolidated the
seven Petitions considering that they either
originated from the same case or involved similar
issues.

Expropriation Case
(G.R. No. 170375)
65 | P a g e

JURISDICTION; FOR CASE


DIGEST

The Complaint for Expropriation was


originally filed on August 15, 1983 by the Iron
and Steel Authority (ISA), now the NSC, against
Maria Cristina Fertilizer Corporation (MCFC), and
the latters mortgagee, the Philippine National
Bank (PNB). The Complaint was docketed as
Civil Case No. 106 and raffled to RTC-Branch 1,
presided over by Judge Mangotara.

ISA was created pursuant to Presidential


Decree No. 2729[24] dated August 9, 1973, to
strengthen, develop, and promote the iron and
steel industry in the Philippines. Its existence
was extended until October 10, 1988.

On November 16, 1982, during the existence of


ISA, then President Ferdinand E. Marcos issued
Presidential Proclamation No. 2239,[25] reserving
in favor of ISA a parcel of land in Iligan City,
measuring 302,532 square meters or 30.25
hectares, to be devoted to the integrated steel
program of the Government. MCFC occupied
certain portions of this parcel of land. When
negotiations with MCFC failed, ISA was
compelled to file a Complaint for Expropriation.

When the statutory existence of ISA expired


during the pendency of Civil Case No. 106, MCFC
filed a Motion to Dismiss the case alleging the
lack of capacity to sue of ISA. The RTC-Branch 1

granted the Motion to Dismiss in an Order dated


November 9, 1988. ISA moved for
reconsideration or, in the alternative, for the
substitution of the Republic as plaintiff in Civil
Case No. 106, but the motion was denied by RTCBranch 1. The dismissal of Civil Case No. 106
was affirmed by the Court of Appeals, thus, ISA
appealed to this Court. In Iron and Steel
Authority v. Court of Appeals[26] (ISA case), the
Court remanded the case to RTC-Branch 1, which
was ordered to allow the substitution of the
Republic for ISA as plaintiff. Entry of Judgment
was made in the ISA case on August 31, 1998. In
an Order[27] dated November 16, 2001, the RTCBranch 1 allowed the substitution of the Republic
for ISA as plaintiff in Civil Case No. 106.

Alleging that Lots 1 and 2 involved in


the 1997 Cacho case encroached and overlapped
the parcel of land subject of Civil Case No. 106,
the Republic filed with the RTC-Branch 1 a Motion
for Leave to File Supplemental Complaint dated
October 7, 2004 and to Admit the Attached
Supplemental Complaint dated September 28,
2004[28] seeking to implead in Civil Case No. 106
Teofilo Cacho and Demetria Vidal and their
respective successors-in-interest, LANDTRADE
and AZIMUTH.

MCFC opposed the Motion for leave to file


and to admit the Supplemental Complaint on the
ground that the Republic was without legal
personality to file the same because ISA was the
plaintiff in Civil Case No. 106. MCFC argued that
the Republic failed to move for the execution of

the decision in the ISA case within the


prescriptive period of five years, hence, the only
remedy left was for the Republic to file an
independent action to revive the
judgment. MCFC further pointed out that the
unreasonable delay of more than six years of the
Republic in seeking the substitution and
continuation of the action for expropriation
effectively barred any further proceedings
therein on the ground of estoppel by laches.

In its Reply, the Republic referred to the


Order dated November 16, 2001 of the RTCBranch 1 allowing the substitution of the
Republic for ISA.

In an Order dated April 4, 2005, the RTCBranch 1 denied the Motion of the Republic for
leave to file and to admit its Supplemental
Complaint. The RTC-Branch 1 agreed with MCFC
that the Republic did not file any motion for
execution of the judgment of this Court in the ISA
case. Since no such motion for execution had
been filed, the RTC-Branch 1 ruled that its Order
dated November 16, 2001, which effected the
substitution of the Republic for ISA as plaintiff in
Civil Case No. 106, was an honest mistake. The
Republic filed a Motion for Reconsideration of the
April 4, 2005 Order of the RTC-Branch 1.

MCFC then filed a Motion to Dismiss Civil


Case No. 106 for: (1) failure of the Republic to
66 | P a g e

JURISDICTION; FOR CASE


DIGEST

implead indispensable parties because MCFC


insisted it was not the owner of the parcels of
land sought to be expropriated; and (2) forum
shopping considering the institution by the
Republic on October 13, 2004 of an action for the
reversion of the same parcels subject of the
instant case for expropriation.

Judge Mangotara of RTC-Branch 1 issued a


Resolution[29] on July 12, 2005, denying for lack of
merit the Motion for Reconsideration of the Order
dated April 4, 2005 filed by the Republic, and
granting the Motion to Dismiss Civil Case No. 106
filed by MCFC. Judge Mangotara justified the
dismissal of the Expropriation Case thus:

What the Republic seeks [herein] is the


expropriation of the subject parcels of
land. Since the exercise of the power of eminent
domain involves the taking of private lands
intended for public use upon payment of just
compensation to the owner x x x, then a
complaint for expropriation must, of necessity,
be directed against the owner of the land subject
thereof. In the case at bar, the decision of the
Supreme Court in Cacho v. Government of the
United States x x x, decreeing the registration of
the subject parcels of land in the name of the
late Doa Demetria Cacho has long attained
finality and is conclusive as to the question of
ownership thereof. Since MCFC, the only
defendant left in this case, is not a proper party
defendant in this complaint for expropriation, the
present case should be dismissed.

This Court notes that the Republic [has filed


reversion proceedings] dated September 27,
2004, involving the same parcels of land,
docketed as Case No. 6686 pending before the
Regional Trial Court of Lanao del Norte, Iligan
City Branch 4. [The Republic], however, did not
state such fact in its Verification and
Certification of Non-Forum Shopping attached to
its Supplemental Complaint dated September 28,
2004. [It is therefore] guilty of forum
shopping. Moreover, considering that in the
Reversion case, [the Republic] asserts ownership
over the subject parcels of land, it cannot be
allowed to take an inconsistent position in this
expropriation case without making a mockery of
justice.[30]

The Republic filed a Motion for


Reconsideration of the Resolution dated July 12,
2005, insofar as it dismissed Civil Case No. 106,
but said Motion was denied by Judge Mangatora
in a Resolution[31] dated October 24, 2005.

On January 16, 2006, the Republic filed


with this Court the consolidated Petition for
Review on Certiorari and Petition
for Certiorari under Rules 45 and 65 of the Rules
of Court, respectively, docketed as G.R. No.
170375.

The Quieting of Title Case


(G.R. Nos. 178779 and 178894)

Demetria Vidal (Vidal) and AZIMUTH filed on


November 18, 1998, a Petition[32] for Quieting of
Title against Teofilo, Atty. Cabildo, and the
Register of Deeds of Iligan City, which was
docketed as Civil Case No. 4452 and raffled to
RTC-Branch 3.

In the Petition, Vidal claimed that she, and not


Teofilo, was the late Doa Demetrias sole
surviving heir, entitled to the parcels of land
covered by OCT Nos. 0-1200 (a.f.) and 0-1201
(a.f.). She averred that she is the daughter of
Francisco Cacho Vidal (Francisco) and Fidela
Arellano Confesor. Francisco was the only child
of Don Dionisio Vidal and Doa Demetria.

AZIMUTH, for its part, filed the Petition as Vidals


successor-in-interest with respect to a 23-hectare
portion of the subject parcels of land pursuant to
the Memorandum of Agreement dated April 2,
1998 and Deed of Conditional Conveyance dated
August 13, 2004, which Vidal executed in favor of
AZIMUTH.
67 | P a g e

JURISDICTION; FOR CASE


DIGEST

Teofilo opposed the Petition contending that it


stated no cause of action because there was no
title being disturbed or in danger of being lost
due to the claim of a third party, and Vidal had
neither legal nor beneficial ownership of the
parcels of land in question; that the matter and
issues raised in the Petition had already been
tried, heard, and decided by the RTC of Iligan
City and affirmed with finality by this Court in
the 1997 Cacho case; and that the Petition was
barred by the Statute of Limitations and laches.

LANDTRADE, among other parties, was allowed


by the RTC-Branch 3 to intervene in Civil Case
No. 4452. LANDTRADE alleged that it is the
owner of a portion of the subject parcels of land,
measuring 270,255 square meters or about
27.03 hectares, which it purportedly acquired
through a Deed of Absolute Sale dated October
1, 1996 from Teofilo, represented by Atty.
Cabildo. LANDTRADE essentially argued that
Vidal's right as heir should be adjudicated upon
in a separate and independent proceeding and
not in the instant Quieting of Title Case.

Vidal and AZIMUTH submitted testimonial and


documentary evidence during the trial before the
RTC-Branch 3. Teofilo and Atty. Cabildo failed to
present any evidence as they did not appear at
all during the trial, while LANDTRADE was
declared by the RTC-Branch 3 to have waived its
right to present evidence on its defense and
counterclaim.

On July 17, 2004, the RTC-Branch 3 rendered its


Decision[33] in Civil Case No. 4452 in favor of Vidal
and AZIMUTH, the dispositive portion of which
reads:

WHEREFORE, judgment is hereby rendered in


favor of the petitioners and against the
respondents and intervenors:

1) DECLARING:

a.) Petitioner Demetria C. Vidal the sole surviving


heir of the late Doa Demetria Cacho;
During the pre-trial conference, the parties
manifested that there was no possibility of any
amicable settlement among them.

b.) Petitioner Demetria C. Vidal alone has the


hereditary right to and interest in the Subject
Property;
c.) Petitioner Azimuth International Development
Corporation is the successor-in-interest of
petitioner Demetria C. Vidal to a portion of the

Subject Property to the extent provided in their 2


April 1998 Memorandum of Agreement and 13
August 1998 Deed of Conditional Conveyance;
d.) Respondent Teofilo Cacho is not a son or heir
of the late Dona Demetria Cacho; and
e.) Respondent Teofilo Cacho, Godofredo Cabildo
and any of their transferees/assignees have no
valid right to or interest in the Subject Property.

2) ORDERING:

a.) Respondent Register of Deeds of Iligan City,


and any other person acting in his behalf, stop,
cease and desist:

i) From accepting or registering any affidavit of


self- adjudication or any other document
executed by respondents Teofilo Cacho,
Godofredo Cabildo and/or any other person
which in any way transfers the title to the
Subject Property from Dona Demetria Cacho to
respondent Teofilo Cacho, Godofredo Cabildo
and/or any of their transferees/assignees,
including the intervenors.
ii) From cancelling the OCTs or any certificate of
title over the Subject Property in the name of
Demetria Cacho or any successor certificate of
title, and from issuing new certificates of title in
the name of respondents Teofilo Cacho,
68 | P a g e

JURISDICTION; FOR CASE


DIGEST

Godofredo Cabildo their transferees/assignees,


including the intervenors.

b) Respondents Teofilo Cacho, Godofredo


Cabildo, their transferees/assignees, and any
other person acting in their behalf, to stop, cease
and desist:

3) ORDERING respondents Teofilo Cacho and


Atty. Godofredo Cabildo to pay petitioners, jointly
and severally, the following:

a) For temperate
damages
-

P 80,000.00

b) For nominal
damages
P 60,000.00
i) From executing, submitting to any Register of
Deeds, or registering or causing to be registered
therein, any affidavit of self-adjudication or any
other document which in any way transfers title
to the Subject Property from Demetria Cacho to
respondents Teofilo Cacho, Godofredo Cabildo
and/or any of their transferees/assignees,
including the intervenors.
ii) From canceling or causing the cancellation of
OCTs or any certificate of title over the Subject
Property in the name of Demetria Cacho or any
successor certificate of title, and from issuing
new certificates of title in the name of
respondent Teofilo Cacho, Godofredo Cabildo
and/or any of their transferees/assignees,
including the intervenors.
iii) From claiming or representing in any manner
that respondent Teofilo Cacho is the son or heir
of Demetria Cacho or has rights to or interest in
the Subject Property.

c) For moral
damages
-

Likewise, the counterclaim of intervenor


IDD/Investa is dismissed for lack of basis as the
petitioners succeeded in proving their cause of
action.

On the cross-claim of intervenor IDD/Investa,


respondents Teofilo Cacho and Atty. Godofredo
Cabildo are ORDERED to pay IDD/Investa, jointly
and severally, the principal sum of P5,433,036
with 15% interest per annum.

P500,000.00

d) For exemplary
damages
P 500,000.00

For lack of legal basis, the counterclaim of


Intervenor Landtrade Realty Development
Corporation is dismissed.

e) For attorney's fees (ACCRA Law)P1,000,000.00

fees

f) For Attorney's
P500,000.00

Likewise, Intervenor Manguera's counterclaim is


dismissed for lack of legal basis.[34]

(Atty. Voltaire Rovira)


g) For litigation
expenses
P300,000.00

For lack of factual and legal basis, the


counterclaim of Teofilo Cacho and Atty.
Godofredo Cabildo is hereby dismissed.

The joint appeal filed by LANDTRADE, Teofilo, and


Atty. Cabildo with the Court of Appeals was
docketed as CA-G.R. CV No. 00456. The Court of
Appeals, in its Decision[35] of January 19, 2007,
affirmed in toto the Decision dated July 17, 2004
of the RTC-Branch 3.

69 | P a g e

JURISDICTION; FOR CASE


DIGEST

According to the Court of Appeals, the RTCBranch 3 did not err in resolving the issue on
Vidals status, filiation, and hereditary rights as it
is determinative of the issue on ownership of the
subject properties. It was indubitable that the
RTC-Branch 3 had jurisdiction over the person of
Teofilo and juridical personality of LANDTRADE as
they both filed their Answers to the Petition for
Quieting of Title thereby voluntarily submitting
themselves to the jurisdiction of said trial
court. Likewise, the Petition for Quieting of Title
is in itself within the jurisdiction of the RTCBranch 3. Hence, where there is jurisdiction over
the person and subject matter, the resolution of
all other questions arising in the case is but an
exercise by the court of its
jurisdiction. Moreover, Teofilo and LANDTRADE
were guilty of estoppel by laches for failing to
assail the jurisdiction of the RTC-Branch 3 at the
first opportunity and even actively participating
in the trial of the case and seeking affirmative
reliefs.

In addition, the Court of Appeals held that


the 1997 Cacho case only determined the
validity and efficacy of the Affidavit of
Adjudication that Teofilo executed before the
Philippine Consulate General in the U.S.A. The
decision of this Court in the 1997 Cacho case,
which had become final and executory, did not
vest upon Teofilo ownership of the parcels of land
as it merely ordered the re-issuance of a lost
duplicate certificate of title in its original form
and condition.

The Court of Appeals agreed in the finding of the


RTC-Branch 3 that the evidence on record
preponderantly supports Vidals claim of being
the granddaughter and sole heiress of the late
Doa Demetria. The appellate court further
adjudged that Vidal did not delay in asserting her
rights over the subject parcels of land. The
prescriptive period for real actions over
immovables is 30 years. Vidals rights as Doa
Demetrias successor-in-interest accrued upon
the latters death in 1974, and only 24 years
thereafter, in 1998, Vidal already filed the
present Petition for Quieting of Title. Thus,
Vidals cause of action had not yet
prescribed. And, where the action was filed
within the prescriptive period provided by law,
the doctrine of laches was also inapplicable.

LANDTRADE, Teofilo, and Atty. Cabildo filed


separate Motions for Reconsideration of the
January 19, 2007 Decision of the Court of
Appeals, which were denied in the July 4, 2007
Resolution[36] of the same court.

On August 24, 2007, LANDTRADE filed with


this Court a Petition for Review
on Certiorari under Rule 45 of the Rules of Court,
which was docketed as G.R. No. 178779. On
September 6, 2007, Teofilo and Atty. Cabildo filed
their own Petition for Review on Certiorari under
Rule 45 of the Rules of Court, which was
docketed as G.R. No. 178894.

The Ejectment or Unlawful Detainer Case


(G.R. Nos. 170505, 173355-56, and 173563-64)

Three Petitions before this Court are rooted in the


Unlawful Detainer Case instituted by LANDTRADE
against NAPOCOR and TRANSCO.

On August 9, 1952, NAPOCOR took possession of


two parcels of land in Sitio Nunucan, Overton,
Fuentes, Iligan City, denominated as Lots 2029
and 2043, consisting of 3,588 square meters (or
0.36 hectares) and 3,177 square meters (or 0.32
hectares), respectively. On Lot 2029, NAPOCOR
constructed its power sub-station, known as the
Overton Sub-station, while on Lot 2043, it built a
warehouse, known as the Agus 7 Warehouse,
both for the use of its Agus 7 Hydro-Electric
Power Plant. For more than 30 years, NAPOCOR
occupied and possessed said parcels of land
pursuant to its charter, Republic Act No. 6395.
[37]
With the enactment in 2001 of Republic Act
No. 9136, otherwise known as the Electric Power
Industry Reform Act (EPIRA), TRANSCO assumed
the functions of NAPOCOR with regard to
electrical transmissions and took over possession
of the Overton Sub-station.

Claiming ownership of the parcels of land where


the Overton Sub-station and Agus 7 Warehouse
are located, LANDTRADE filed with the MTCC on
April 9, 2003 a Complaint for Unlawful Detainer
70 | P a g e

JURISDICTION; FOR CASE


DIGEST

against NAPOCOR and TRANSCO, which was


docketed as Civil Case No. 11475-AF.

In its Complaint, LANDTRADE alleged that it


acquired from Teofilo, through Atty. Cabildo, two
parcels of land at Sitio Nunucan, Overton,
Fuentes, Brgy. Maria Cristina,Iligan City, with a
combined area of 270,255 square meters or
around 27.03 hectares, as evidenced by a Deed
of Absolute Sale[38] dated October 1,
1996. Certain portions of said parcels of land
were being occupied by the Overton Sub-station
and Agus 7 Warehouse of NAPOCOR and
TRANSCO, through the tolerance of
LANDTRADE. Upon failure of NAPOCOR and
TRANSCO to pay rentals or to vacate the subject
properties after demands to do so, LANDTRADE
filed the present Complaint for Unlawful
Detainer, plus damages in the amount
of P450,000.00 as yearly rental from date of the
first extra-judicial demand until NAPOCOR and
TRANSCO vacate the subject properties.

In their separate Answers, NAPOCOR and


TRANSCO denied the material allegations in the
Complaint and countered, by way of special and
affirmative defenses, that the Complaint was
barred by res judicata; that the MTCC has no
jurisdiction over the subject matter of the action;
and that LANDTRADE lacked the legal capacity to
sue.

On February 17, 2004, the MTCC rendered its


Decision[39] in favor of LANDTRADE. The MTCC
disposed:

WHEREFORE, premises considered,


judgment is hereby rendered in favor of Plaintiff
Land Trade Realty Corporation represented by
Atty. Max C. Tabimina and against defendant
National Power Corporation represented by its
President, Mr. Rogelio M. Murga and codefendant TRANSCO represented by its President
Dr. Allan T. Ortiz and Engr. Lorrymir A. Adaza,
Manager, NAPOCOR-Mindanao, Regional Center,
Ma. Cristina, Iligan City, ordering:

1. Defendants National Power Corporation and


TRANSCO, their agents or representatives or any
person/s acting on its behalf or under its
authority to vacate the premises;

Execution shall issue immediately upon motion,


unless an appeal has been perfected and the
defendant to stay execution files a sufficient
supersedeas bond, approved by this Court and
executed in favor of the plaintiff, to pay the
rents, damages, and costs accruing down to the
time of judgment appealed from, and unless,
during the pendency of the appeal, defendants
deposit with the appellate court the amount
of P500,000.00 per month, as reasonable value
of the use and occupancy of the premises for the
preceding month or period on or before the tenth
day of each succeeding month or period.[40]

NAPOCOR and TRANSCO seasonably filed a Joint


Notice of Appeal. Their appeal, docketed as Civil
Case No. 6613, was initially assigned to the RTCBranch 5, presided over by Judge Maximino
Magno Libre (Judge Libre).

2. Defendants NAPOCOR and TRANSCO to pay


Plaintiff jointly and solidarily:

a. Php500,000.00 a month representing fair


rental value or compensation since June 29, 1978
until defendant shall have vacated the premises;
b. Php20,000.00 for and as attorneys fees and
c. Cost of suit.

LANDTRADE filed on June 24, 2004 a Motion for


Execution, asserting that NAPOCOR and
TRANSCO had neither filed a supersedeas bond
with the MTCC nor periodically deposited with the
RTC the monthly rental for the properties in
question, so as to stay the immediate execution
pending appeal of the MTCC
judgment. However, the said Motion failed to
comply with the required notice of hearing under
Rule 15, Section 5 of the Rules of
71 | P a g e

JURISDICTION; FOR CASE


DIGEST

Court. LANDTRADE then filed a Motion to


Withdraw and/or Replace Notice of Hearing.

NAPOCOR and TRANSCO filed on July 13, 2004 a


Joint Motion to Suspend Proceedings
citing Amagan v. Marayag,[41] in which the Court
ruled that if circumstances should require, the
proceedings in an ejectment case may be
suspended in whatever stage it may be
found. Since LANDTRADE anchors its right to
possession of the subject parcels of land on the
Deed of Sale executed in its favor by Teofilo on
October 1, 1996, the ejectment case should be
held in abeyance pending the resolution of other
cases in which title over the same properties are
in issue, i.e., (1) Civil Case No. 6600, the action
for the annulment of the Deed of Sale dated
October 1, 1996 filed by Teofilo against
LANDTRADE pending before the RTC-Branch 4;
and (2) Civil Case No. 4452, the Quieting of Title
Case filed by Vidal and AZIMUTH against Teofilo
and Atty. Cabildo pending before the RTC-Branch
3.

LANDTRADE filed on July 19, 2004 another


Motion for Execution, which was heard together
with the Joint Motion to Suspend Proceedings of
NAPOCOR and TRANSCO. After said hearing, the
RTC-Branch 5 directed the parties to file their
memoranda on the two pending Motions.

LANDTRADE, in its Memorandum, maintained


that the pendency of Civil Case No. 4452, the
Quieting of Title Case, should not preclude the
execution of the MTCC judgment in the Unlawful
Detainer Case because the issue involved in the
latter was only the material possession
or possession de facto of the parcels of land in
question. LANDTRADE also reported that Civil
Case No. 6600, the action for annulment of the
Deed of Sale dated October 1, 1996 instituted by
Teofilo, was already dismissed given that the
RTC-Branch 4 had approved the Compromise
Agreement executed between LANDTRADE and
Teofilo.

NAPOCOR and TRANSCO likewise filed their


respective Memoranda. Subsequently, NAPOCOR
filed a Supplement to its Memorandum to bring
to the attention of the RTC-Branch 5 the Decision
rendered on July 17, 2004 by the RTC-Branch 3 in
Civil Case No. 4452, the Quieting of Title Case,
categorically declaring Teofilo, the predecessorin-interest of LANDTRADE, as having no right at
all to the subject parcels of land. Resultantly, the
right of LANDTRADE to the two properties, which
merely emanated from Teofilo, was effectively
declared as non-existent too.

On August 4, 2004, the RTC-Branch 5 issued an


Order[42] denying the Joint Motion to Suspend
Proceedings of NAPOCOR and TRANSCO. The
RTC held that the pendency of other actions
involving the same parcels of land could not stay
execution pending appeal of the MTCC judgment

because NAPOCOR and TRANSCO failed to post


the required bond and pay the monthly rentals.

Five days later, on August 9, 2004, the RTCBranch 5 issued another Order[43] granting the
Motion of LANDTRADE for execution of the MTCC
judgment pending appeal.

The next day, on August 10, 2004, the Acting


Clerk of Court, Atty. Joel M. Macaraya, Jr., issued
a Writ of Execution Pending Appeal[44] which
directed Sheriff IV Alberto O. Borres (Sheriff
Borres) to execute the MTCC Decision dated
February 17, 2004.

A day later, on August 11, 2004, Sheriff


Borres issued two Notices of
Garnishment[45] addressed to PNB and Land Bank
of the Philippines in Iligan City, garnishing all the
goods, effects, stocks, interests in stocks and
shares, and any other personal properties
belonging to NAPOCOR and TRANSCO which were
being held by and under the possession and
control of said banks. On even date, Sheriff
Borres also issued a Notification[46] to NAPOCOR
and TRANSCO for them to vacate the subject
parcels of land; and to pay LANDTRADE the sums
of (a) P156,000,000.00, representing the total
fair rental value for the said properties,
computed at P500,000.00 per month, beginning
June 29, 1978 until June 29, 2004, or for a period

72 | P a g e

JURISDICTION; FOR CASE


DIGEST

of 26 years, and (b) P20,000.00 as attorney's


fees.

Thereafter, NAPOCOR and TRANSCO each


filed before the Court of Appeals in Cagayan de
Oro City a Petition for Certiorari, under Rule 65 of
the Rules of Court, with prayer for the issuance
of a TRO and writ of preliminary injunction. The
Petitions, docketed as CA-G.R. SP Nos. 85174 and
85841, were eventually consolidated.

The Court of Appeals issued on August 18, 2004


a TRO[47] enjoining the enforcement and
implementation of the Order of Execution and
Writ of Execution Pending Appeal of the RTCBranch 5 and Notices of Garnishment and
Notification of Sheriff Borres.

The Court of Appeals, in its Decision [48] dated


November 23, 2005, determined that public
respondents did commit grave abuse of
discretion in allowing and/or effecting the
execution of the MTCC judgment pending appeal,
since NAPOCOR and TRANSCO were legally
excused from complying with the requirements
for a stay of execution specified in Rule 70,
Section 19 of the Rules of Court, particularly, the
posting of a supersedeas bond and periodic
deposits of rental payments. The decretal
portion of said appellate court Decision states:

ACCORDINGLY, the two petitions at bench are


GRANTED; the Order dated 9 August 2004, the
Writ of Execution Pending Appeal dated 10
August 2004, the two Notices of Garnishment
dated 11 August 2004, and the Notification dated
11 August 2004, are ANNULLED and SET ASIDE.
[49]

Displeased, LANDTRADE elevated the case to


this Court on January 10, 2006 via a Petition for
Review on Certiorari under Rule 45 of the Rules
of Court, which was docketed as G.R. No.
170505.

In the meantime, with the retirement of Judge


Libre and the inhibition[50] of Judge Oscar
Badelles, the new presiding judge of RTC-Branch
5, Civil Case No. 6613 was re-raffled to the RTCBranch 1, presided over by Judge
Mangotara. The RTC-Branch 1 promulgated on
December 12, 2005 a Decision[51] in Civil Case
No. 6613 which affirmedin toto the February 17,
2004 Decision of the MTCC in Civil Case No.
11475-AF favoring LANDTRADE.

NAPOCOR and TRANSCO filed with the


RTC-Branch 1 twin Motions, namely: (1) Motion
for Reconsideration of the Decision dated
December 12, 2005; and (2) Motion for Inhibition
of Judge Mangotara. The RTC-Branch 1 denied

both Motions in a Resolution dated January 30,


2006.

NAPOCOR and TRANSCO filed with the Court of


Appeals separate Petitions for Review with prayer
for TRO and/or a writ of preliminary injunction,
which were docketed as CA-G.R. SP Nos. 00854
and 00889, respectively. In a Resolution dated
March 24, 2006, the Court of Appeals granted
the prayer for TRO of NAPOCOR and TRANSCO.

With the impending lapse of the effectivity


of the TRO on May 23, 2006, NAPOCOR filed on
May 15, 2006 with the Court of Appeals a
Manifestation and Motion praying for the
resolution of its application for preliminary
injunction.

On May 23, 2006, the same day the TRO


lapsed, the Court of Appeals granted the motions
for extension of time to file a consolidated
comment of LANDTRADE. Two days later,
LANDTRADE filed an Omnibus Motion seeking the
issuance of (1) a writ of execution pending
appeal, and (2) the designation of a special
sheriff in accordance with Rule 70, Section 21 of
the Rules of Court.

In a Resolution[52] dated June 30, 2006, the


Court of Appeals granted the Omnibus Motion of
73 | P a g e

JURISDICTION; FOR CASE


DIGEST

LANDTRADE and denied the applications for the


issuance of a writ of preliminary injunction of
NAPOCOR and TRANSCO. In effect, the appellate
court authorized the execution pending appeal of
the judgment of the MTCC, affirmed by the RTCBranch 1, thus:

IN LIGHT OF THE ABOVE DISQUISITIONS, this


Court resolves to grant the [LANDRADE]s
omnibus motion for execution pending appeal of
the decision rendered in its favor which is being
assailed in these consolidated petitions for
review. Accordingly, the [NAPOCOR and
TRANSCOs] respective applications for issuance
of writ of preliminary injunction are both denied
for lack of factual and legal bases. The Municipal
Trial Court in Cities, Branch 2, Iligan City, which
at present has the custody of the records of the
case a quo, is hereby ordered to cause the
immediate issuance of a writ of execution
relative to its decision dated 17 February 2004 in
Civil Case No. 11475-AF.[53]

On July 21, 2006, NAPOCOR filed an Urgent


Motion for the Issuance of a TRO in G.R. No.
173355-56. In a Resolution[54] dated July 26,
2006, the Court granted the Motion of NAPOCOR
and issued a TRO,[55] effective immediately,
which enjoined public and private respondents
from implementing the Resolution dated June 30,
2006 of the Court of Appeals in CA-G.R. SP Nos.
00854 and 00889 and the Decision dated
February 17, 2004 of the MTCC in Civil Case No.
11475-AF.

On July 31, 2006, Vidal and AZIMUTH filed


a Motion for Leave to Intervene and to Admit
Attached Comment-in-Intervention, contending
therein that Vidal was the lawful owner of the
parcels of land subject of the Unlawful Detainer
Case as confirmed in the Decision dated July 17,
2004 of the RTC-Branch 3 in Civil Case No.
4452. In a Resolution dated September 30,
2006, the Court required the parties to comment
on the Motion of Vidal and AZIMUTH, and
deferred action on the said Motion pending the
submission of such comments.
Issue: WON there was a grave abuse of
dicretion on the part of the judge

On July 20, 2006, NAPOCOR filed with this Court


a Petition for Certiorari and Prohibition under
Rule 65 of the Rules of Court with an urgent plea
for a TRO, docketed as G.R. No. 173355-56. On
August 2, 2006, TRANSCO filed with this Court its
own Petition for Certiorari, docketed as G.R. No.
173563-64.

Held: Jurisdiction vis--vis exercise of


jurisdiction
LANDTRADE, Teofilo, and/or Atty. Cabildo
argue that the RTC-Branch 3 had no
jurisidiction to resolve the issues of status,
filiation, and heirship in an action for quieting of
title as said issues should be ventilated and
adjudicated only in special proceedings under

Rule 90, Section 1 of the Rules of Court, pursuant


to the ruling of this Court in Agapay v.
Palang[84] (Agapay case) and Heirs of Guido
Yaptinchay and Isabel Yaptinchay v. Del
Rosario[85] (Yaptinchay case). Even on the
assumption that the RTC-Branch 3 acquired
jurisdiction over their persons, LANDTRADE,
Teofilo, and/or Atty. Cabildo maintain that the
RTC-Branch 3 erred in the exercise of its
jurisdiction by adjudicating and passing upon the
issues on Vidals status, filiation, and heirship in
the Quieting of Title Case. Moreover,
LANDTRADE, Teofilo, and/or Atty. Cabildo aver
that the resolution of issues regarding status,
filiation, and heirship is not merely a matter of
procedure, but of jurisdiction which cannot be
waived by the parties or by the court.

The aforementioned arguments fail to persuade.

In the first place, jurisdiction is not the


same as the exercise of jurisdiction. The Court
distinguished between the two, thus:

Jurisdiction is not the same as the exercise of


jurisdiction. As distinguished from the exercise
of jurisdiction, jurisdiction is the authority to
decide a cause, and not the decision rendered
therein. Where there is jurisdiction over the
person and the subject matter, the decision
on all other questions arising in the case is
but an exercise of the jurisdiction. And the
74 | P a g e

JURISDICTION; FOR CASE


DIGEST

errors which the court may commit in the


exercise of jurisdiction are merely errors of
judgment which are the proper subject of an
appeal.[86] (Emphasis supplied.)

SEC. 19. Jurisdiction in civil cases. Regional


Trial Courts shall exercise exclusive original
jurisdiction:

xxxx

Here, the RTC-Branch 3 unmistakably had


jurisdiction over the subject matter and the
parties in Civil Case No. 4452.

Jurisdiction over the subject matter or nature of


the action is conferred only by the Constitution
or by law. Once vested by law on a particular
court or body, the jurisdiction over the subject
matter or nature of the action cannot be
dislodged by anybody other than by the
legislature through the enactment of a law. The
power to change the jurisdiction of the courts is a
matter of legislative enactment, which none but
the legislature may do. Congress has the sole
power to define, prescribe and apportion the
jurisdiction of the courts.[87]

The RTC has jurisdiction over an action for


quieting of title under the circumstances
described in Section 19(2) of Batas Pambansa
Blg. 129, as amended:

(2) In all civil actions which involve the title to,


or possession of, real property, or any
interest therein, where the assessed value
of the property involved exceeds Twenty
thousand pesos (P20,000.00) or, for civil
actions in Metro Manila, where such value
exceeds Fifty thousand pesos (P50,000.00)
except actions for forcible entry into and
unlawful detainer of lands or buildings, original
jurisdiction over which is conferred upon the
Metropolitan Trial Courts, Municipal Trial Courts,
and Municipal Circuit Trial Courts.

Records show that the parcels of land subject of


Civil Case No. 4452 have a combined assessed
value of P35,398,920.00,[88] undisputedly falling
within the jurisdiction of the RTC-Branch 3.

The RTC-Branch 3 also acquired jurisdiction over


the person of Teofilo when he filed his Answer to
the Complaint of Vidal and AZIMUTH; and over
the juridical personality of LANDTRADE when the

said corporation was allowed to intervene in Civil


Case No. 4452.

Considering that the RTC-Branch 3 had


jurisdiction over the subject matter and parties in
Civil Case No. 4452, then it can rule on all issues
in the case, including those on Vidals status,
filiation, and heirship, in exercise of its
jurisdiction. Any alleged erroneous finding by
the RTC-Branch 3 concerning Vidals status,
filiation, and heirship in Civil Case No. 4452, is
merely an error of judgment subject to the
affirmation, modification, or reversal by the
appellate court when appealed.
The action for reconveyance is based on Section
55 of Act No. 496, otherwise known as the Land
Registration Act, as amended, which states
[t]hat in all cases of registration procured by
fraud the owner may pursue all his legal and
equitable remedies against the parties to such
fraud, without prejudice, however, to the rights
of any innocent holder for value of a certificate of
title.

The Court, in Heirs of Eugenio Lopez, Sr. v.


Enriquez,[94] described an action for
reconveyance as follows:

An action for reconveyance is an action in


personam available to a person
whose property has been wrongfully
75 | P a g e

JURISDICTION; FOR CASE


DIGEST

registered under the Torrens system in


anothers name. Although the decree is
recognized as incontrovertible and no longer
open to review, the registered owner is not
necessarily held free from liens. As a remedy, an
action for reconveyance is filed as anordinary
action in the ordinary courts of justice and not
with the land registration court. Reconveyance is
always available as long as the property has not
passed to an innocent third person for value. x x
x (Emphases supplied.)

On the other hand, Article 476 of the Civil Code


lays down the circumstances when a person may
institute an action for quieting of title:

ART. 476. Whenever there is a cloud on title to


real property or any interest therein, by reason of
any instrument, record, claim, encumbrance or
proceeding which is apparently valid or effective
but is in truth and in fact invalid, ineffective,
voidable, or unenforceable, and may be
prejudicial to said title, an action may be brought
to remove such cloud or to quiet the title.

An action may also be brought to prevent a cloud


from being cast upon title to real property or any
interest therein.

In Calacala v. Republic,[95] the Court


elucidated on the nature of an action to quiet
title:

Regarding the nature of the action filed


before the trial court, quieting of title is
a common law remedy for the removal of any
cloud upon or doubt or uncertainty with respect
to title to real property. Originating in equity
jurisprudence, its purpose is to secure x x x an
adjudication that a claim of title to or an interest
in property, adverse to that of the complainant,
is invalid, so that the complainant and those
claiming under him may be forever afterward
free from any danger of hostile claim. In an
action for quieting of title, the competent court is
tasked to determine the respective rights of
the complainant and other claimants, x x x
not only to place things in their proper place, to
make the one who has no rights to said
immovable respect and not disturb the other, but
also for the benefit of both, so that he who has
the right would see every cloud of doubt over the
property dissipated, and he could afterwards
without fear introduce the improvements he may
desire, to use, and even to abuse the property as
he deems best x x x . (Emphases supplied.)

The Court expounded further in Spouses Portic v.


Cristobal[96] that:

Suits to quiet title are characterized


as proceedings quasi in rem. Technically,
they are neither in rem nor in personam. In an
action quasi in rem, an individual is named as
defendant. However, unlike suits in rem, a quasi
in rem judgment is conclusive only between the
parties.

Generally, the registered owner of a property


is the proper party to bring an action to quiet
title. However, it has been held that this remedy
may also be availed of by a person other than
the registered owner because, in the Article
reproduced above, title does not necessarily
refer to the original or transfer certificate of title.
Thus, lack of an actual certificate of title to a
property does not necessarily bar an action to
quiet title. x x x (Emphases supplied.)

The Court pronounced in the Agapay and


Yaptinchay cases that a declaration of heirship
cannot be made in an ordinary civil
action such as an action for reconveyance, but
must only be made in a special proceeding, for
it involves the establishment of a status or right.

The appropriate special proceeding would have


been the settlement of the estate of the
76 | P a g e

JURISDICTION; FOR CASE


DIGEST

decedent. Nonetheless, an action for quieting of


title is also a special proceeding, specifically
governed by Rule 63 of the Rules of Court on
declaratory relief and similar remedies.
[97]
Actions for declaratory relief and other similar
remedies are distinguished from ordinary civil
actions because:

2.
In declaratory relief, the subject-matter is
a deed, will, contract or other written instrument,
statute, executive order or regulation, or
ordinance. The issue is the validity or
construction of these
documents. The relief sought is the declaration
of the petitioners rights and
duties thereunder.

The concept of a cause of action in ordinary civil


actions does not apply to declaratory relief as
this special civil action presupposes that there
has been no breach or violation of the
instruments involved. Consequently, unlike
other judgments, the judgment in an action for
declaratory relief does not essentially entail any
executional process as the only relief to be
properly granted therein is a declaration of the
rights and duties of the parties under the
instrument, although some exceptions have been
recognized under certain situations.[98]

Civil Case No. 4452 could not be considered an


action for reconveyance as it is not based on the
allegation that the two parcels of land, Lots 1
and 2, have been wrongfully registered in
another persons name. OCT Nos. 0-1200 (a.f.)
and 0-1201 (a.f.), covering the subject
properties, are still in Doa Demetrias
name. Vidal and Teofilo each claims to have
inherited the two parcels of land from the late
Doa Demetria as said decedents sole heir, but
neither Vidal nor Teofilo has been able to transfer
registration of the said properties to her/his
name as of yet.

Instead, Civil Case No. 4452 is indisputably an


action for quieting of title, a special proceeding
wherein the court is precisely tasked to
determine the rights of the parties as to a
particular parcel of land, so that the complainant
and those claiming under him/her may be
forever free from any danger of hostile
claim. Vidal asserted title to the two parcels of
land as Doa Demetrias sole heir. The cloud on
Vidals title, which she sought to have removed,
was Teofilos adverse claim of title to the same
properties, also as Doa Demetrias only
heir. For it to determine the rights of the parties
in Civil Case No. 4452, it was therefore crucial for
the RTC-Branch 3 to squarely make a finding as
to the status, filiation, and heirship of Vidal in
relation to those of Teofilo. A finding that one is
Doa Demetrias sole and rightful heir would
consequently exclude and extinguish the claim of
the other.

Even assuming arguendo that the proscription in


the Agapay and Yaptinchay cases against making
declarations of heirship in ordinary civil actions
also extends to actions for quieting of title, the
same is not absolute.

In Portugal v. Portugal-Beltran[99] (Portugal


case), the Court recognized that there are
instances when a declaration of heirship need
not be made in a separate special proceeding:

The common doctrine


in Litam, Solivio and Guilas in which the adverse
parties are putative heirs to the estate of a
decedent or parties to the special proceedings
for its settlement is that if the special
proceedings are pending, or if there are no
special proceedings filed but there is, under the
circumstances of the case, a need to file one,
then the determination of, among other issues,
heirship should be raised and settled in said
special proceedings. Where special proceedings
had been instituted but had been finally closed
and terminated, however, or if a putative heir
has lost the right to have himself declared in the
special proceedings as co-heir and he can no
longer ask for its re-opening, then an ordinary
civil action can be filed for his declaration as heir
in order to bring about the annulment of the
partition or distribution or adjudication of a
property or properties belonging to the estate of
the deceased.[100]

77 | P a g e

JURISDICTION; FOR CASE


DIGEST

In the Portugal case itself, the Court directed the


trial court to already determine petitioners
status as heirs of the decedent even in an
ordinary civil action, i.e., action for annulment of
title, because:

It appearing x x x that in the present case


the only property of the intestate estate of
Portugal is the Caloocan parcel of land, to still
subject it, under the circumstances of the case,
to a special proceeding which could be long,
hence, not expeditious, just to establish the
status of petitioners as heirs is not only
impractical; it is burdensome to the estate with
the costs and expenses of an administration
proceeding. And it is superfluous in light of the
fact that the parties to the civil casesubject of
the present case, could and had already in fact
presented evidence before the trial court which
assumed jurisdiction over the case upon the
issues it defined during pre-trial.

In fine, under the circumstances of the


present case, there being no compelling reason
to still subject Portugals estate to administration
proceedings since a determination of petitioners
status as heirs could be achieved in the civil case
filed by petitioners, the trial court should proceed
to evaluate the evidence presented by the
parties during the trial and render a decision
thereon upon the issues it defined during pretrial, x x x.[101]

Another case, Heirs of Teofilo Gabatan v. Court of


Appeals[102] (Gabatan case), involved an action
for recovery of ownership and possession of
property with the opposing parties insisting that
they are the legal heirs of the
deceased. Recalling the Portugal case, the Court
ruled:

Similarly, in the present case, there appears to


be only one parcel of land being claimed by the
contending parties as their inheritance from Juan
Gabatan. It would be more practical to dispense
with a separate special proceeding for the
determination of the status of respondent as the
sole heir of Juan Gabatan, specially in light of the
fact that the parties to Civil Case No. 89-092, had
voluntarily submitted the issue to the RTC and
already presented their evidence regarding the
issue of heirship in these proceeding. Also the
RTC assumed jurisdiction over the same and
consequently rendered judgment thereon.

In Fidel v. Court of Appeals[103] (Fidel


case), therein respondents, the heirs of the late
Vicente Espineli (Vicente) from his first marriage,
instituted an action to annul the sale of Vicentes
property to therein petitioners, the spouses
Fidel. The subject property was sold to

petitioners by Vicentes heirs from his second


marriage. Even though ones legitimacy can only
be questioned in a direct action seasonably filed
by the proper party, the Court held that it was
necessary to pass upon respondents relationship
to Vicente in the action for annulment of sale so
as to determine respondents legal rights to the
subject property. In fact, the issue of whether
respondents are Vicentes heirs was squarely
raised by petitioners in their Pre-Trial
Brief. Hence, petitioners were estopped from
assailing the ruling of the trial court on
respondents status.

In Civil Case No. 4452, Teofilo and/or Atty.


Cabildo themselves asked the RTC-Branch 3 to
resolve the issue of Vidal's legal or beneficial
ownership of the two parcels of land.[104] During
trial, Vidal already presented before the RTCBranch 3 evidence to establish her status,
filiation, and heirship. There is no showing that
Doa Demetria left any other property that would
have required special administration
proceedings. In the spirit of the Portugal,
Gabatan, and Fidel cases, the Court deems it
more practical and expeditious to settle the issue
on Vidals status, filiation, and heirship in Civil
Case No. 4452.

5 SECTION 19. Eminent Domain. A local


government unit may, through its chief executive
and acting pursuant to an ordinance, exercise
the power of eminent domain for public use, or
purpose, or welfare for the benefits of the poor
and the landless, upon payment of just
78 | P a g e

JURISDICTION; FOR CASE


DIGEST

compensation, pursuant to the provisions of the


Constitution and pertinent laws; Provided,
however, That the power of eminent domain may
not be exercised unless a valid and definite offer
has been previously made to the owner, and
such offer was not accepted: Provided, further,
That the local government unit may immediately
take possession of the property upon the filing of
the expropriation proceedings and upon making
a deposit with the proper court of at least fifteen
percent (15%) of the fair market value of the
property based on the current tax declaration of
the property to be expropriated: Provided, finally,
That the amount to be paid for the expropriated
property shall be determined by the proper
court, based on the fair market value at the time
of the taking of the property."
REPUBLIC V MANGOTARA
Facts: (Long and confusing case)

7 consolidated cases stemmed from the 1914


case of Cacho v. Government of the United
States (1914 Cacho case).
1914 Cacho Case
In the early 1900s, the late Dona Demetria
applied for the registration of 2 parcels of land in
the Municipality of Iligan, Moro Province (now
called Iligan City, Lanao Del Norte). Only the
Government opposed Doa Demetria's
applications for registration on the ground that
the two parcels of land were the property of the
United States and formed part of a military
reservation, generally known as Camp Overton.
The land registration court ruled that the
applicant Doa Demetria Cacho is owner of the
portion of land occupied and planted by the
deceased Datto Anandog only; and her

application as to all the rest of the land solicited


in said case is denied. Moreover, the applicant
should present the corresponding deed from
Datto Darondon on or before the abovementioned 30th day of March, 1913. Final
decision in these cases is reserved until the
presentation of the said deed and the new plan.
Dissatisfied, Doa Demetria appealed to the
Supreme Court. SC affirmed the LRC Decision.
83 years later, the Court was again called upon
to settle a matter concerning the registration of
the Lots in the case of Cacho v. CA.
1997 Cacho case
Teofilo Cacho (Teofilo), claiming to be the late
Doa Demetria's son and sole heir, filed before
the RTC a petition for

79 | P a g e

reconstitution of two original certificates of title (OCTs). RTC granted Teofilo's petition and ordered the reconstitution
and re-issuance of Decree Nos. 10364 and 18969. The original issuance of these decrees presupposed a prior
judgment that had become final.
CA reversed the RTC Decision. Teofilo appealed to the SC. The SC reversed the judgment of the CA and reinstated the
decision of the RTC approving the re-issuance of Decree Nos. 10364 and 18969. The Court found that such decrees had
in fact been issued and had attained finality, as certified by the Acting Commissioner, Deputy Clerk of Court III,
Geodetic Engineer, and Chief of Registration of the then Land Registration Commission. MR denied. Hence, the decrees
of registration were re-issued bearing new numbers and OCTs were issued for 2 parcels of land in Dona Demetrias
name.
THE ANTECENT FACTS OF THE PETITIONS AT BAR
The dispute did not end with the termination of the 1997 Cacho case. Another 4 cases involving the same parcels of
land were instituted before the trial courts during and after the pendency of the 1997 Cacho case. These cases are: (1)
Expropriation Case (2) Quieting of Title Case (3) Ejectment or Unlawful Detainer Case and (4) Cancellation of Titles and
Reversion Case. These cases proceeded independently of each other in the courts a quo until they reached the SC,
that consolidated the seven Petitions.
Note: Ill just discuss the expropriation issue, the case is very long with lots of different issues
The Complaint for Expropriation was originally filed by the Iron and Steel Authority (ISA), now the NSC, against Maria
Cristina Fertilizer Corporation (MCFC), and the latter's mortgagee, the Philippine National Bank (PNB). During the
existence of ISA, Pres. Marcos issued Presidential Proclamation No. 2239, reserving in favor of ISA a parcel of land in
Iligan City. MCFC occupied certain portions of this parcel of land. When negotiations with MCFC failed, ISA was
compelled to file a Complaint for Expropriation.
When the statutory existence of ISA expired during the pendency of Civil Case No. 106, the RTC-Branch 1 allowed the
substitution of the Republic for ISA as plaintiff in Civil Case No. 106.
Alleging that the lots involved in the 1997 Cacho case encroached and overlapped the parcel of land subject of the
case, Republic filed with the RTC a Motion for Leave to File Supplemental Complaint and to Admit the Attached
Supplemental Complaint, seeking to implead Teofilo Cacho and Demetria Vidal and their respective successors-ininterest, LANDTRADE and AZIMUTH. However, the RTC denied the Motion of the Republic for leave to file and to admit
its Supplemental Complaint. RTC agreed with MCFC that the Republic did not file any motion for execution of the
judgment of this Court in the ISA case. Since no such motion for execution had been filed within the prescriptive period
of 5 years, RTC ruled that its Order dated November 16, 2001, which effected the substitution of the Republic for ISA as
plaintiff in the case, was an honest mistake. MR of the Republic denied because MCFC (the only defendant left in the
case) is NOT a proper party defendant in the complaint for expropriation. Hence, the case was dismissed. The Republic
filed with the SC the consolidated Petition for Review on Certiorari and Petition for Certiorari under Rules 45 and 65.
Issues:
1. Who are the proper parties in an expropriation proceeding?
2. W/N forum shopping was committed by the Republic with the filing of the expropriation and reversion complaint

First Issue:

80 | P a g e

The court ruled that defendants in an expropriation case are NOT limited to the owners of the property to be
expropriated, and just compensation is not due to the property owner alone. They include all other persons owning,
occupying or claiming to own the property. In the American jurisdiction, the term 'owner' when employed in statutes
relating to eminent domain to designate the persons who are to be made parties to the proceeding, refer, as is the rule
in respect of those entitled to compensation, to all those who have lawful interest in the property to be condemned,
including a mortgagee, a lessee and a vendee in possession under an executory contract. Every person having an
estate or interest at law or in equity in the land taken is entitled to share in the award. If a person claiming an interest
in the land sought to be condemned is not made a party, he is given the right to intervene and lay claim to the
compensation.
At the time of the filing of the Complaint for Expropriation, possessory/occupancy rights of MCFC over the parcels of
land sought to be expropriated were undisputed. Letter of Instructions No. 1277 expressly recognized that portions of
the lands reserved by Presidential Proclamation No. 2239 for the use and immediate occupation by the NSC, were then
occupied by an idle fertilizer plant/factory and related facilities of MCFC. It was ordered in the same Letter of
Instruction that NSC shall negotiate with the owners of MCFC, for and on behalf of the Government, for the
compensation of MCFC's present occupancy rights on the subject lands. Being the occupant of the parcel of land
sought to be expropriated, MCFC could very well be named a defendant in the case. The RTC evidently
erred in dismissing the Complaint for Expropriation against MCFC for not being a proper party. Also
erroneous was the dismissal by the RTC of the original Complaint for Expropriation for having been filed
only against MCFC, the occupant of the subject land, but not the owner/s of the said property. Dismissal
is not the remedy for misjoinder or non-joinder of parties.
The owner of the property is not necessarily an indispensable party in an action for expropriation. According to Rule
67, Section 1, expropriation proceedings may be instituted even when "title to the property sought to be condemned
appears to be in the Republic of the Philippines, although occupied by private individuals." The same rule provides that
a complaint for expropriation shall name as defendants "all persons owning or claiming to own, or occupying, any part
thereof or interest" in the property sought to be condemned. Clearly, when the property already appears to
belong to the Republic, there is no sense in the Republic instituting expropriation proceedings against
itself. It can still, however, file a complaint for expropriation against the private persons occupying the
property. In such an expropriation case, the owner of the property is not an indispensable party.
To recall, Presidential Proclamation No. 2239 explicitly states that the parcels of land reserved to NSC are part of the
public domain, hence, owned by the Republic. Letter of Instructions No. 1277 recognized only the occupancy rights of
MCFC and directed NSC toinstitute expropriation proceedings to determine the just compensation for said occupancy
rights. Therefore, the owner of the property is not an indispensable party in the original Complaint for Expropriation.
Moreover, the right of the Republic to be substituted for ISA as plaintiff in Civil Case No. 106 had long been affirmed by
no less than this Court in the ISA case. The failure of the Republic to actually file a motion for execution does not
render the substitution void. A writ of execution requires the sheriff or other proper officer to whom it is directed to
enforce the terms of the writ. The Order of the RTC should be deemed as voluntary compliance with a final and
executory judgment of this Court, already rendering a motion for and issuance of a writ of execution superfluous.
Second Issue: The Republic did not commit Forum shopping
Forum-shopping takes place when a litigant files multiple suits involving the same parties, either simultaneously or
successively, to secure a favorable judgment. Thus, it exists where the elements of litis pendentia are present, namely:
(a) identity of parties, or at least such parties who represent the same interests in both actions; (b) identity of rights
asserted and relief prayed for, the relief being founded on the same facts; and (c) the identity with respect to the two
preceding particulars in the two cases is such that any judgment that may be rendered in the pending case, regardless
of which party is successful, would amount to res judicata in the other case.
Here, the elements of litis pendencia are wanting. There is no identity of rights asserted and reliefs prayed for in Civil
Case No. 106 (expropriation) and Civil Case No. 6686 (cancellation of OCTs of Dona Demetria because the certificates
exceeded the areas granted by the LRC reversion).
81 | P a g e

Expropriation vis--vis reversion


The Republic is not engaging in contradictions when it instituted both expropriation and reversion proceedings for the
same parcels of land. The expropriation and reversion proceedings are distinct remedies that are not necessarily
exclusionary of each other. The filing of a complaint for reversion does not preclude the institution of an action for
expropriation. Even if the land is reverted back to the State, the same may still be subject to expropriation as against
the occupants thereof.
Also, Rule 67, Section 1 of the Rules of Court allows the filing of a complaint for expropriation even when "the title to
any property sought to be condemned appears to be in the Republic of the Philippines, although occupied by private
individuals, or if the title is otherwise obscure or doubtful so that the plaintiff cannot with accuracy or certainty specify
who are the real owners."
Hence, the filing by the Republic of the Supplemental Complaint for Expropriation impleading Teofilo, Vidal,
LANDTRADE, and AZIMUTH, is not necessarily an admission that the parcels of land sought to be expropriated are
privately owned. At most, the Republic merely acknowledged in its Supplemental Complaint that there are private
persons also claiming ownership of the parcels of land. The Republic can still consistently assert, in both actions for
expropriation and reversion, that the subject parcels of land are part of the public domain.
In sum, the RTC erred in dismissing the original Complaint and disallowing the Supplemental Complaint. The Court
reinstates the Complaint for Reversion of the Republic.
Biaco vs. Phil Country Rural Bank
15 SCRA 106 Civil Procedure In rem vs In personam proceedings Service of Summons Resident Defendant
Extrinsic Fraud
Ernesto Biaco, husband of Teresa Biaco, acquired several loans from Philippine Countryside Rural Bank (PCRB) from
1996 to 1998. To secure the loans, he mortgaged certain property in favor of the bank. He was able to pay loans from
1996 to 1997 but he defaulted in loans obtained in 1998 which amounted to more than a million pesos.
Eventually, PCRB filed a complaint for foreclosure against the spouses Biaco. Summons were issued by the trial judge.
The Sherriff served the summons to Ernesto at the latters office. No summons was served to Teresa.
Ernesto did not file a responsive pleading (so did Teresa because she was not aware sans the summons being served
her). The case was heard ex-parte and the spouses were ordered to satisfy the debt and failure to do so will authorize
the Sheriff to auction the mortgaged the property.
Eventually, the mortgaged property was auctioned for P150k which is not sufficient to cover the P1 M+ debt. Upon
motion by PCRB, a notice of levy was issued against the personal properties of Teresa to satisfy the deficiency.
It was only at this point that Teresa learned of the previous ex parte proceedings. She then sought to have the
judgment annulled as she now claims that she was deprived of due process when she did not receive summons; that it
was only her husband who received the summons; that there was extrinsic fraud because her husband deliberately hid
the fact of the foreclosure proceeding.
PRCB argued that the foreclosure proceeding is an action quasi in rem, hence Teresas participation is not required so
long as the court acquires jurisdiction over the res which is what happened in the case at bar; that Teresa cannot
invoke extrinsic fraud because such situation cannot occur in her case because she is a co-defendant of Ernesto.
ISSUE: Whether or not the judgment of the trial court should be annulled.

82 | P a g e

HELD: Yes. It is admitted that the proceeding is a quasi in rem proceeding and that the presence of Teresa is not
required because the trial court was able to acquire jurisdiction over the res (mortgaged property). HOWEVER, her
constitutional right to due process is superior over the procedural matters mentioned. Her right to due process was
violated when she did not receive summons. Teresa, as a resident defendant, who does not voluntary appear in court
must be personally served with summons as provided under Section 6, Rule 14 of the Rules of Court. Even if the
action is quasi in rem, personal service of summons is essential in order to afford her due process. The substituted
service made by the sheriff at her husbands office cannot be deemed proper service absent any explanation that
efforts had been made to personally serve summons upon her but that such efforts failed. Further, the order of the trial
court compelling Teresa to pay off the debt using her personal property is a judgment in personam which the court
cannot do because it only acquired jurisdiction over the res and not over the person of Teresa.
On the issue of extrinsic fraud, the Court of Appeals, agreeing with PCRB, is correct that there is none in the case at
bar. Extrinsic fraud exists when there is a fraudulent act committed by the prevailing party outside of the trial of the
case, whereby the defeated party was prevented from presenting fully his side of the case by fraud or deception
practiced on him by the prevailing party. Extrinsic fraud is present where the unsuccessful party had been prevented
from exhibiting fully his case, by fraud or deception practiced on him by his opponent, as by keeping him away from
court, a false promise of a compromise; or where the defendant never had knowledge of the suit, being kept in
ignorance by the acts of the plaintiff; or where an attorney fraudulently or without authority assumes to represent a
party and connives at his defeat; or where the attorney regularly employed corruptly sells out his clients interest to
the other side. The above is not applicable in the case of Teresa. It was not PCRB which made any fraud. It should be
noted that spouses Biaco were co-defendants in the case and shared the same interest.
Velayo Fong vs Velayo
Facts: On August 9, 1993, Raymond Velayo (Raymond) and his wife, Maria Hedy Velayo (respondents) filed a
complaint for sum of money and damages with prayer for preliminary attachment against Erlinda R. Velayo-Fong
(petitioner), Rodolfo R. Velayo, Jr. (Rodolfo Jr.) and Roberto R. Velayo (Roberto). [3] Raymond is the half-brother of
petitioner and her co-defendants.
In their Complaint, respondents allege that petitioner, a resident of 1860 Alamoana Boulevard, Honolulu, Hawaii, USA,
and her co-defendants, who are residents of the Philippines, made it appear that their common father, Rodolfo Velayo,
Sr. (Rodolfo Sr.) and petitioner had filed a complaint against Raymond before the National Bureau of Investigation
(NBI), accusing Raymond of the crimes of estafa and kidnapping a minor; that petitioner and her co-defendants also
requested that respondents be included in the Hold Departure List of the Bureau of Immigration and Deportation (BID)
which was granted, thereby preventing them from leaving the country and resulting in the cancellation of respondents
trips abroad and caused all of respondents business transactions and operations to be paralyzed to their damage and
prejudice; that petitioner and her co-defendants also filed a petition before the Securities and Exchange Commission
(SEC) docketed as Case No. 4422 entitled Rodolfo Velayo Sr. et al. v. Raymond Velayo et al. which caused
respondents funds to be frozen and paralyzed the latters business transactions and operations to their damage and
prejudice. Since petitioner was a non-resident and not found in the Philippines, respondents prayed for a writ of
preliminary attachment against petitioners properties located in the Philippines.
Before respondents application for a writ of preliminary attachment can be acted upon by the RTC, respondents filed
on September 10, 1993 an Urgent Motion praying that the summons addressed to petitioner be served to her at Suite
201, Sunset View Towers Condominium, Roxas Boulevard, Pasay City and at No. 5040 P. Burgos Street, T. Towers
Condominium, Makati.[4] In its Order dated September 13, 1993, the RTC granted the said motion.
Upon ex-parte motions[7] of respondents, the RTC in its Order dated November 23, 1993 and January 5, 1994, declared
petitioner and her co-defendant in default for failure to file an answer and ordered the ex-parte presentation of
respondents evidence. On September 1, 1994, petitioner filed a Motion to Set Aside Order of Default claiming that she
was prevented from filing a responsive pleading and defending herself against respondents complaint because of
fraud, accident or mistake; that contrary to the Officers Return, no summons was served upon her; that she has valid
and meritorious defenses to refute respondents material allegations. [10] Respondents opposed said Motion.[11]

83 | P a g e

In its Order dated May 29, 1995, the RTC denied petitioners Motion ruling that the presumption of regularity in the
discharge of the function of the Process Server was not sufficiently overcome by petitioners allegation to the contrary;
that there was no evident reason for the Process Server to make a false narration regarding the service of summons to
defaulting defendant in the Officers Return. [12]

On September 4, 1995, respondents filed a Motion for Execution. [13] On September 22, 1995, petitioner filed an
Opposition to Motion for Execution contending that she has not yet received the Decision and it is not yet final and
executory as against her.[14]

In its Order dated January 3, 1996, the RTC, finding that the Decision dated June 15, 1994 and the Order
dated May 29, 1995 were indeed not furnished or served upon petitioner, denied respondents motion for execution
against petitioner and ordered that petitioner be furnished the said Decision and Order.[15]

On March 28, 1996, the RTC issued an Order directing the issuance of the writ of execution against petitioners codefendant.[16]

On May 23, 1996, petitioner, through her counsel, finally received the Decision dated June 15, 1994 and the Order
dated May 29, 1995.[17]

Petitioner filed an appeal with the CA questioning the propriety and validity of the service of summons made upon
her. Respondents opposed the appeal, arguing that the petition should be dismissed since it raised pure questions of
law, which is not within the CAs jurisdiction to resolve under Section 2 (c) of Rule 41 of the Revised Rules of Court;
that, in any case, petitioners reliance on the rule of extraterritorial service is misplaced; that the judgment by default
has long been final and executory since as early as August 1994 petitioner became aware of the judgment by default
when she verified the status of the case; that petitioner should have filed a motion for new trial or a petition for relief
from judgment and not a motion to set aside the order of default since there was already a judgment by default.
On May 14, 2002, the CA rendered its Decision affirming the Decision and Order of the RTC [18] ruling that it (CA) has
jurisdiction since the petition raised a question of fact, that is, whether petitioner was properly served with summons;
that the judgment by default was not yet final and executory against petitioner since the records reveal and the RTC
Order dated January 3, 1996 confirmed that she was not furnished or served a copy of the decision; that petitioner was
validly served with summons since the complaint for damages is an action in personam and only personal, not
extraterritorial service, of summons, within the forum, is essential for the acquisition of jurisdiction over her person;
that petitioners allegations that
she did not know what was being served upon her and that somebody just hurled papers at her were not substantiated
by competent evidence and cannot overcome the presumption of regularity of performance of official functions in
favor of the Officers Return.
Petitioner filed a Motion for Reconsideration[19] but the CA denied it in its Resolution dated October 1, 2002.
84 | P a g e

Petitioner argues that summons should have been served through extraterritorial service since she is a non-resident;
that the RTC should have lifted the order of default since a default judgment is frowned upon and parties should be
given their day in court; that she was prevented from filing
a responsive pleading and defending against respondents complaint
through fraud, accident or mistake considering that the statement in the Officers Return that she was personally
served summons is inaccurate; that
she does not remember having been served with summons during the said date but remembers that a man hurled
some papers at her while she was entering the elevator and, not knowing what the papers were all about, she threw
back the papers to the man before the elevator closed; that she has a valid and meritorious defense to refute the
material allegations of respondents complaint.

On the other hand, respondents contend that petitioner was validly served with summons since the rules do not
require that service be made upon her at her place of residence as alleged in the complaint or stated in the summons;
that extraterritorial service applies only when the defendant does not reside and is not found in the Philippines; that
petitioner erred in filing a motion to set aside the order of default at the time when a default judgment was already
rendered by the RTC since the proper remedy is a motion for new trial or a petition for relief from judgment under Rule
38; that the issue on summons is a pure question of law which the CA does not have jurisdiction to resolve under
Section 2 (c) of Rule 41 of the 1997 Rules of Civil Procedure.
Issue: WON summons was properly exercised by CA
Held: How may service of summons be effected on a non-resident?
Section 17,[28] Rule 14 of the Rules of Court provides:
Section 17. Extraterritorial service When the defendant does not
reside and is not found in the Philippines and the action affects the
personal status of the plaintiff or relates to, or the subject of which, is property within the Philippines, in which the
defendant has or claims a
lien or interest, actual or contingent, or in which relief demanded consists, wholly or in part, in excluding the defendant
from any interest therein, or the property of the defendant has been attached in the Philippines, service may,
by leave of court, be effected out of the Philippines by personal service as under section
7; or by publication in a newspaper of general
circulation in such places and for such time as the court may order, in which case a copy of the summons and order of
the court shall be sent by registered mail to the last known address of the defendant, or in any other manner the court
may deem sufficient. Any order granting such leave shall
specify a reasonable time, which shall not be less than sixty (60) days after notice, within which the defendant must
answer.

Under this provision, when the defendant is a nonresident and he is not found in the country, summons may be served
extraterritorially. There are only four instances when extraterritorial service of summons is proper, namely: (a) when
the action affects the personal status of the plaintiffs; (b) when the action relates to, or the subject of which is
property, within the Philippines, in which the defendant claims a lien or interest, actual or contingent; (c) when the
85 | P a g e

relief demanded in such action consists, wholly or in part, in excluding the defendant from any interest in property
located in the Philippines; and (d) when the defendants property has been attached within the Philippines. In these
instances, service of summons may be effected by (a) personal service out of the country, with leave of court; (b)
publication, also with leave of court; or (c) any other manner the court may deem sufficient.

Thus, extrajudicial service of summons apply only where the action is in rem, that is, an action against the thing itself
instead of against the person, or in an action quasi in rem, where an individual is named as defendant and the purpose
of the proceeding is to subject his interest therein to the obligation or loan burdening the property. The rationale for
this is that in in rem and quasi in rem actions, jurisdiction over the person of the defendant is
not a prerequisite to confer jurisdiction on the court provided that the court acquires jurisdiction over the res.[29]

Where the action is in personam, that is, one brought against a person
on the basis of her personal liability, jurisdiction over the person of the defendant is necessary for the court to
validly try and decide the case. When the defendant is a non-resident, personal service of summons within the state is
essential to the acquisition of jurisdiction over the person. [30] Summons on the defendant must be served by handing a
copy thereof to the defendant in person, or, if he refuses to receive it, by tendering it to him. [31]This cannot be done,
however, if the defendant is not physically present in the country, and thus, the court cannot acquire jurisdiction over
his person and therefore cannot validly try and decide the case against him. [32]

In the present case, respondents cause of action in Civil Case No. Q-93-17133 is anchored on the claim that petitioner
and her co-defendants maliciously instituted a criminal complaint before the NBI and a petition before the SEC which
prevented the respondents from leaving the country and paralyzed the latters business transactions. Respondents
pray that actual and moral damages, plus attorneys fees, be awarded in their favor. The action instituted by
respondents affect the parties alone, not the whole world. Any judgment therein is binding only upon the parties
properly impleaded.[33] Thus, it is an action in personam. As such, personal service of summons upon the defendants is
essential in order for the court to acquire jurisdiction over their persons. [34]

The Court notes that the complaint filed with the RTC alleged that petitioner is a non-resident who is not found in
the Philippines for which reason respondents initially prayed that a writ of preliminary attachment be issued against
her properties within the Philippines to confer jurisdiction upon the RTC. However, respondents did not pursue its
application for said writ when petitioner was subsequently found physically present in the Philippines and personal
service of summons was effected on her.

Was there a valid service of summons on petitioner? The answer is in the affirmative.

Petitioners bare allegation that the statement in the Officers Return that she was personally served summons is
inaccurate is not sufficient. A process servers certificate of service is prima facie evidence of the facts as set out in
the certificate.[35] Between the claim of non-receipt of summons by a party against the assertion of an official whose
duty is to send notices, the latter assertion is fortified by the presumption that official duty has been regularly
performed.[36] To overcome the presumption of regularity of performance of official functions in favor of such Officers
86 | P a g e

Return, the evidence against it must be clear and convincing. Petitioner having been unable to come forward with the
requisite quantum of proof to the contrary, the presumption of regularity of performance on the part of the process
server stands.

The Court need not make a long discussion on the propriety of the remedy adopted by petitioner in the RTC of filing a
motion to set aside the order of default at a time when there was already a judgment by default. As aptly held by the
CA, since petitioner was not furnished or served a copy of the judgment of default, there was no notice yet of such
judgment as against her. Thus, the remedy of filing a motion to set aside the order of default in the RTC was proper.

Petitioners argument that the RTC should have set aside the order of default and applied the liberal interpretation of
rules with a view of affording parties their day in court is not tenable. While indeed default orders are not viewed with
favor, the party seeking to have the order of default lifted must
first show that her failure to file an answer or any other responsive pleading was due to fraud, accident, mistake, or
excusable neglect and then she must show that she has a valid and meritorious defense. [37]

In this case, petitioner failed to show that her failure to file an answer was due to fraud, accident, mistake or excusable
neglect. Except for her bare unsupported allegation that the summons were only thrown to her at the elevator,
petitioner did not present any competent evidence to justify the setting aside of the order of default.

Moreover, when a party files a motion to lift order of default, she must also show that she has a meritorious defense or
that something would be gained by having the order of default set aside. [38] The term meritorious defense implies that
the applicant has the burden of proving such a defense in order to have the judgment set aside. The cases usually do
not require such a strong showing. The test employed appears to be essentially the same as used in considering
summary judgment, that is, whether there is enough evidence to present an issue for submission to the trier
of fact, or a showing that on the undisputed facts it is not clear that the judgment is warranted as a matter of
law. [39] The defendant must show that she has a meritorious defense otherwise the grant of her motion will prove to
be a useless exercise. Thus, her motion must be accompanied by a statement of the evidence which she
intends to present if the motion is granted and which is such as to warrant a reasonable belief that the
result of the case would probably be otherwise if a new trial is granted.
In the present case, petitioner contented herself with stating in her affidavit of merit that the cases against respondent
Raymond were filed at the instance of her father. [41]Such allegation is a conclusion rather than a statement of facts
showing a meritorious defense. The affidavit failed to controvert the facts alleged by the respondents. Petitioner
has not shown that she has a meritorious defense.
Thus, since petitioner failed to show that her failure file an answer was not due to fraud, accident, mistake, or
excusable neglect; and that she had a valid and meritorious defense, there is no merit to her prayer for a liberal
interpretation of procedural rules.
WHEREFORE, the instant petition is DENIED. The assailed Decision and Resolution of the Court of Appeals
are AFFIRMED.

87 | P a g e

G.R. No. 170783 June 18, 2012


LEGASPI TOWERS 300, INC., LILIA MARQUINEZ PALANCA, ROSANNA D. IMAI, GLORIADOMINGO and RAY
VINCENT,
Petitioners,vs.
AMELIA P. MUER, SAMUEL M. TANCHOCO, ROMEO TANKIANG, RUDEL PANGANIBAN,DOLORES AGBAYANI,
ARLENEDAL A. YASUMA, GODOFREDO M. CAGUIOA and EDGARDO M.SALANDANAN,
Respondents.
FACTS:
Pursuant to the by-laws of Legaspi Towers 300, Inc., petitioners Lilia Marquinez Palanca, Rosanna D. Imai, Gloria
Domingo and Ray Vincent, the incumbent Board of Directors, set the annual meeting of the members of the
condominium corporation and the election of the new Board of Directors at the lobby of Legaspi Towers 300, Inc. The
Committee on Elections of Legaspi Towers 300, Inc., however, found most of the proxy votes, at its face value,
irregular, thus, questionable; and for lack of time to authenticate the same, petitioners adjourned the meeting for lack
of quorum. However, the group of respondents challenged the adjournment of the meeting. Despite petitioners'
insistence that no quorum was obtained during the annual meeting held on April 2, 2004, respondents pushed through
with the scheduled election and were elected as the new Board of Directors and officers of Legaspi Towers 300, Inc.
and subsequently submitted a General Information Sheet to the Securities and Exchange Commission (SEC).
On plaintiffs motion to admit amended complaint (to include Legaspi Towers 300, Inc. as plaintiff),the RTC ruled
denying the motion for being improper. Then, petitioners filed with the Court of Appeals and held that Judge Antonio I.
De Castro of the Regional Trial Court (RTC) of Manila, did not commit grave abuse of discretion in issuing the Orders
denying petitioners Motion to Admit Second Amended Complaint and that petitioners the justified the inclusion of
Legaspi Towers 300, Inc. as plaintiff by invoking the doctrine of derivative suit.
Petitioners motion for reconsideration was denied by the Court of Appeals thereafter. Hence, this petition.
ISSUE:
Whether or not Derivative Suit proper in this case.
RULING:
The Supreme Court DENIED the petition and AFFIRMED the Decision of the Court of Appeals.Derivative Suit is not
applicable.Since it is the corporation that is the real party-in-interest in a derivative suit, then the reliefs prayed
for must be for the benefit or interest of the corporation.
When the reliefs prayed for do not pertain to the corporation, then it is an improper derivative suit. The requisites for
a derivative suit are as follows:
a) the party bringing suit should be a shareholder as of the time of the act or transaction complained of, the number of
his shares not being material; b) he has tried to exhaust intra-corporate remedies, i.e., has made a demand on the
board of directors for the appropriate relief but the latter has failed or refused to heed his plea; and c) the cause of
action actually devolves on the corporation, the wrongdoing or harm having been, or being caused to the corporation
and not to the particular stockholder bringing the suit.
As stated by the Court of Appeals, petitioners complaint seek to nullify the said election, and to protect and enforce
their individual right to vote. The cause of action devolves on petitioners, not the condominium corporation, which did
88 | P a g e

not have the right to vote. Hence, the complaint for nullification of the election is a direct action by petitioners, who
were the members of the Board of Directors of the corporation before the election, against respondents, who are the
newly-elected Board of Directors. Under the circumstances, the derivative suit filed by petitioners in behalf of
the condominium corporation in the Second Amended Complaint is improper.

89 | P a g e

You might also like